Ευκλειδης Β 75

84

description

Ευκλειδης Β 75

Transcript of Ευκλειδης Β 75

Page 1: Ευκλειδης Β 75
Page 2: Ευκλειδης Β 75

Επιμέλεια: Αντώνης Κυριακόπουλος Γιώργος Τασσόπουλος

Προτεινόμενες Ασκήσεις

Για την Α' τάξη. AS. Έστω ότι: α2 + β2 + γ2 + δ2 +α β+ γ δ= 1. Να αποδείξετε ότι: αδ-βγ -:1- 1.

Για την Β'τάξη. Β4. Να λυθεί η εξίσωση: 9σuv2x + 9σuv'x = 4. Για την Γ τάξη. Γ3. Μια συνάρτηση f είναι ορισμένη και τρεις φορές παραγωγίσιμη στο διάστημα Δ= (α, β) . Υποθέτουμε ότιf(χ) �Ο, για κάθε χ ε Δ και ότι η fέχει δύο (διαφορετικές) ρίζες στο Δ. Να αποδείξετε ότι η εξίσωση: f'"(x) =Ο έχει μία τουλάχιστον ρίζα στο Δ. Λύσεις Προηγούμενου τεύχους 74. Α4. Αν ΚΔ+ΔΓ+ΓΛ�ΚΑ+ΑΒ+ΒΛ, τότε

π ΚΔ+ΔΓ+ΓΛ�-. 4

Έστω Μ σημείο της περιμέτρου του ΚΔΓ Λ.

Αρκεί να δείξουμε ότι: π ΟΜ<-. 4

π , ΟΜ ΜΚ+ΜΛ ΚΔ+ΔΓ+ΓΛ Π ραγματι: � < �-2 2 4

Άρα: ΟΜ < Π

. Χρ. Παλυβός 4° Λύκ. Πετρούπολης 4

Β3. ΜΝ//ΔΒ=?ΑΓ μεσοκάθετος των

ΜΝ, ΕΖ=?ΜΕ=ΝΖ=χ. z

Εξάλλου: ΜΝ = ΔΒ =ρ= 1, 2

ΑΒ=λ =ρJ2 =J2 =? ΜΑ=ΜΒ= J2 4 2 '

οπότε: ME·MZ�MA·MB=>x (x+l) �( �J' =>

.J3-1 .J3-1 2χ 2 + 2χ -1 =Ο::::? χ = -- ::::? ΜΕ=-- ,

2 2 .J3-1 .J3+1 Λ

ΜΖ=--+1=--. Όμως Μ1 =45°, οπότε: 2 2

(ΕΒ)2 =( ΜΕ)2 +(ΜΒ)2 -2ΜΕ·ΜΒσυν45σ =

( .J3 -1:

2 + ( J2 J2 -2 . .J3 -1 . J2 . J2 =

2 2 2 2 2

4 -2.J3 1 .J3-1 2-.J3 1 .J3-1 ---+----=--+----= 4 2 2 2 2 2

4-2.J3 (.J3-1)2 .J3-1 J6-J2 2 2

(ΕΒ) = J2 =

2 .

Έχουμε: (EZ)=(ME)+(MZ)=.J3 =ρ.J3 =λ3

οπότε ΕΟΖ = &3 = 120°

Άρα: (ΜΑΕΜ) =�[(Ο· m) -(ΟΕΖ)-( ΑΜΝ)] =

1 [ 2 120 1 2 ο 1 ( J2 J2] =2

πρ 360 -2ρ ημ120 -2 2

=

� Ξ(�-� -±J��- 13s+1

Γ. Κλάδης Σχολή Αυγουλέα- Λιναρδάτου Άσκηση Α2. i) Έχουμε:, α +β+ γ = Ο ::::? α = -β1- γ ::::? α 2 = -α β-α γ . Έτσι έχουμε: 2α2 + βγ = α2 -αβ-αγ + βγ = =α( α -β) -γ( α-β)= (α -β)( α -γ) -:1- Ο.

Όμοια: 2β2 + αγ =(β-α)(β-γ) -:1- Ο

και2γ2 +αβ=(γ-α(γ-β)-:�-0. ii). Ονομάζομαι Α το πρώτο μέλος της (1). Έχουμε:

α2 β2 γ2 Α= +---'---(α-β)(α-γ) (α-β)(β-γ) (γ-α)(γ-β) α2(β -γ)-β2(α -γ)+ γ2(α-β)

(α-β)(α-γ)(β-γ) Ο αριθμητής του τελευταίου κλάσματος είναι ίσος με: α2(β-γ) _ β2α+ β2γ + γ2α -γ2β = α2(β-γ) -α(β -γ)(β+ γ) +βγ(β-γ) = =(β-γ)(α2 -αβ-αγ+βγ) = ... = (β-γ)(α-β)(α-γ). Άρα Α=1 .

Ερ. Χατζηφωτιάδης, Λύκειο Μελισσίων

Άσκηση Γl. Θεωρούμε δύο πραγματικούς αριθμούς α και β. Αρκεί να δείξουμε ότι: f(α)=f(β). Εξετάζουμε αν υπάρχουν πραγματικοί αριθμοί χ και y για τους οποίους ισχύουν: 3χ-2y=α και 2χ-y=β. Έχουμε:

{3χ-2y =α {3χ -2y =α

{χ = 2β-α

2χ-y=β <=> -4χ +2y=-2β <=> y=3β-2α

Άρα, θέτοντας στη δοσμένη ισότητα χ=2β-α

και y= 3β -2α βρίσκουμε ότι f(α)=f(β).

Συνεπώς η συνάρτηση f είναι σταθερή. Αν. Σχοινά, 1° Λύκειο Γέρακα

Page 3: Ευκλειδης Β 75

ΕΛΛΗΝΙΚΗ ΜΑΘΗΜΑΤΙΚΗ ETAIPEIA Τ ε ύ χ ο c; 75 • Ι α ν ο υ ά ρ ι ο c; • Φεβρουάριοc; • Μ ά ρ τ ι ο c; 2010· Έτ οc; λ θ' - Ευρώ: 3,50

e-mail: info@hms,gr www,hms,gr

ΜΑΘΗΜΑΤΙΚΟ nΕΡΙΟΔΙΚΟ ΓΙΑ ΤΟ ΛΥΚΕΙΟ

ΠΕΡΙΕΧΟΜΕΝΑ .f Αρχαίοι Ρόδιοι Μαθηματικοί και Αστρονόμοι

.f Τα περίφημα "άλυτα" προβλήματα της αρχαιότητας

.f Μαθηματικοί Διαγωνισμοί - Μαθηματικές Ολυμπιάδες

.f Homo Mathematicus

Μαθηματικά Α' Τάξης .f Άλγεβρα: Σύστημα δύο γραμμικών εξισώσεων

με δύο αγνώστους .f fεωμετρία: Εγγράψιμα Τετράπλευρα

Μαθηματικά Β' Τάξης .f Άλγεβρα: Εκθετική-Λογαριθμική συνάρτηση

.f rεωμετρία: Κανονικά Πολύγωνα - Μέτρηση Κύκλου .f Κατεύθυνση: Κωνικές Τομές- Θεωρία Αριθμών

Μαθηματικά Γ' Τάξης

4

7

14

18 26

29 35 42

.f Μαθηματικά fενικής Παιδείας: JYXAIO -ΠΙΘΑΝΟ -ΒΕΒΑΙΟ 52 .f Μαθηματικά Κατεύθυνσης: Παραγώγιση - Ολοκλήρωση 59

•••••••••••• • ••••••••• .f Το Βήμα του Ευκλείδη 72

.f Χάρης Καλλιγάς 7 4

.f Ο Ευκλείδης προτείνει... 76 .f Τα Μαθηματικά μας Διασκεδάζουν 80 .f Στήλη του Μαθητή 82

ΜαΘηματικός Διαγωνισμός ;\? /UJ\J-JΔJ-JZ: 27 Φεβρουαρίου 201 Ο

Γράμμα της Σύνταξης Αγαπητοί μαθητές και συνάδελφοι,

Σας ευχαριστούμε που περιβάλλετε με εκ­

τίμηση το περιοδικό μας, και βοηθάτε στην α­

ναβάθμισή του με την καλοπροαίρετη κριτική

σας και τις ενδιαφέρουσες εργασίες σας •

Η συνεργασία σας μας είναι πάντα απαραί­

τητη και μας ενδυναμώνει στη συνέχεια της

προσπάθειάς μας

Σε μικρό χρονικό διάστημα θα έχετε στα

χέρια σας και το 4ο τεύχος για μια ολοκληρ­

ωμένη επανάληψη

Με ευχές για υγεία και πρόοδο

Ο Πρόεδρος της Συντακτικής Επιτροπής

Γ.Σ. Τασσόπουλος

Ο Αντιπρόεδρος της Συντακτικής Επιτροπής

β. Ευσταθίου

Σύνθεση εξωφύλλου:

Από το έντυπο "L'edυcαtion Aυjoυrd'hυi"

της UNESCO του 2005 σε ελεύθερη σχεδίαση

••••••••••••••••••••••••••••••••••••••••••••••••••••••••••••••••••••••••••••••

ΕΚΔΟΣΗ ΤΗΣ ΜΑΘΗΜΑ1ΙΚΗΣ ΗΑΙΡΕΙΑΣ

ΠΑΝΕΠΙΣΤΗΜΙΟΥ 34 106 79 ΑΘΗΝΑ Trt-.: 210 3617784-3616532 Fox: 2103641025

Εκτελεστική Γραμματεία Πρόεδρος:

Τασσόπουλος Γιώργος

Εκδότης: Αντιπρόεδρος:

Καλογερόποuλος Γρηγ6ριος Ευσταθίου Βαγγέλης

Διευθυντής: Γραμματέας:

Τuρλής Ιωάννης Χριατ6ποuλος Πανaγιώτης Επιμέλεια Έκδοσης: Μέλη: Αργυράκης Δ. Τασσόποuλος Γιώργος Λουρίδας Σ. Ευσταθίου Βαγγέλης Ταπεινός Ν. Ζώτος Βαγγέλης

Κωδικός ΕΛ.ΤΑ.: 2055 ISSN: 1105- 7998

Αθανασόπουλο<; Γεώρyιοc;

Αναστασίου Γιάννηc;

Ανδρουλακάκηc; Νίκοc;

Αντωνόπουλο<; Νίκοc;

Αρyυράκηc; Δημήτριοc;

Βακαλόπουλοc; Κώσταc;

Ευσταθίου Βαyyέληc;

zαχαρόπουλοc; Κων/νοc;

Ζώτοc; ΒαyyεΛηc;

Κακκαβάc; Απόστολοc;

Καλίκαc; Σταμάτηc;

ΚανεΛλοc; Χρήστοc;

Καραyκούνηc; Δημήτρη<;

Καρκάνηc; Βασίληc;

Κατσούληc; Γιώρyοc;

Κερασαρίδηc; Γιάννηc;

Καρδαμίτσηc; Σπύροc;

Κηπουρό<; Χρήστοc;

Κλάδη Κατερίνα

Συντακτική επιτροπή

Κόντζιαc; Νίκοc; Κοτσιφάκηc; Γιώρyοc; ΚουτρουμπεΛαc; Κώσταc; Κυριαζήc; ιωάννηc; Κυριακόπουλο<; Αντώνηc; Κυριακοπούλου Κων/να Κυβερνήτου Χρυστ. Λαζαρίδη<; Χρήστοc; Λάππαc; Λευτέρη<; Λουρίδα<; Ιωτήρηc; Μαλαφέκαc; Θανάσηc; Μανωλάκου Στοματική Μαυροyιαννάκηc; Λεωνίδαc; Μενδρινόc; Γιάννηc; Μεταξάc; Νικόλαοc; Μπρίνοc; Παναyιώτηc; Μυλωνάc; Δημήτρη<; Μώκοc; Χρήστοc; Πανουσάκηc; Νίκοc; Ρέyκλης Δημήτρη<;

Ιdί'rη Εύα

Ιταϊκοc; Κώσταc;

Ιτάϊκοc; Παναyιώτηc;

Ιτρατήc; Γιάννηc;

Ταπεινός Νικόλαοc;

Τασσόπουλοc; Γιώρyοc;

Τζιώτζιοc; Θανάσηc;

Τριάντοc; Γεώρyιοc;

Τσαyκάρηc; Ανδρέαc;

Τσατούραc; Ευάyyελοc;

Τσικαλουδάκηc; Γιώρyοc;

Τσιούμαc; Θανάσηc;

Τυρλήc; Ιωάννηc;

ΦανεΛη Άννυ

Χαραλαμποπούλου Λίνα

Χαραλάμπουc; Θάνοc;

Χριστιάc; Ιπύροc;

Χριστόπουλο<; Παναyιώτηc;

Ψύχαc; ΒαyyεΛηc; ••••••••••••••••••••••••••••••••••••••••••••••••••••••••••••••••••••••••••••••

• Τα διαφημιζόμενα βιβλία δε σημαίνει ότι προτείνονται από την Ε.Μ.Ε. • Οι συνερyάτεc;, τα άρθρα, οι προτεινόμενες ασκήσεις, οι λύσεις ασκήσεων κτλ. πρέπει να σrέλνονται έγκαιρα, σrα yραφεία τηc; Ε.Μ.Ε.

με την lνδειξη "fια τον Ευκλείδη β'". Τα χειρόyραφα δεν εmσrρέφοvται. Τιμή Τεύχους ευρώ 3,50 Ετήσια συνδρομή (12,00 + 2,00 Ταχυδρομικά= ευρώ 14,00) Ετήσια συνδρομή για Σχολεία ευρώ 12,00 Το αντίτιμο yια τα τεύχη που παραyyέλνοvται σrε'λνεται με απλή εmταyή σε διαταyή Ε.Μ.Ε. Ταχ. fραφείο Αθήνα 54 Τ.Θ. 30044 ή πληρώνεται σrα yραφεία τηc; Ε.Μ.Ε.

Εκτunωση: ΙΝ7ΕΡΠΡΕΣ Α.Ε. τηλ.: 210 8/60330 Υπεύθυνος τυnοypαφείου: Β. Σωτηριάδης

Page 4: Ευκλειδης Β 75

Αρχαίοι Ρόδιοι Μαθηματικοί και Αστρονόμοι Καραγιάννης Γιάννης-Τσομαρέλη Τριανταφυλλιά,

Σ την εργασία αυτή θα προσπαθήσουμε να αναδείξουμε αρχαίους μαθηματικούς και αστρονόμους που γεννήθηκαν, έζησαν ή

εργάστηκαν στη Ρόδο ανάμεσα στον 4°v - 1 ον αι π.χ που ,δυστυχώς, οι περισσότεροι δεν είναι ευρύτερα γνωστοί.

Σχεδόν σε όλα τα μαθηματικά sites ανάμεσα στα ονόματα σπουδαίων Αρχαίων Μαθηματικών και Αστρονόμων αναφέρονται και τα ονόματα Ρο­δίων που ξεχώρισαν για το έργο τους. Ανάμεσα τους αυτά των Άτταλου του Ροδίου, Γέμινου του Ροδίου, Εύδημου του Ροδίου, Ίππαρχου του Ρο­δίου, Παναίτιου του Ροδίου και Ποσειδωνίου του Αππαμεύς ή του Ροδίου. Είναι ιστορικά απο­δεδειγμένο ότι η Αρχαία Ρόδος ήταν επιστημονικά και πολιτιστικά πολύ αναπτυγμένη και ότι κατά καιρούς είχαν περάσει από τη Ρόδο πολλοί αρχαίοι φιλόσοφοι και επιστήμονες για να δουν και να μά­θουν από τις σχολές που υπήρχαν σε αυτήν.

Ας δούμε όμως πιο αναλυτικά μερικά συνο­πτικά στοιχεία από τη βιογραφία και το έργο τους (

I) Άτταλος ο Ρόδιος (4°ς αι π.Χ): Ο Άτταλος ο Ρόδιος ήταν αρχαίος γραμματικός, μαθηματικός και αστρονόμος, καταγόμενος από την Ρόδο. τί­ποτε δεν έχει διασωθεί από τυχόν έργα του. Μνη­μονεύεται όμως το όνομά του σε υπόμνημα του Ιππάρχου περί των "Φαινομένων του Αράτου και του Ευδόξου". Στο σύγγραμμά του αυτό ο Ίππαρ­χος αναφέρει διάφορα παραδείγματα επί των πα-

ραπάνω φαινομένων, αποσπάματα του οποίου έ­χουν διασωθεί. 2) Γέμινος ή Γεμίνος ο Ρόδιος(l l Ο-40 π.Χ). : Ο Γεμίνος έζησε και εργάστηκε στη Ρόδο στη σχο­λή του Ποσειδωνίου, του οποίου υπήρξε μαθητής. Αναφέρεται ως Μαθηματικός, Αστρονόμος, Γε­ωγράφος και Μετεωρολόγος. Το μοναδικό σωζό­μενο έργο του Γεμίνου είναι η «Εiσαγωγή εiς τα Φαινόμενα», βασισμένο σε έργα παλαιότερων α­στρονόμων που είχε σκοπό να διδάξει την επιστή­μη αυτή σε αρχάριους όπου περιγράφει τη φαινό­μενη κίνηση του ηλίου, τις σεληνοηλιακές περιο­δικότητες, τις εκλείψεις, ότι η Γη είναι σφαιρική , η Σελήνη είναι ετερόφωτο σώμα, οι διαστάσεις της Γης είναι σημειακές (αμελητέες) σε σχέση με το μέγεθος του Σύμπαντος κ.α. Ο Γεμίνος συνέ­γραψε και ένα σχόλιο επί του έργου του Ποσειδω­νίου «Μετεωρολογικά». Σπαράγματα αυτού του σχολίου διασώθηκαν από τον Σιμπλίκιο στο σχό­λιό του επί των «Φυσικών» του Αριστοτέλη .

Το κυριότερο μαθηματικό έργο του Γεμίνου είναι το «Μαθηματικών Δόγμα>> (δηλ. θεωρία των Μαθηματικών). Παρότι αυτό το έργο δεν έχει σω­θεί, αρκετά αποσπάσματά του υπάρχουν σε έργα των Πρόκλου, Ευτοκίου και άλλων. Ο Γεμίνος δι­αχώρισε τα Μαθηματικά σε δύο μέρη : τα «νοητά» και τα «αισθητά», ή, όπως θα λέγαμε σήμερα, σε Καθαρά και Εφαρμοσμένα Μαθηματικά. Μακρά αποσπάσματα του έργου του διασώθηκαν και στο Σχόλιο των «Στοιχείων» του Ευκλείδη από τον Al­Nayriz. Επίσης ο Γεμίνος έγραψε ιστορία των μα­θηματικών συνεχίζοντας το έργο του Ευδήμου του Ροδίου. Πολλοί ερευνητές, ανάμεσα τους και ο Άγγλος φυσικός Derek de Solla Price, ισχυρίστη­καν ότι ο Γεμίνος ήταν ο κατασκευαστής του Μη­χανισμού των Αντικηθύρων, ίσως της σπουδαιό­τερης τεχνολογικής κατασκευής του αρχαίου κό­σμου, του πρώτου υπολογιστή που τον χρησιμο­ποιούσαν για να υπολογίζουν τις κινήσεις των πλανητών και των αστέρων. Ονομάστηκε προς τι­μή του ο κρατήρας Γέμινος (Geminus) στο Βόρειο ημισφαίριο της Σελήνης και υπάρχει αφιέρωμα του στο Τεχνολογικό Μουσείο Θεσσαλονίκης. 3) Εύδημος ο Ρόδιος (350-290 π.Χ) : Ο Εύδημος ο Ρόδιος, ο πρώτος ιστορικός των μαθηματικών,

ΕΥΚΛΕΙΔΗΣ Β' 75 τ.3/1

Page 5: Ευκλειδης Β 75

----------- Αρχαίοι Ρόδιοι Μαθηματικοί και Αστρονόμοι -----------

υπήρξε ένας διακεκριμένος μαθητής του Αριστο­τέλη , ο οποίος τον περιέβαλλε με μεγάλη εκτίμη­ση . Λέγεται ότι ο μεγάλος φιλόσοφος βρέθηκε για αρκετό καιρό σε δίλημμα, σχετικά με το πρόβλημα της διαδοχής του στο Λύκειο είχε δυσκολία να ε­πιλέξει μεταξύ του Θεοφράστου και του Ευδήμου, λόγω των πολλαπλών προσόντων τους. Τελικά ε­πέλεξε τον Θεόφραστο, χωρίς φυσικά αυτή του η επιλογή να υποτιμά το έργο του Ευδήμου(«ο μεν Ρόδιος οίνος και αδρός, αλλ ' ο Λέσβιος γλυκύτε­ρος»). Ο Εύδημος υπήρξε πολυγραφότατος. Έ­γραψε έργα στην ιστορία των επιστημών, καθώς και στις θετικές και θεωρητικές επιστήμες. Γνωστά έργα του είναι τρία : Η ιστορία της αριθμητικής αναφερόμενη από τον Πορφύριο, η ιστορία της Γεωμετρίας αναφερόμενη από τον Πρόκλο, και η Αστρονομική ιστορία που μνημονεύεται από τον Διογένη το Λαέρτιο . Πολλοί σύγχρονοι ιστορικοί των επιστημών αποδίδουν στον Εύδημο διασκευές διαφόρων aριστοτελικών έργων. Είναι πιθανόν το έργο "Ευδήμια ηθικά" που αποδίδεται στον Αρι­στοτέλη , να είναι έργο του Ευδήμου και να αποτε­λεί περίληψη της διδασκαλίας περί ηθικής του Σταγειρίτη φιλοσόφου . Από το μεγάλο συγγραφικό έργο του Ευδήμου δεν έχει σωθεί σχεδόν τίποτε. Ευτυχώς ορισμένοι αρχαίοι συγγραφείς , Πρόκλος, Πάππος, Θέων ο Σμυρναίος, Σιμπλίκιος, Αιλιανός, Βοήθιος και Πορφύριος, διέσωσαν ορισμένα απο­σπάσματα. Υπάρχει αφιέρωμα του στο Τεχνολογι­κό Μουσείο Θεσσαλονίκης. 4) Ίππαρχος ο Ρόδιος (190 - 120 π.Χ.) : Ο Ίππαρ­χος ο Ρόδιος ή Ίππαρχος ο Νικαεύς ήταν Έλληνας aστρονόμος, γεωγράφος, χαρτογράφος και μαθη­ματικός, θεωρούμενος από αρκετούς ως ο «πατέ­ρας της Αστρονομίας» .

Ίππαρχος ο Ρόδιος(1 90 - 120 π.Χ)

Η υπομονή του, η οξυδέρκειά του αλλά και το βεβαιούμενο ιστορικά πάθος του με ότι καταπια­νόταν τον οδήγησαν σε δρόμους που σήμερα, ανα­λογικά με τα δεδομένα της εποχής του, σίγουρα

εντυπωσιάζουν. Ο Ίππαρχος γεννήθηκε στη Νίκαια της Βιθυνίας στη Μικρά Ασία γύρω στο 1 90 π.Χ� Το μεγαλύτερο διάστημα της ζωής του έζησε και εργάστηκε στη Ρόδο και την Αλεξάν­δρεια Ανέπτυξε μαθηματικά μοντέλα για την κί­νηση του Ηλίου και της Σελήνης, από παρατηρή­σεις αιώνων αρχίζοντας από τους Χαλδαίους της Μεσοποταμίας. Υπήρξε επίσης ο πρώτος που συ­νέταξε τριγωνομετρικό πίνακα, πράγμα που του επέτρεπε να επιλύει οποιοδήποτε τυχαίο τρίγωνο. Τα έξι κορυφαία πάντως επιτεύγματά του ήταν: (I Από τον 2ο αιώνα π.Χ. υπολόγισε πως το

ηλιακό ή τροπικό έτος είναι 365,242 ημέρες, όταν σήμερα τα σύγχρονα ατομικά ρολόγια τον επιβεβαιώνουν υπολογίζοντάς το σε 365,242199 ημέρες !

.. Η ανακάλυψη της μεταπτώσεως των ισημε­ριών.

• Υπολόγισε τη διάμετρο της Σελήνης και τη κυμαινόμενη απόστασή της από τη Γη .

" Η δημιουργία του πρώτου καταλόγου αστέρων, τουλάχιστον στο δυτικό κόσμο.

• Η επινόηση της κλίμακας των μεγεθών των αστέρων από τη μέτρηση της φωτεινότητάς των, που χρησιμοποιείται ακόμα και σήμερα από όλους τους aστρονόμους του κόσμου.

Το 1 34 π.Χ. ο Ίππαρχος ανακάλυψε ένα αστέ­ρα που δεν υπήρχε πριν (πιθανόν κομήτη) στον αστερισμό του Σκορπιού, τότε διατύπωσε την αρχή της αστρονομίας ότι «οι αστέρες δεν είναι αιώνιοι

στον ουρανό». Ο Ίππαρχος ο Ρόδιος έγραψε 17 γνωστούς τίτλους βιβλίων εκ των οποίων μόνο το «Περί των Αράτου και Ευδόξου φαινομένων - βι­βλία τρία» διεσώθη . Υπάρχει αφιέρωμα του στο Τεχνολογικό Μουσείο Θεσσαλονίκης.

Ίππαρχος Λιθογραφία που αναπαριστά τον Ίππαρχο

(από βιβλίο του 1 88 1 ) .

ΕΥΚΛΕΙΔΗΣ Β' 75 τ.3/2

Page 6: Ευκλειδης Β 75

Αρχαίοι Ρόδιοι Μαθηματικοί και Αστρονόμοι -----------

Παναίτιος ο Ρόδιος (185-109 π.Χ): Αρχαίος Έλληνας Μαθηματικός και Φιλόσοφος των στω­ικών δογμάτων και μελετητής της πλατωνικής και aριστοτελικής φιλοσοφίας. Ασχολήθηκε περισσό­τερο με την ερμηνεία των μαθηματικών εννοιών σε φιλοσοφική διάσταση . Γεννήθηκε στη Ρόδο και πέθανε στην Αθήνα. Υπήρξε μαθητής του Διο­γένη του Βαβυλώνιου και του Α ντίπατρου του Ταρσέα. αργότερα, όταν εγκαταστάθηκε στη Ρώ­μη, μαθήτευσε κοντά στο Σκιπίωνα τον Αιμιλιανό και το Λέλιο . Θεωρείται ο πραγματικός ιδρυτής της "Μέσης Στοάς" και ως φιλόσοφος προσπάθησε να συγχωνεύσει τη φιλοσοφία των Ρωμαίων με τη στωική ηθική . Υποστήριξε την κυριαρχία της Ρώ­μης ως έργο της θείας πρόνοιας, γεγονός που τον κατέταξε μεταξύ των ιδεολόγων του κύκλου του Σκιπίωνα και συντέλεσε ώστε να γίνει ο στωικι­σμός η τυπική φιλοσοφία των δημοκρατικών και aριστοκρατικών φιλοσοφικών κέντρων της Ρώμης. Δάσκαλος του Ποσειδωνίου του Απαμεύς.

Ποσειδώνιος ο Ρόδιος (135 .- 51 π.Χ.) : Ο Πο­σειδώνιος ο Ρόδιος ή ο Απαμεύς ήταν Έλληνας πολυμαθής Στωικός φιλόσοφος, aστρονόμος, μαθηματικός γεωγράφος, πολιτικός, ιστορικός και δάσκαλος που γεννήθηκε στην Απάμεια της Συρίας. Τον θεωρούσαν τον πολυμαθέστερο άν­θρωπο του κόσμου για την εποχή του . τίποτα από το τεράστιο έργο του δεν έχει σωθεί ως ολότητα σήμερα, αλλά μόνο αποσπάσματα. Το μεγαλύτερο μέρος της ζωής του το έζησε στη Ρόδο και πέθανε είτε στη Ρόδο είτε στη Ρώμη . Και όχι μόνο έγινε πολίτης, αλλά συμμετείχε ενεργά στην πολιτική ζωή της Ρόδου, φθάνοντας στα ανώτερα αξιώματα ως ένας από τους πρυτάνεις της και υπηρέτησε εξάλλου ως πρεσβευτής της Ρόδου στη Ρώμη το 87 - 86 π.Χ. Τα πολλά έργα και οι διαλέξεις του Πο-

Ο Ποσειδώνιος έκανε επίσης ένα υπολογισμό της διαμέτρου και της αποστάσεως της Σελήνης. Γνω­ρίζουμε επίσης ότι είχε κατασκευαστεί στη σχολή του ένας «φορητός υπολογιστής» των κινήσεων των ουράνιων σωμάτων, παρόμοιο με τον Μηχανισμό των Α ντικυθήρων όπως αυτός αποκα­λύφθηκε μετά την πρόσφατη ανάλυση (2006-2008), και ίσως τον ίδιο τον μηχανισμό αυτό, που χρονολογείται στην ίδια περίπου περίοδο. Σύμφω­να με τον Κικέρωνα (De Natura Deorum, Π 34), το φορητό «πλανητάριο» της σχολής του Ποσειδωνί­ου έδειχνε τις ημερήσιες κινήσεις του Ηλίου, της Σελήνης και των 5 γνωστών τότε πλανητών. Εκτός από τα όσα έγραψε για τη Γεωμετρία, ο Ποσειδώ­νιος πιστώνεται με τη διατύπωση-δημιουργία μα­θηματικών ορισμών, ή τουλάχιστον για την ξεκά­θαρη διατύπωση απόψεων για όρους όπως π.χ. «θεώρημα» και «πρόβλημα» . Ο κρατήρας Ποσειδώνιος στο βόρειο ημισφαίριο της Σελήνης πήρε το όνομά του από τον Ποσειδώνιο τον Ρόδιο Υπάρχει αφιέρωμα του στο Τεχνολογικό Μουσείο Θεσσαλονίκης.

Προτομή του Ποσειδωνίου (135 .- 51 π.Χ.) Μουσείο Νάπολης.

Βι!:iλωγι�υφ�κi:ς πηγf:: σειδωνίου του χάρισαν μια αυθεντία και φήμη πα- 1 . Θέματα από την ιστορία των Μαθηματικών,

Γιάννη Χριστιανάκη , Πανεπιστημιακές Εκ­δόσεις Κρήτης.

ντού στον ελληνορωμαϊκό κόσμο, ενώ στη Ρόδο αναπτύχθηκε γύρω του μία σχολή . Παρότι λίγα πράγματα είναι γνωστά για την οργάνωση της 2 . σχολής του, είναι βέβαιο ότι ο Ποσειδώνιος είχε ένα σταθερό αριθμό Ελλήνων και Ρωμαίων φοιτη- 3 . τών. Αποσπάσματα από το αστρονομικό έργο του Ποσειδωνίου σώζονται μέσα από την πραγματεία 4 . του Κλεομήδη «Κυκλική θεωρία μετεώρων», όπου το πρώτο κεφάλαιο του δεύτερου βιβλίου φαίνεται ότι έχει στο μεγαλύτερο μέρος του αντιγραφεί από 5 . τον Ποσειδώνιο . Για τη διάμετρο του Ηλίου βρήκε 6 . μία τιμή μεγαλύτερη και ακριβέστερη από αυτές που πρότειναν άλλοι αρχαίοι Έλληνες αστρονόμοι.

Εισαγωγή εις τα φαινόμενα του Γεμίνου του Ροδίου, Ε. Σπανδάγου, Εκδόσεις Αίθρα. Ο Εύδημος ο Ρόδιος. Ε. Σπανδάγου, Εκδό­σεις Αίθρα. Ιστοσελίδα του Τεχνολογικού Μουσείου Θεσσαλονίκης( διαθέσιμο στο

www.tmth.edu.gr) . Ηλεκτρονική Εγκυκλοπαίδεια βικιπαίδεια. Εγκυκλοπαίδεια Παύλου Δρανδάκη .

ΕΥΚΛΕΙΔΗΣ Β' 75 τ.3/3

Page 7: Ευκλειδης Β 75

Τα περίφημα "άλυτα" προβλήματα ,

της αρχαιοτητας Σωτήρης Χ. Γκουντουβάς

Τα τρία περίφημα "άλυτα" γεωμετρικά προβλήματα της αρχαιότητας εμφανίστηκαν περίπου στα μέσα του 5°υ π.Χ αιώνα στην αρχαία Ελλάδα. Είναι προβλήματα γεωμετρικών κατασκευών που πρέπει να πραγματοποιηθούν με αποκλειστική χρήση κανόνα και διαβήτη. Αυτά είναι ο διπλασιασμός του κύβου ή Δήλιο πρόβλημα, ο τετραγωνισμός του κύκλου, και η τριχοτό­μηση γωνίας.

Το πρόβλημα του διπλασιασμού του κύβου είναι το εξής: Να κατασκευαστεί κύβος με όγκο διπλάσιο από τον όγκο δοθέντος κύβου ακμής α. Ζητείται λοιπόν, να κατασκευαστεί η ακμή χ ενός κύβου που θα έχει διπλάσιο όγκο από κύ-

βο δοθείσης ακμής α. Δηλαδή να κατασκευαστεί το τμήμα x=αifi. Το πρόβλημα είναι αδύνατο να επιλυθεί με κανόνα και διαβήτη όπως αποδείχθηκε το 1 83 7 με τη βοήθεια της θεωρίας Galois. Παρόλα αυτά οι αρχαίοι Έλληνες Γεωμέτρες έδωσαν πολλές λύσεις στο πρόβλημα με άλλα μέσα κατασκευής.

Ο Ιπποκράτης ο Χίος (�470-400 π.Χ) πρώτος ανήγαγε το πρόβλημα στην παρεμβολή δύο μέσων αναλόγων χ, y μεταξύ των τμημάτων α και 2α. Το πρόβλημα τότε ικανοποιεί τις συνεχείς

αναλογίες α = � = _)'_ , που οδηγούν στη ζητούμενη σχέση χ=α if2 . χ y 2α

Ο Ερατοσθένης ο Κυρηναίος (276- 1 94 π.Χ) σε επιστολή που έστειλε στον βασιλιά του Ελ­ληνιστικού κράτους της Αιγύπτου Πτολεμαίο Γ' τον Ευεργέτη (284-222 π.Χ) αναφέρει την ι­στορία του προβλήματος, διάφορες λύσεις που έχουν δοθεί και τη δική του λύση της κατα­σκευής δύο μέσων αναλόγων με τη βοήθεια του μεσολάβου, ενός οργάνου που επενόησε γι' αυ­τό το σκοπό.

Ο Ευτόκιος (6°ς μ.Χ αι.) στα σχόλια του για την πραγματεία του Αρχιμήδη "Περί σφαίρας και κυλίνδρου" αναφέρει 1 2 λύσεις του προβλήματος. Λύσεις στο πρόβλημα εκτός του Ιπποκρά­τη έδωσαν οι :

• Αρχύτας ο Ταραντίνος ( 428-365 π.Χ) με ημικυλίνδρους • Πλάτων (427-347 π.Χ) με παρεμβολή δύο μέσων αναλόγων • Εύδοξος ο Κνίδιος (407-354 π.Χ) με την "καμπύλη του Ευδόξου" • Μέναιχμος (� 350 π.Χ) με δύο παραβολές • Αρχιμήδης (287-21 2 π.Χ) με δύο κωνικές τομές, παραβολή και υπερβολή • Ερατοσθένης με το μεσολάβο • Απολλώνιος (�270- 1 90 π.Χ) με κύκλο και ισοσκελή υπερβολή • Νικομήδης (�200 π.Χ) με την κογχοειδή καμπύλη • Ήρων (�200 π.Χ) με νεύση Κ R • Διοκλής (�1 °ς π.Χ αι.) με την κισσοειδή καμπύλη • Πάππος (�3 °ς μ.Χ αι.) με νεύση.

Θα δώσουμε τώρα τη λύση που αποδίδεται στον Πλάτωνα. Έστω δύο κάθετοι άξονες που τέμνονται στο Ο. Στον ένα άξονα παίρνουμε τμήμα ΟΑ=α και Λ στον άλλο τμήμα ΟΒ=2α. Φέρουμε τώρα τις ΑΚ//ΒΛ έτσι ώστε ΑΚJ..ΚΛ και ΒΛ_lΚΛ.Το ζητούμενο τμήμα χ είναι το ΟΚ.

, 2 ΟΑ ΟΚ Στο ορθογωνιο ΑΚΛ: ΟΚ = ΟΑ·ΟΛ<::::>-=-

ΟΚ ΟΛ

ΕΥΚΛΕΙΔΗΣ Β' 75 τ.3/4

Page 8: Ευκλειδης Β 75

--------- Τα περίφημα «άλυτα» προβλήματα της αρχαιότητας ---------

, 2 ΟΚ ΟΛ Στο ορθογωνιο ΒΚΛ: ΟΛ = ΟΚ.ΟΒ<=>-= -.

ΟΛ ΟΒ

Ά ΟΑ ΟΚ ΟΛ α χ Υ Ο , ζ , , _ ΟΚ- 3

r;::;2 ρα -- = -- = -<=>- = - = - . ποτε το ητουμενο τμημα χ - - α ν L . ΟΚ ΟΑ ΟΒ χ y 2α

Η τεθλασμένη γραμμή ΑΚΛΒ με ορθές γωνίες στα Κ και Λ μπορεί να επιτευχθεί με ειδικό όρ­γανο, όπου σε ένα μεταβλητό ορθογώνιο πλαίσιο ολισθαίνει μια πλευρά κάθετα σε δύο άλλες που είναι σταθερές.

Το πρόβλημα του τετραγωνισμού του κύκλου είναι το εξής: Να κατασκευαστεί τετράγωνο με εμβαδόν ίσο με δοθέντα κύκλο ακτίνας α. Ζητείται λοιπόν, να κατασκευαστεί η πλευρά χ ενός τετραγώνου που θα έχει ίσο εμβαδόν με κύ-κλο ακτίνας α. Δηλαδή να κατασκευαστεί το τμήμα χ=α ,J;.. Ο πρώτος που αναφέρεται ότι α­σχολήθηκε με το πρόβλημα είναι ο Αναξαγόρας ο Κλαζομένιος (500-428 π.Χ) . Το αδύνατο της κατασκευής με κανόνα και διαβήτη αποδείχθηκε το 1 882 από τον C.F.Von Lindemann.

Ο Ιάμβλιχος (3°ς μ.Χ αι.) αναφέρει ότι το πρόβλημα έλυσαν οι: • Αρχιμήδης με την έλικα • Νικομήδης με την τετραγωνίζουσα του Ιππία • Απολλώνιος με την κοχλιοειδή • Κάρπος με την καμπύλη "εκ διπλής κινήσεως προερχόμενη". Επίσης ο Πάππος στη Συναγωγή του δίνει μια λύση με την

τετραγωνίζουσα του Ιππία και την αποδίδει στο Δεινόστρατο (� 350 π.Χ) αδελφό του Μέναιχμου. Στη συνέχεια θα δούμε τη λύση αυτή.

Δr-� __ r_ι __ �r

Η τετραγωνίζουσα είναι καμπύλη που παράγεται από δύο ομαλές κινήσεις, μια ευθύγραμμη και μια κυκλική κίνηση. Στο α παρακάτω τετράγωνο ΟΑΓ Δ θεωρούμε την πλευρά ΔΓ να κινείται ομαλά προς τα κάτω παράλληλα προς τον εαυτό της και την ΟΔ να στρέφεται ομαλά γύρω από το Ο, μέχρι να συμπέσει με την ΟΑ σε ίσο χρόνο με την ΔΓ. Η τροχιά του σημείου τομής των ΔΓ και ΟΔ, κάθε στιγμή της κίνησής τους παράγει την τετραγωνίζουσα. Στο σχήμα το τόξο ΔΕ είναι η τετραγωνίζουσα.

ο Ε

Στο σχήμα λοιπόν έχουμε την τετραγωνίζουσα ΔΕ και ένα τεταρτοκύκλιο ΑΔ. Έστω

q το μήκος του τόξου ΑΔ. Ο Πάππος αποδεικνύει τη σχέση : _g_ = � . α ΟΕ

Α

Από τη σχέση αυτή προκύπτει ότι το μήκος q του τεταρτοκυκλίου ΑΔ προκύπτει σαν τέταρ­τη ανάλογος των τμημάτων α, α και ΟΕ. Μπορούμε τώρα να κατασκευάσουμε ένα ευθύγραμμο τμήμα με μήκος 4 q, δηλαδή ίσο με το μήκος του κύκλου.

Χρησιμοποιούμε τώρα την πρόταση που απέδειξε ο Αρχιμήδης στο έργο του ''Κύκλου μέ­τρησις" ότι το εμβαδόν ενός κύκλου είναι ίσο με το εμβαδόν ορθογωνίου τριγώνου με δύο κάθετες πλευρές ίσες με το μήκος του κύκλου και την ακτίνα αντίστοιχα.

Για να τετραγωνίσουμε τον κύκλο κάνουμε τα παρακάτω βήματα: • Κατασκευάζουμε ένα ορθογώνιο τρίγωνο με κάθετες πλευρές ΑΓ=r και AB=4 q. • Παίρνουμε το μέσο Μ της ΑΒ και σχηματίζουμε το ορθογώνιο ΑΜΔΓ του οποίου το εμ-

βαδόν είναι ίσο με του τριγώνου ΑΒΓ. • Προεκτείνουμε την Γ Δ και παίρνουμε τμήμα ΔΕ=ΔΜ • Κατασκευάζουμε το ημικύκλιο με διάμετρο την ΓΕ • Στο Δ φέρουμε κάθετη στην ΓΕ που τέμνει το ημικύκλιο στο Ζ. Το τμήμα ΔΖ είναι η ζη­

τούμενη πλευρά χ του τετραγώνου. Άρα το εμβαδόν του τετραγώνου ΔΖΗΘ ισούται με το εμβαδόν του δοθέντος κύκλου.

ΕΥΚΛΕΙΔΗΣ Β' 75 τ.3/5

Page 9: Ευκλειδης Β 75

--------- Τα περίφημα «άλυτα» προβλήματα της αρχαιότητας ---------2 ΑΒ 2πr 2 Πράγματι: χ = ΓΔ·ΔΕ= ΓΔ·ΔΜ =-ΑΓ =-r=πr.

Η τετραγωνίζουσα από μερικούς συγγραφείς αναφέρεται ως τετραγω­νίζουσα του Δεινόστρατου. Όμως ο Ιππίας ο Ηλείος είναι αρχαιότερος του Δεινόστρατου και με τη βοήθεια της τετραγωνίζουσας είχε τριχοτο­μήσει την γωνία. Για το λόγο αυτό οι περισσότεροι συγγραφείς την αποδί­δουν στον Ιππία. Επίσης με τη βοή­θεια της τετραγωνίζουσας μπορούμε να διαιρέσουμε μια γωνία σε όσα ίσα μέρη θέλουμε. Η εξίσωση της τετραγωνίζουσας σε

καρτεσιανή μορφή είναι :

2 2

Α 2πr= 4q Μ

Έμβαδόν κύκλου = Έμβαδον ορθογωνίου τριγώνου ΑΒΓ

χ

Β

Στην τριχοτόμηση γωνίας ζητείται να κατασκευαστεί γωνία ίση με το 1 /3 δοθείσης τυχαίας γωνίας. Το αδύνατο της κατασκευής (με κανόνα και διαβήτη) αποδείχθηκε το 1 837 . Πρέπει εδώ να σημειώσουμε ότι για ορισμένες γωνίες η τριχοτόμηση με κανόνα και διαβήτη είναι εφικτή. Για παράδειγμα η ορθή γωνία τριχοτομείται. Για να τριχοτομηθεί η ορθή γωνία πρέπει να κατα­σκευαστεί μια γωνία 30°. Αυτό μπορεί να γίνει με την εγγραφή σε κύκλο ενός κανονικού δωδε­καγώνου. Τότε η κεντρική του γωνία είναι 30°. Γενικά μπορεί να αποδειχθεί ότι οι μόνες ακέ­ραιες γωνίες που κατασκευάζονται με κανόνα και διαβήτη είναι πολλαπλάσια του 3 . Λύσεις του προβλήματος με άλλα μέσα κατασκευής έδωσαν οι:

Ψ Ιππίας ο Ηλείος (5°ς π.Χ αι.) με την τετραγωνίζουσα •:• Αρχιμήδης δύο λύσεις με νεύση και την έλικα του •:• Νικομήδης με την κογχοειδή, •:• Πάππος με νεύση. Επίσης λύσεις έδωσαν μετά την αρχαιότητα και οι Pascal ( 1 650) , Ceva (1699), Mac Laurin

( 1 742) , Delenges ( 1 783) , Plateau ( 1 826) , Longchamps (1888) και Pergassi ( 1 893) . Θα δώσουμε τώρα τη λύση με νεύση που αποδίδεται στον Αρχιμήδη.

Έστω η προς τριχοτόμηση γωνία ΑΟΒ. Γράφουμε κύκλο με κέντρο Ο και τυχαία ακτίνα. Προεκτείνουμε τη ΒΟ και φέρνουμε την ευθεία ΑΔ έτσι ώστε ΟΑ=Γ Δ. Τότε η γωνία ΟΔΓ θα είναι το 1/3 της ΑΟΒ. Η απόδειξη στηρίζεται στα δύο ισοσκελή τρίγωνα ΑΟΓ και ΟΓΔ

και στις εξωτερικές γωνίες τους.

Α

I Β\ ο Δ

I /

Η κατασκευή (νεύση) της ευθείας ΑΓΔ με την ιδιότητα ΟΑ=ΓΔ μπορεί να γίνει με ένα ειδι­κό όργανο που επινοήθηκε από τον Αρχιμήδη.

Κλείνοντας αυτό το άρθρο μπορούμε να θυμηθούμε τη ρήση του David Hilbert από την πε­ρίφημη ομιλία του στο 2° Διεθνές συνέδριο των μαθηματικών στο Παρίσι το 1 900.

«Αλλά πόσο σφρίγος, ζωτικής σημασίας για τη μαθηματική επιστήμη, θα χάνονταν με το ξερί­ζωμα της γεωμετρίας και της μαθηματικής φυσικής.»

ΕΥΚΛΕΙΔΗΣ Β' 75 τ.3/6

Page 10: Ευκλειδης Β 75

Π ρόβλη μα I

,αθηματtκοi Δtαyωvtσμοi

αθηματtκές Ολυμπιάδες Επιτροπή Διαγωνισμών της Ε.Μ.Ε.

70ος ΠΑΝΕΛΛΗΝΙΟΣ ΜΑΘΗΤΙΚΟΣ ΔΙΑΓΩΝΙΣΜΟΣ ΣΤΑ ΜΑΘΗΜΑΤΙΚΆ 'Ό ΕΥΚΛΕΙΔΗΣ"

ΣΑΒΒΑΤΟ, 23 ΙΑΝΟΥΑΡΙΟΥ 2010

ΕΝΔΕΙΚΤΙΚΕΣ ΛΥΣΕΙΣ Α' Λυκείου

(i) Να βρείτε τις τιμές του ρητού αριθμού α, για τις οποίες ο αριθμός Α= αJ3 είναι ρητός. (ii) Να αποδείξετε ότι ο αριθμός Β= ( 1 + J3Y είναι άρρητος. Λίιση

(i) Για α = Ο είναι Α = Ο, ρητός. 'Εστω α ::F Ο . Αν ήταν ο Α = αJ3 ρητός, τότε ο αριθμός Α = J3, θα α

ήταν επίσης ρητός, ως πηλίκο δύο ρητών αριθμών, που είναι άτοπο. Επομένως, ο αριθμός Α είναι ρητός μόνο για α = Ο .

(ii) 'Ε χουμε Β = ( Ι + J3)2 = 4 + 2J3 . Αν ο αριθμός Β ήταν ρητός, τότε ο αριθμός Β - 4 = 2J3 θα ήταν

επίσης ρητός, ως διαφορά δύο ρητών, το οποίο είναι άτοπο, σύμφωνα με το (ί) .

Π ρόβλη μα 2 Να αποδείξετε ότι η εξίσωση χ+ 1 - 2lxl =αχ, έχει, για κάθε τιμή της παραμέτρου α Ε JR, μία του­λάχιστον πραγματική λύση. Για ποιες τιμές του α η εξίσωση έχει δύο διαφορετικές μεταξύ τους πραγματικές λύσεις; Λίιση Επειδή στην εξίσωση εμφανίζεται η απόλυτη τιμή του αγνώστου χ διακρίνουμε δύο περιπτώσεις: (i) Έστω χ �Ο . Τότε ισχύει lxl = χ και η δεδομένη εξίσωση είναι ισοδύναμη με το σύστημα: {χ = -Ι- , αν α> - 1

χ + Ι- 2χ = αχ, χ � Ο <::::> (α + Ι) χ = Ι, χ � Ο <::::> α + Ι αδύνατο, αν α � - 1 .

(ii) Έστω χ < Ο . Τότε ισχύει lxl = -χ και η δεδομένη εξίσωση είναι ισοδύναμη με το σύστημα: {χ = -Ι- , αν α< 3 χ + 1 + 2χ = αχ, χ < Ο <::::> ( α- 3 ) χ = Ι , χ < Ο <::> α-3

αδύνατο, αν α� 3 . Επομένως, για κάθε τιμή της παραμέτρου α Ε JR, η εξίσωση έχει μία τουλάχιστον πραγματική λύση . Η εξίσωση έχει 2 πραγματικές λύσεις διαφορετικές μεταξύ τους, αν ισχύει: - 1 < α < 3 .

Πράγματι, για -1 < α < 3 η εξίσωση έχει τις λύσεις χ 1 = -1 - < Ο και χ 2 = -1 - > Ο που είναι διαφορε-α- 3 α+ l

τικές μεταξύ τους.

ΕΥΚΛΕΙΔΗΣ Β' 75 τ.3/7

Page 11: Ευκλειδης Β 75

---------- Μαθηματικοί Διαγωνισμοί- Μαθηματικές Ολυμπιάδες ---------11 ρ611λημυ J Δίνεται τρίγωνο ABC εγγεγραμμένο σε κύκλο C(O, R) και έστω A1 , B1 ,C1 τα aντιδιαμετρικά

σημεία των κορυφών του A,B,C . Στις ευθείες που ορίζουν οι πλευρές BC,AC,AB θεωρούμε τα σημεία A2 , B2 ,C2 αντίστοιχα και έστω (ε1 ) η ευθεία που ορίζουν τα σημεία Α1 , Α2 , (ε2 ) η ευθεία που ορίζουν τα σημεία Β 1 , Β2 και (ε3 ) η ευθεία που ορίζουν τα σημεία C1 ,C2 •

Έστω ακόμη (δ1 ) η παράλληλη ευθεία που φέρουμε από το σημείο Α προς την (ε1 ) , (δ2 ) η πα­ράλληλη ευθεία που φέρουμε από το σημείο Β προς την (ε2 ) και (δ3 ) η παράλληλη ευθεία που φέ­ρουμε από το σημείο C προς την ( ε3 ) • Ν α αποδείξετε ότι οι ευθείες ( ε1 ) , ( ε2 ) και ( ε3 ) συντρέχουν (περνάνε από το ίδιο σημείο), αν, και μόνο αν, οι ευθείες (δ 1 ) , (δ2 ) και (δ3 ) συντρέχουν

ΛίJση Οι ευθείες (ε ι ) και (δ ι ) είναι συμμετρικές

ως προς το κέντρο Ο του περιγεγραμμένου κύ­κλου του τριγώνου ABC , αφού το Ο είναι μέ­σο της ΑΑι .

Οι ευθείες ( ε2 ) και (δ2 ) είναι συμμετρικές ως προς το κέντρο Ο του περιγεγραμμένου κύ­κλου του τριγώνου ABC , αφού το Ο είναι μέ­σο της ΒΒι .

Οι ευθείες ( ε3 ) και (δ3 ) είναι συμμετρικές ως προς το κέντρο Ο του περιγεγραμμένου κύ­κλου του τριγώνου ABC , αφού το Ο είναι μέ­σο της ccι .

Σύμφωνα με τη θεωρία, αν περιστρέψουμε μία ευθεία κατά 1 80° γύρω από το κέντρο συμ­μετρίας, τότε αυτή θα συμπέσει με τη συμμετρι­κή της ευθεία , ως προς κέντρο το σημείο Ο . Επομένως, οι ευθείες (ε ι ) , (ε2 ) και (ε3 ) συ-

Α

Β\

Σχήμα 3

ντρέχουν, έστω στο σημείο Κ , αν, και μόνο αν, οι ευθείες (δ ι ), (δ2 ) και (δ3 ) συντρέχουν στο σημείο Κ' , που είναι το συμμετρικό του σημείου Κ ως προς το σημείο Ο .

nuραη1ρηση Το σημείο Κ ταυτίζεται με το ορθόκεντρο του τριγώνου ABC , αν, και μόνο αν, τα σημεία

A2 , B2 , C2 είναι τα μέσα των πλευρών BC,AC,AB αντίστοιχα. Στη περίπτωση αυτή μπορούμε να χρησιμοποιήσουμε τη γνωστή πρόταση : "Τα συμμετρικά του ορθοκέντρου ως προς τα μέσα των πλευρών τριγώνου, βρίσκονται επάνω στο περι­

γεγραμμένο του κύκλο και είναι aντιδιαμετρικά των κορυφών του"

Π ρ6/1λημΗ 4 Οι πραγματικοί αριθμοί χ, y και z ικανοποιούν τις ισότητες:

χ3 - y3 = 26z3 x2y - xy2 = 6z3 •

(u) Να εκφράσετε τους x, y συναρτήσει του z . (β) Αν επιπλέον ισχύει ότι χ + 2y + 3z = 8 , να βρείτε τους πραγματικούς αριθμούς x, y και z . Λί1ση Πολλαπλασιάζουμε την δεύτερη ισότητα επί 3 και την αφαιρούμε από την πρώτη, οπότε λαμβάνουμε

Τότε η δεύτερη ισότητα γίνεται: οπότε διακρίνουμε δύο περιπτώσεις:

( χ - y )3 = 8z3 <::> χ - y = 2z . ( I) 2zxy = 6z3 , (2)

ΕΥΚΛΕΙΔΗΣ Β' 75 τ.3/8

Page 12: Ευκλειδης Β 75

---------- Μαθηματικοί Διαγωνισμοί- Μαθηματικές Ολυμπιάδες ---------( i) Έστω z ::�; Ο . Τότε η (2) είναι ισοδύναμη με την σχέση xy = 3z2 , (3) Από τις ( I ) και (3) , προκύπτει η σχέση χ ( χ - 2z) = 3z2 <=:> χ 2 - 2zx - 3 z2 = Ο<=:> χ = 3z ή χ = -z , οπότε θα είναι χ = 3z, y = z ή χ = -z, y = -3z .

{( χ - y ) ( χ 2 + xy + y2 ) = ο} (ii) Για z = Ο οι δύο πρώτες εξισώσεις γίνονται : <=> χ - y = Ο ή χ = y = Ο , xy ( x - y) = 0 οπότε προκύπτει ότι: χ = y, ανεξάρτητα από το z . (β) Για χ = 3z, y = z η εξίσωση χ + 2y + 3z = 8 γίνεται 8z =8 <=:> z = 1 , οπότε έχουμε ότι ( x, y, z) = (3, 1 , 1 ) , ενώ για χ = -z, y = -3z , η εξίσωση γίνεται -4z = 8 <=> z = -2, οπότε έχουμε ότι ( x , y, z) = (2, 6, -2 ) . Για z = Ο , είναι χ = y , οπότε από την εξίσωση χ + 2y + 3z = 8 προκύπτει ότι ( x , y, z) = ( � ,� , 0) .

Β' Λυκείου Πρ6βλημα 1 Να προσδιορίσετε όλες τις τριάδες (x, y , z) πραγματικών αριθμών που είναι λύσεις του συστή­

ματος:

.\\)ση

χ3 + y3 = 65z3 x2y + xy2 = 20z3 x - y + 2z = 10 .

Πολλαπλασιάζουμε την δεύτερη εξίσωση επί 3 και την προσθέτουμε στην πρώτη , οπότε λαμβάνουμε την εξίσωση ( χ + y γ = 1 25 z3 <=> χ + y = 5 z . ( 1 ) Τότε η δεύτερη εξίσωση γίνεται 5 zxy = 20z3 , (2) οπότε διακρίνουμε δύο περιπτώσεις: (ί) Έστω z ::�; Ο . Τότε από την εξίσωση (2) λαμβάνουμε: xy = 4z2 • (3) Από τις ( 1 ) και (3) , προκύπτει η εξίσωση χ ( 5z - χ ) = 4z2 <=:> χ2 - 5 zx + 4z2 = Ο<=:> χ = 4z ή χ = z , οπότε θα είναι χ = 4z, y = z ή χ = z, y = 4z . Για χ = 4z, y = z η τρίτη εξίσωση του συστήματος γίνεται 5 z = 1 Ο <=:> z = 2 , οπότε το σύστημα έχει τη λύση ( χ , y, z) = ( 8, 2, 2 ) , ενώ για χ = z, y = 4z η τρίτη εξίσωση γίνεται -z = 1 0 <=:> z = - 1 0, οπότε το σύστημα έχει τη λύση ( x , y, z) = ( - 1 0, -40, - 1 0) . (ii) Για z = Ο οι δύο πρώτες εξισώσεις γίνονται: {( χ + y)( χ2 - xy + y2 ) = Ο}<=> χ + y = 0 ή χ = y = O <=>χ = -y , xy( χ + y) = 0 οπότε από την τρίτη εξίσωση προκύπτει ότι ( χ , y, z) = ( 5, -5, Ο) . Πρόβλημο 2 Δίνεται οξυγώνιο και σκαληνό τρίγωνο ABC , Κ τυχόν σημείο στο εσωτερικό του και τα ύψη του AH1 , BH2 ,C H3 • Ο περιγεγραμμένος κύκλος του τριγώνου ΑΗ2 Η3 τέμνει την ημιευθεία ΑΚ στο σημείο Κ 1 , ο περιγεγραμμένος κύκλος του τριγώνου ΒΗ 1 Η3 τέμνει την ημιευθεία ΒΚ στο σημείο Κ2 και ο περιγεγραμμένος κύκλος του τριγώνου C H1 Η2 τέμνει τη ημιευθεία C K στο σημείο Κ3 • Να aποδείξτε ότι τα σημεία Κ 1 ,Κ2 ,Κ3 , Η και Κ είναι ομοκυκλικά ( δηλαδή ανήκουν στον ίδιο κύ­κλο), όπου Η είναι το ορθόκεντρο του τριγώνου ABC . . \\)ση Έστω ( c 1 ) ο περιγεγραμμένος κύκλος του τριγώνου ΑΗ 2 Η 3 , ( c2 ) ο περιγεγραμμένος κύκλος του τριγώ­νου ΒΗ1Η3 και (c3 ) ο περιγεγραμμένος κύκλος του τριγώνου CH1 Η2 •

ΕΥΚΛΕΙΔΗΣ Β' 75 τ.3/9

Page 13: Ευκλειδης Β 75

---------- Μαθηματικοί Διαγωνισμοί- Μαθηματικές Ολυμπιάδες ---------

Σχήμα 4

ABC .

Το τετράπλευρο ΑΗ2ΗΗ3 εί­ναι εγγράψιμο, οπότε ο κύ­κλος ( c 1 ) περνάει από το ση­μείο Η . Το τετράπλευρο ΒΗ1ΗΗ3 εί­ναι εγγράψιμο, οπότε ο κύ­κλος ( c2 ) περνάει από το ση­μείο Η . Το τετράπλευρο CH1HH2 εί­ναι εγγράψιμο, οπότε ο κύ­κλος ( c3 ) περνάει από το ση­μείο Η . Τελικά, οι τρεις κύκλοι ( C1 ) , (c2 ) και (c3 ) περνάνε από το ορθόκεντρο Η του τριγώνου

Ο κύκλος (c 1 ) έχει διάμετρο την ΑΗ , οπότε ΗΚ 1 .l ΑΚ1 , δηλαδή το σημείο Κ1 ανήκει στο κύκλο δια­μέτρου ΗΚ . Όμοια αποδεικνύουμε ότι και τα σημεία Κ2 , Κ3 , ανήκουν στον ίδιο κύκλο.

Να αποδείξετε ότι η εξίσωση χ2 + χ + 1 - 2lxl = αχ, α Ε JR, έχει για κάθε α Ε JR δύο διαφορετικές μεταξύ τους λύσεις στο σύνολο JR . Για ποιες τιμές του α οι δύο ρίζες είναι ετερόσημες;

Λόγω της ύπαρξης του lxl , διακρίνουμε δύο περιπτώσεις : (ί)Έστω χ � Ο . Τότε η εξίσωση γίνεται: χ 2 + χ + 1 - 2χ = αχ, α Ε JR <=> χ 2 - (α + 1 ) χ + 1 = Ο, α Ε JR, ( 1 ) η οποία έχει διακρίνουσα Δ = (α + Ι )2 - 4 = (α - 1 )(α + 3 ) . Άρα η εξίσωση ( Ι ) έχει πραγματικές ρίζες, όταν είναι α::::; -3 ή α � 1 . Επειδή το γινόμενο των ριζών είναι Ρ = 1 > Ο οι ρίζες είναι ομόσημες, οπότε για να είναι και οι δύο θετικές πρέπει και αρκεί S = α + 1 > Ο <=> α > - 1 . Επομένως έχουμε:

Για α > 1 , η εξίσωση ( 1 ) έχει δύο ακριβώς διαφορετικές θετικές ρίζες στο JR .

Για α = 1 , η εξίσωση ( 1 ) έχει τη διπλή θετική ρίζα χ = 1 στο JR . Για α < 1 , η εξίσωση ( 1 ) δεν έχει μη αρνητικές ρίζες στο JR .

(ii) Έστω χ < Ο . Τότε η εξίσωση γίνεται: χ 2 + χ + 1 + 2χ = αχ , α Ε JR <=> χ 2 - (α - 3) χ + 1 = Ο, α Ε JR, (2)

η οποία έχει διακρίνουσα Δ = (α - 3 )2 -4 = (α - 5 ) ( α - 1 ) . Άρα η εξίσωση (2) έχει πραγματικές ρίζες όταν είναι α::::; 1 ή α � 5. Επειδή το γινόμενο των ριζών είναι Ρ = 1 > Ο οι ρίζες είναι ομόσημες, οπότε για να είναι και οι δύο αρνητικές πρέπει και αρκεί S = α - 3 < Ο <=> α < 3 . Επομένως έχουμε:

ο Για α < 1 , η εξίσωση (2) έχει δύο ακριβώς διαφορετικές αρνητικές ρίζες στο JR Για α = 1 , η εξίσωση (2) έχει τη διπλή αρνητική ρίζα χ = - 1 στο JR . Για α > 1 , η εξίσωση (2) δεν έχει αρνητικές ρίζες στο JR.

Από τις περιπτώσεις ( 1 ) και (2) προκύπτει ότι η δεδομένη εξίσωση έχει, για κάθε α Ε JR , δύο πραγματι­κές ρίζες διαφορετικές μεταξύ τους, οι οποίες είναι ετερόσημες για α = 1 .

μυ -' Να λύσετε στους πραγματικούς αριθμούς την εξίσωση .J2x2 + 3χ + 2 - z.Jx2 + χ + 1 = χ + 1 .

ΕΥΚΛΕΙΔΗΣ Β' 75 τ.3/10

Page 14: Ευκλειδης Β 75

---------- Μαθηματικοί Διαγωνισμοί- Μαθηματικές Ολυμπιάδες ---------

. 1'!j�)'η Κατ' αρχή παρατηρούμε ότι ισχύει: 2χ 2 + 3χ + 2 > Ο και χ 2 + χ + 1 > Ο, για κάθε χ ε IR .

Αν θέσουμε a = �2χ 2 + 3χ + 2 , b = �χ2 + χ + 1 , χ ε IR , τότε λαμβάνουμε :

a 2 - b2 = ( 2χ 2 + 3χ + 2 ) - (χ 2 + χ + 1 ) = χ 2 + 2χ + I = (χ + I )2 , οπότε από τη δεδομένη εξίσωση προκύπτει η εξίσωση με αγνώστους a, b ,

a2 - b2 = ( a - 2b )2 <=> 4ab - 5b2 = Ο <=> b ( 4a - 5b ) = Ο <=> 4a = 5b, αφού είναι b 1= Ο . 'Ετσι έχουμε την εξίσωση 4�2χ 2 + 3χ + 2 = 5�χ2 + χ + 1 , της οποίας τα δύο μέλη είναι θετικά, για κάθε χ ε IR , οπότε είναι ισοδύναμη με την εξίσωση

( ) ( ? ) -23±3J37 1 6 · 2χ2 + 3χ + 2 = 25 · χ - + χ + 1 <::> 7χ2 + 23χ + 7 = 0 <=> χ = 1 4 .

Μ 'λ δ ' ' ' -23 + 3J37 δ λ θ ' ξ' ' δ ' ' ε ε εγχο ιαπιστωνουμε οτι η τιμη χ = εν επα η ευει την ε ισωση, οποτε η μονα ικη ρι-1 4

ζ ' -23 - 3J37 Α ' θ ' ' ' ' 2b Ο ' α της ειναι η χ = . υτο α μπορουσε να προκυψει και απο τη σχεση a - < η οποια 1 4

αληθεύει για κάθε χ ε IR , οπότε πρέπει να είναι χ < - 1 .

Η ακολουθία a., η ε Ν*, ορίζεται αναδρομικά από τις σχέσεις a.+ 1 = a. + kn, η ε Ν*, όπου k θετικός ακέραιος και a 1 = 1 . Να βρείτε για ποια τιμή του k ο αριθμός 201 1 είναι όρος της ακολουθίας a. ,η ε Ν*.

Από τη δεδομένη αναδρομική σχέση έχουμε a 1 = 1 a2 = a 1 + k a3 = a2 + 2k

an-1 = an-2 + (η - 2 ) k a . = an-1 + (η - 1 ) k

από τις οποίες με πρόσθεση κατά μέλη λαμβάνουμε k (η - 1 ) η an =l+k ( l + 2 + . . . + η - 1 ) = 1 +k ( 1 + 2 + . . . + η - 1 ) = 1 + . 2

Επομένως, αρκεί να προσδιορίσουμε τις τιμές των k και η για τις οποίες ισχύει η ισότητα: k (η - 1 ) η a. = 1 + 2 = 20 1 1 <=> k (η - l ) η = 4020 <=> k (η -l ) η = 1 · 2 · 2 · 3 · 5 · 67

<=> (η, k ) = ( 2, 20 1 0) ή ( η, k ) = ( 3, 670 )ή (η, k ) = ( 4, 335 )ή ( η , k ) = ( 5, 20 1 ) ή (η, k ) = ( 6, 1 34) Επομένως, για k = 20 1 Ο είναι a2 = 20 1 1 , για k = 670 είναι a3 = 20 1 1 , για k = 335 είναι a4 = 20 1 1 , για k = 20 1 είναι a5 = 20 1 1 και για k = 1 34 είναι a6 = 20 1 1 .

Δίνεται οξυγώνιο και σκαληνό τρίγωνο ABC και έστω Μ1 , Μ 2 , Μ3 τυχόντα σημεία των πλευρών του ΒC, ΑC,ΑΒ , αντίστοιχα. Έστω ακόμη τα ύψη του AH1 ,BH2 ,CH3 • Να αποδείξετε ότι οι περιγε­γραμμένοι κύκλοι των τριγώνων ΑΗ2 Η3 , ΒΜ1Η3 , CM1H2 περνάνε από το ίδιο σημείο (έστω Κ1 ), οι

ΕΥΚΛΕΙΔΗΣ Β' 75 τ.3/11

Page 15: Ευκλειδης Β 75

---------- Μαθηματικοί Διαγωνισμοί- Μαθηματικές Ολυμπιάδες --------­

περιγεγραμμένοι κύκλοι των τριγώνων ΒΗ1 Η3 , ΑΜ2Η3 , CM2H1 περνάνε από το ίδιο σημείο (έστω Κ2 ) και οι περιγεγραμμένοι κύκλοι των τριγώνων CH1 Η2 , ΑΜ3Η2 , ΒΜ3Η1 περνάνε από το ίδιο σημείο (έστω Κ3 ). Στη συνέχεια να αποδείξετε ότι οι ευθείες ΑΚ 1 , ΒΚ2 , CK3 συντρέχουν, δηλαδή περνάνε από το ίδιο σημείο, αν, και μόνο αν, οι ευθείες ΑΜ1 , ΒΜ2 , CM3 συντρέχουν.

Β

Λύση

Α Έστω ( c ι ) ο περιγεγραμμένος κύκλος

Σχήμα 5

του τριγώνου Β Μι Η3 , (c2 ) ο περιγε­γραμμένος κύκλος του τριγώνου CΜι Η 2 , (c3 ) ο περιγεγραμμένος κύ-κλος του τριγώνου Α Η2 Η3 και (c) ο περιγεγραμμένος κύκλος του εγγράψι­μου τετραπλεύρου BH3 H2C . Θεωρώντας τις τέμνουσες ΑΒ και AC του κύκλου ( c) , συμπεραίνουμε :

ΑΒ · ΑΗ3 = AC · A H2 . Το γινόμενο όμως ΑΒ · ΑΗ3 εκφράζει τη δύναμη του σημείου Α ως προς το κύκλο (c ι ) ενώ το γινόμενο AC · ΑΗ2 εκφράζει τη δύναμη του σημείου Α ως προς το κύκλο (c2 ) . Άρα το σημείο Α ,

ανήκει στον ριζικό άξονα των κύκλων ( c ι ) και ( c2 ) . Έστω τώρα ότι οι κύκλοι (c ι ) και (c2 ) τέμνονται στο σημείο Κ ι (εκτός βέβαια από το σημείο Μ ι ) . Τό­τε η ευθεία που ορίζουν τα σημεία αυτά (δηλαδή τα Κι και Μ ι ) είναι ο ριζικός άξονας των κύκλων (c ι ) και (c2 ) . Από τους παραπάνω συλλογισμούς προκύπτει ότι τα σημεία Α, Κ ι και Μι είναι συνευθειακά. Θα αποδείξουμε ότι και ο κύκλος ( c3 ) περνάει από το σημείο Κ ι , δηλαδή ότι το τετράπλευρο

ΑΗ2 Κ ιΗ3 είναι εγγράψιμο.

Από το εγγράψιμο τετράπλευρο BH3 H2C έχουμε: φ =Β. Από το εγγεγραμμένο τετράπλευρο ΒΜιΚι Η3 έχουμε : ω= Β. Άρα είναι ω= φ και κατά συνέπεια το τετράπλευρο ΑΗ2 Κ ι Η3 είναι εγγράψιμο . Με όμοιο τρόπο αποδεικνύουμε ότι και οι δύο άλλες τριάδες κύκλων, περνάνε από το ίδιο σημείο. Προφανώς τώρα οι ευθείες ΑΚι , ΒΚυCΚ3 συντρέχουν, αν, και μόνο αν, συντρέχουν οι ευθείες AMι , BMz , CM3 (δεδομένου ότι τα σημεία Α, Κι , Μι , τα σημεία Β, Κ2 , Μ2 και τα σημεία C, K3 , M3 , είναι συνευθειακά.

Π ρόβλη μα 3

Α ν a, b, χ, y ε JR με { a, b) :;t: {Ο, Ο) και {χ, y ) :;t: {Ο, Ο) και ισχύουν a ( χ2 - y2 ) - 2bxy = χ { a2 - b2 ) - 2aby b( χ2 - y2 ) + 2axy = y { a2 - b2 ) + 2abx,

να αποδείξετε ότι χ = a και y = b . ΛίJση Σύμφωνα με τον ορισμό της ισότητας μιγαδικών αριθμών, προκύπτει ότι το σύστημα των δύο δεδομένων εξισώσεων είναι ισοδύναμο με την εξίσωση :

[ a (χ 2 - y2 ) - 2bxy J + [ b ( χ 2 - y2 ) + 2axy} = [χ ( a 2 - b2 ) - 2aby J + [ y ( a 2 - b 2 ) + 2abx} � ( a + bi ) · [ (χ 2 - y2 ) + 2xyi J = [ ( a 2 - b2 ) + 2abi J · (χ + yi ) � ( a + bi ) · (χ + yi )2 = ( a + bi γ · (χ + yi ) �χ + yi = a + bi (αφού (α, β ) :;e ( Ο, Ο) και (x, y) :;e (Ο, Ο)) � χ = a, y = b.

ΕΥΚΛΕΙΔΗΣ Β' 75 τ.3/12

Page 16: Ευκλειδης Β 75

---------- Μαθηματικοί Διαγωνισμοί- Μαθηματικές Ολυμπιάδες ---------Π ρόβλη μα 4 Σημείο Μ βρίσκεται στο εσωτερικό κύκλου c( O, r) , όπου r = 1 5cm , σε απόσταση 9cm από το κέ-ντρο του κύκλου. Να βρείτε τον αριθμό των χορδών του κύκλου C( O, r) που περνάνε από το ση­μείο Μ και το μήκος τους είναι ακέραιος αριθμός. ΛίJση Θεωρούμε τη χορδή ΑΒ που περνάει από το σημείο Μ και το κέντρο Ο του κύκλου, καθώς και την κά­θετη προς αυτήν χορδή ΓΜΔ , οπότε το σημείο Μ είναι το μέσο της χορδής Γ Δ. Η χορδή ΑΒ έχει ακέ­ραιο μήκος 30cm . Από το θεώρημα τεμνομένων χορδών έχουμε ότι:

Γ

( ΓΔ )2 ( ΓΔ )2 ΓΔ ΓΜ · ΜΔ = ΑΜ · ΜΒ � 2 = 6 · (9 + 1 5 ) � 2 = 1 44 � 2 = 1 2 � ΓΔ = 24.

r�" ' . / /

- · )�� !

Σχήμα 6

Έτσι μέχρι τώρα έχουμε βρει δύο χορδές του κύκλου C (O, r ) που περνάνε από το σημείο Μ και έχουν ακέραιο μήκος. Θεωρούμε τυχούσα χορδή ΚΛ του κύκλου C ( O, r ) που περνάει από το Μ και έστω ΜΕ = χ , ΜΟΕ = θ, όπου Ε είναι το μέσο της ΚΛ, σχήμα

6. Α ν υποθέσουμε ότι Ο � θ � 2: , τότε έχουμε θεωρήσει όλες τις 2 χορδές του κύκλου C (Ο, r ) που περνάνε από το Μ και τα άκρα

τους Κ και Λ βρίσκονται στα ελάσσονα τόξα ΑΓ και ΒΔ , αντί­στοιχα . Για κάθε μία από αυτές τις χορδές αντιστοιχεί και μία α­κόμη που είναι η συμμετρική της ως προς τη διάμετρο ΑΒ. Για τη χορδή ΚΛ, αν συμβολίσουμε το μήκος της ως e ( θ) , έχουμε

e (θ ) = 2.J225 - 8 1συν2θ , ο � θ � 2: . 2 Επειδή είναι e ' ( θ) =

.J 8 1ημ2θ > 0, θ ε (ο,2:) , έπεται ότι η συνάρτηση e (θ) είναι γνησίως αύ-

225 - 8 1συν2θ 2

ξουσα στο διάστημα [Ο, �] , οπότε η συνάρτηση e ( θ ) έχει σύνολο τιμών το διάστημα

[ ( ( Ο ) , ( ( �)] = [ 24, 30] . Άρα το μήκος της χορδής ΚΛ μπορεί να πάρει όλες τις ακέραιες τιμές του δια­

στήματος [24, 30] . Αν λάβουμε υπόψιν και τη συμμετρική χορδή της ΚΛ ως προς τη διάμετρο ΑΒ, τότε τα πέντε μήκη 25 , 26, 27 , 28 , 29 λαμβάνονται δύο φορές το καθένα, ενώ τα μήκη 24 και 30 λαμβάνονται από μία φορά. Έτσι έχουμε συνολικά 1 2 χορδές που περνάνε από το Μ με ακέραιο μήκος.

Παρατή ρηση I : Θα μπορούσαμε επίσης να χρησιμοποιήσουμε το θεώρημα μέγιστης και ελάχιστης τιμής

για τη συνεχή συνάρτηση e ( θ) = 2.J225 - 8 1συν2θ , Ο � θ � 2: , η οποία έχει ελάχιστη τιμή την e ( o) = 24 2 και μέγιστη τιμή την e ( �) = 30 . Αυτό προκύπτει από την παρατήρηση ότι τα μήκη των χορδών είναι

αντιστρόφως ανάλογα από τα αποστήματά τους και ότι το μέγιστο aπόστημα λαμβάνεται για θ = Ο , ενώ

το ελάχιστο aπόστημα λαμβάνεται για θ = 2: . 2 Π αρατή ρηση 2 : Σημειώνουμε ακόμη ότι οι χορδές με ακέραια μήκη 25 , 26, 27 , 28 , 29, μπορούν να κα­τασκευαστούν γεωμετρικά, αφού αν θέσουμε ΚΜ = χ και ΜΛ = y , τότε έχουμε

x + y = m, m ε {25, 26, 27, 28, 29} και xy = 1 44 = 1 22 • Έτσι εξασφαλίζουμε την ύπαρξη αυτών των χορδών με ακέραιο μήκος, χωρίς τη χρήση του διαφορικού λογισμού.

ΕΥΚΛΕΙΔΗΣ Β . 75 τ.3/13

Page 17: Ευκλειδης Β 75

ΗΟΜΟ MATHEMATICUS Η Homo Mathematicus είναι μια στήλη στο περιοδικό μας, με σκοπό την ανταλλαγή απόψεων και την ανάπτυξη

προβληματισμού πάνω στα εξής θέματα: I ) Τι είναι τα Μαθηματικά, 2) Πρέπει ή όχι να διδάσκονται, 3) Ποιοι είναι οι κλάδοι των Μαθηματικών και ποιο το αντικείμενο του καθενός, 4) Ποιες είναι οι εφαρμογές τους, 5) Ποιες επιστήμες ή κλάδοι επιστημών απαιτούν καλή γνώση των Μαθηματικών για να μπορέσει κάποιος να τους σπουδάσει.

rιu το1 ι.: ι>ι"·ψγ{π::,;: της σηj i.ι1 .: : παράκληση ! τα κείμενα της στήλης αυτής, ως προς το περιεχόμενό τους και ως προς το

επίπεδό τους, θα πρέπει να είναι συμβιβαστά με τα ενδιαφέροντα και το επίπεδο κατανόησης από μέρους των παιδιών.

Ι . "τι ι:ίι •αι τα ι�ΙαΟηματιιιά: "

Δύο κορυφαίοι επιστήμονες ο Jean-Pieπe Changeux (Μοριακός Νευροβιολόγος, Γαλλική Ακαδημία Επιστημών) και ο Alain Connes (Μαθηματικός στο Col!ege de France, βραβείο Fieldes), συνομιλούν, μεταξύ τους, για τη φύση των Μαθηματικών. Απ' αυτή την εξαιρετικά ενδιαφέρουσα συνομιλία παραθέτουμε παρακάτω ένα μικρό απόσπασμα.

«JPC. Ας έρθουμε τώρα στη φύση των

μαθηματικών αντικειμένων. Έχουν υποστηριχθεί

δύο εκ διαμέτρου αντίθετες απόψεις: ο «ρεαλισμός»

και ο «κονστρουκτιβισμός». Για τον «ρεαλιστή», που εμπνέεται απ ' ευθείας από τον Πλάτωνα, ο

κόσμος είναι γεμάτος από Ιδέες η πραγματικότητα

των οποίων διαφέρει από την αισθητή πραγματικότητα. Πολλοί σύγχρονοι μαθηματικοί

θεωρούν τους εαυτούς τους «ρεαλιστές». Ο Dίeudonne, για παράδειγμα, γράφει στο βιβλίο του:

«Είναι αρκετά δύσκολο να περιγράψουμε τις ιδέες

αυτών των μαθηματικών, οι οποίες εξ άλλου

ποικίλλουν από τον έναν στον άλλο. Δέχονται ότι τα

μαθηματικά αντικείμενα κατέχουν μια "πραγματικό­τητα " που διαφέρει από την αισθητή πραγματικότη­τα (παρεμφερή, ίσως, με εκείνη που ο Πλάτων

αναγνώριζε στις "Ιδέες" του;)». Ο Cantor, αυτός ο τόσο μεγάλος μαθηματικός, έγραψε: «Η ύψιστη τελειότητα του Θεού είναι η δυνατότητά του να

δημιουργεί ένα άπειρο σύνολο, και η απέραντη

μεγαλοθυμία του τον ώθησε να το δημιουργήσει».

Βρισκόμαστε στην καρδιά του mathesίs dίνίnα, στην

καρδιά της μεταφυσικής. Αυτό είναι κάτι που μας

εκπλήσσει όταν πρόκειται για σοβαρούς

1 1 . 'Ά υ η) το ξ::ρατ::; "

�:π ιμi:ληο: Κεριισιφ ίδης Γι6ηη

επιστήμονες. Ήδη ο Descartes, μι}"ώντας για τη

γεωμετρία, αναφερόταν στη μεταφυσική: « . . . όταν

φαντάζομαι ένα τρίγωνο», έγραφε, «μολονότι ένα

τέτοιο σχήμα δεν υπάρχει ίσως σε κανένα άλλο μέρος του κόσμου πέρα από τη σκέψη μου, ούτε

υπήρξε ποτέ, αυτό δεν σημαίνει ότι παύει να υπάρχει μια ορισμένη φύση ή μορφή ή ουσία που

καθορίζεται απ αυτό το σχήμα, που είναι

αναλλοίωτη και αιώνια και την οποία ούτε την

επινόησα ούτε εξαρτάται με κανένα τρόπο από το

πνεύμα μου» [R. Descartes, «Meditations metaphysiqus», V, Gallimard, collection Pleiade, 1 970, σελ. 3 1 1 ]

Για τους «κονστρουκτιβιστές», τα μαθηματικά

αντικείμενα είναι ορθολογικές οντότητες που υπάρχουν μόνο μέσα στη σκέψη του μαθηματικού

και όχι σ ' έναν πλατωνικό κόσμο, ανεξάρτητα από την ύλη · δεν υπάρχουν παρά μόνο μέσα στους

νευρώνες και στις συνάψεις των μαθηματικών που

τα δημιουργούν, καθώς κι εκείνων που τα

κατανοούν και τα χρησιμοποιούν. Σε μια εμφανώς

ακραία εκδοχή, αυτή την άποψη την έχουμε ξανασυναντήσει στους εμπειριστές φιλοσόφους,

όπως στον Locke ή τον Hume. Ο Hume, λόγου

χάρη, γράφει ότι «όλες οι ιδέες μας είναι αντίγραφα

των εντυπώσεών μας». Γι ' αυτόν, τα γεωμετρικά αντικείμενα προέρχονται αποκλειστικά από την εμπειρία . . . . » [πηγή : «ΤΑ ΜΑΘΗΜΑτΙΚΆ ΚΑΙ Ο ΕΓΚΕΦΑΛΟΣ», Jeane-Pieπe Changeux και Alain Connes, εκδόσεις ΚΆΤΟΠΤΡΟ (Σειρά: Εγκέφαλος και Νόηση), Αθήνα 1 995]

Γιι'ί\'νης Κ�:pασιφ ί<iης

Πόσα δεκαδικά ψηφία του αριθμού π (3 , 1 4 . . . ) έχουν υπολογιστεί μέχρι σήμερα [η απάντηση στο τέλος της στήλης]

1 1 1 . "οι συ ιι:.ρ ;ιά τι:ς τη ς στιί/.ης γρ άφο ιJ ι •-:φ ω το ι) ν "

Γ/μ (:Υτο Ο/μα: « Ειι()ι)μωιι το υ t.'ι •αι·ί.\tcιι του ιVίιιο ιJ Λ ι;γ::ρ ο ι! fπα ι •;:π. Λ ΙJ (:Υ ιιf fΜ ετά.φραση απ() τα γαλλικά : ΣcJ.νη ΚαπρCι.γκου/

Προ/.ι:;•()μιτα. Ο Νίκος Λυγερός συνηθίζει να μας ξαφνιάζει ευχάριστα, πότε με τα βαθυστόχαστα

ΕΥΚΛΕΙΔΗΣ Β' 75 τ.3/14

Page 18: Ευκλειδης Β 75

ΗΟΜΟ MATHEMAτiCUS

σχόλια πάνω σε μαθηματικά θέματα, πότε με τους στίχους του, πότε με τα ζωγραφικά του έργα, πότε . . . . . Σήμερα δημοσιεύουμε μια ποιητική αναφορά στις τελευταίες ημέρες της ζωής του πρόωρα χαμένου γίγαντα της μαθηματικής σκέψης, του Evariste Galois .

J-.- � ·(.Ι{ψη; • ΠΝ! 1-.ΊιαΓi\/f' Γ .• ..

Όφειλε να συνεχίσει να γράφει ν' αγγίξουν την αλήθεια ως να 'ταν τούτο ένα καθήκον αυτής της παράξενης ομάδας.

παρά την κούραση και τη νύχτα Κι έπειτα δεν υπήρχε επομένη τουλάχιστον όσο η μελάνη ζούσε. εκτός από κείνη της μονομαχίας

Διότι αυτό ήταν το τέλος, αυτό ήταν σίγουρο. όπου το παράλογο νίκησε το ορθόν Όφειλε ν' αφήσει ένα ίχνος δίχως να 'χει να δικαιολογηθεί

ως να 'ταν τούτο διαθήκη για τη διάπραξη ενός εγκλήματος προκειμένου οι άλλοι μαθηματικοί κατά της aνθρωπιάς ενός ανθρώπου.

[πηγή : από την ηλεκτρονική αλληλογραφία μας με τον Ν . Λυγερό]

J c μ ' ' ·' · . . • . . . Ο φίλος μου ο Αντώνης, όταν του τηλεφώνησα για να του ευχηθώ για τη γιορτή του, μου είπε να διαβάσω ένα δημοσίευμα αθηναϊκής εφημερίδας το οποίο, κατά τη γνώμη του, παρουσιάζει ενδιαφέρον. Πραγματικά, διάβασα το άρθρο και επειδή συμμερίστηκα το ενδιαφέρον αυτού του άρθρου, σας το παρουσιάζω

«Για τους περισσότερους το μεσημεριανό μοιράσουν. «Τρώγαμε μαζί τουλάχιστον μία φορά γεύμα με έναν συνάδελφο είναι μια χαλαρή υπόθε- την εβδομάδα»λέει ο κ. Μέιμπρι αναφερόμενος ση όπου οι προβληματισμοί εξαντλούνται στο ποιο στις αρχές της δεκαετίας του 1 990, όταν οι δύο πιάτο θα διαλέξουν και αν θα πάρουν γλυκό . Για μαθηματικοί εργάζονταν στο Πολιτειακό Πανεπι­τον Ρικ Μέιμπρι και τον Πολ Ντάιερμαν όμως το στήμιο της Λουιζιάνας. «0 ένας από τους δυο μας ζήτημα δε\' ήταν ποτέ τόσο απλό. Τους είναι έφερνε ένα τετράδιο και aρχίζαμε να κάνουμε αδύνατον. για παράδειγμα, να δουν στο τραπέζι σχεδιαγράμματα αφήνοντας το φαγητό μας να τους μια πίτσα χωρίς να προσπαθήσουν να βρουν κρυώνει» . τη λύση στο μαθηματικό πρόβλημα του πώς να τη

Ί � Το πρόβλημα που τους απασχολούσε ήταν το

εξής: ας υποθέσουμε ότι στη βιασύνη του ο σερβιτόρος κόβει την πίτσα εκτός κέντρου, με όλες τις τομές να διασταυρώνονται σε ένα σημείο σχηματίζοντας ίσες γωνίες με τη γειτονική τους. Οι εκτός κέντρου τομές σημαίνουν ότι τα κομμάτια δεν θα έχουν το ίδιο μέγεθος. Επομένως δύο άτομα που παίρνουν εναλλάξ διαδοχικά κομμάτια, θα έχουν φάει ίσα μερίδια όταν τελειώσει η πίτσα και, αν όχι, ποιος θα έχει φάει περισσότερο;

Φυσικά, θα μπορούσε κανείς να λύνει κάθε φορά το συγκεκριμένο πρόβλημα υπολογίζοντας την επιφάνεια κάθε κομματιού και προσθέτοντας τα κομμάτια του καθενός μεταξύ τους. Οι δύο ερευνη­τές είναι όμως μαθηματικοί και δεν αρκούνται σε τέτοιου είδους λύσεις : ήθελαν μια θεωρητική κατασκευή χωρίς ακριβείς υπολογισμούς, έναν κανόνα που θα ισχύει πάντοτε και θα μπορεί να εφαρμόζεται για κάθε στρογγυλή πίτσα.

Η προσπάθειά τους ξεκίνησε το 1 994, όταν ο κ. Ντάιερμαν έδειξε στον κ. Μέιμπρι μια διορθωμένη εκδοχή του θεωρήματος της πίτσας που επανεμφανίστηκε στο «Mathematics Magazine» προκαλώντας τους αναγνώστες να

αποδείξουν δύο συγκεκριμένες περιπτώσεις του προβλήματος, αυτή στην οποία η πίτσα κόβεται με 3 τομές και αυτή στην οποία κόβεται με 5 .

Ο κ . Ντάιερμαν έλυσε γρήγορα το πρόβλημα των 3 τομών, το οποίο οι συγγραφείς του άρθρου είχαν λύσει αλλά δεν aποκάλυπταν στο δημοσίευ­μα. Στη συνέχεια οι δύο συνεργάτες βρήκαν τη λύση για τις περιπτώσεις των 5 και των 7 τομών (οι οποίες έδιναν το ίδιο αποτέλεσμα με τις 3) . Ενθουσιασμένοι με την επιτυχία τους, θεώρησαν ότι είχαν βρει την τεχνική που θα έλυνε πλήρως το πρόβλημα. Σε ένα μονό αριθμό τομών, τα aντιδιαμετρικά αντίθετα κομμάτια πηγαίνουν σε διαφορετικούς συνδαιτυμόνες, οπότε συγκρίνοντας κανείς τα μεγέθη τους μπορεί να βρει, για αυτά τα δύο, ποιος παίρνει περισσότερο και πόσο και στη συνέχεια να περάσει στο επόμενο aντιδιαμετρικά αντίθετο «ζευγάρι» .

Παρ ' ότι ακούγεται απλό, στην πράξη αποδείχθηκε σχεδόν αδύνατον να βρεθεί μια λύση που να καλύπτει όλους τους πιθανούς μονούς αριθμούς τομών. Οι δύο μαθηματικοί χρησιμοποίη­σαν ένα γεωμετρικό τέχνασμα για να aπλοποιή­σουν τη διαδικασία, εισάγοντας ένα παραλληλεπί-

ΕΥΚΛΕΙΔΗΣ Β' 75 τ.3/1 5

Page 19: Ευκλειδης Β 75

------------- ΗΟΜΟ MATHEMAτiCUS -------------

πεδο που σχηματίζεται από κάθε τομή και μια παράλληλή της γραμμή η οποία περνάει από το κέντρο της πίτσας. Και πάλι όμως η λύση δεν ήταν ικανοποιητική εφόσον απαιτούσε πολύπλοκους υπολογισμούς. Ήταν, όπως λένε, «άσχημη» .

Όπως συμβαίνει με πολλές μαθηματικές σπαζοκεφαλιές, η απάντηση ήρθε σε στάδια- για διαφορετικές κάθε φορά πιθανές περιπτώσεις του

προβλήματος. Η ευκολότερη προσφέρεται όταν τουλάχιστον μία τομή περνάει από το κέντρο της πίτσας: ένα γρήγορο σχήμα μπορεί να δείξει ότι στην περίπτωση αυτή τα aντιδιαμετρικά κομμάτια είναι συμπληρωματικά μεταξύ τους οπότε μοιρά­ζονται ίσα ανάμεσα στους δύο συνδαιτυμόνες, ανεξάρτητα από το πόσες είναι οι τομές.

Η ευ l(ολία των ζυγc.ί)ν

Τι γίνεται όμως όταν καμία τομή δεν περνάει Ο πρώτος ορίζει ότι αν η πίτσα κοπεί σε ένα από το κέντρο; Αν η πίτσα κόβεται μία φορά, η δεδομένο σημείο με ζυγό αριθμό τομών μεγαλύτε­απάντηση είναι προφανής με το μάτι: όποιος τρώει ρο του 2, θα μοιραστεί ίσα ανάμεσα στους δύο το κέντρο τρώει περισσότερο. Αν γίνουν δύο τομές, συνδαιτυμόνες που παίρνουν εναλλάξ διαδοχικά που δίνουν τέσσερα κομμάτια, το αποτέλεσμα είναι κομμάτια. Προτάθηκε για πρώτη φορά για 4 τομές το ίδιο : όποιος φάει το κομμάτι που περιλαμβάνει (8 κομμάτια) από κάποιον Λ. Τζ. Απτον το 1 967 το κέντρο τρώει περισσότερη πίτσα. Όσο όμως οι στο περιοδικό «Mathematics Magazine», ενώ η τομές αυξάνονται, παρουσιάζονται διάφορες λύση που όριζε ότι η πίτσα μοιράζεται ίσα για ανωμαλίες και η λύση τους οδήγησε, με τον χρόνο, οποιονδήποτε αριθμό ζυγών τομών ήλθε έναν στον σχηματισμό των τριών γενικών κανόνων που χρόνο αργότερα ως απάντηση στην «πρόκληση» απαρτίζουν το θεώρημα της πίτσας. του Απτον.

Ο σκόπ::λος των μο\'{\)\' Με τους μονούς αριθμούς τομών όμως τα

πράγματα περιπλέκονται. Εδώ το θεώρημα της πίτσας λέει ότι αν κόψει

κανείς την πίτσα με 3 , 7, 1 1 , 1 5 . . . τομές χωρίς καμία τομή να περνάει από το κέντρο της, τότε αυτός που θα πάρει το κομμάτι που περιέχει το κέντρο της πίτσας θα φάει συνολικά περισσότερο. Αν όμως οι τομές είναι 5 , 9 , 1 3 , 1 7 . . . αυτός που θα πάρει το κομμάτι με το κέντρο καταλήγει να τρώει λιγότερο .

Το θεώρημα ισχύει αν κάνει κανείς τους υπολογισμούς, απέβη όμως εξαιρετικά δύσκολο να αποδειχθεί από τους μαθηματικούς. Τόσο δύσκολο ώστε οι κκ. Μέιμπρι και Ντάιερμαν μόλις τώρα κατόρθωσαν να ολοκληρώσουν μια απόδειξη η οποία καλύπτει όλες τις πιθανές περιπτώσεις.

Η προσπάθειά τους ξεκίνησε το 1 994, όταν ο κ. Ντάιερμαν έδειξε στον κ. Μέιμπρι μια διορθωμένη εκδοχή του θεωρήματος της πίτσας που επανεμφανίστηκε στο «Mathematics Magazine» προκαλώντας τους αναγνώστες να από­δείξουν δύο συγκεκριμένες περιπτώσεις του προβλήματος, αυτή στην οποία η πίτσα κόβεται με 3 τομές και αυτή στην οποία κόβεται με 5 . Η «άσχημη» λύση

Ο κ. Ντάιερμαν έλυσε γρήγορα το πρόβλημα

των 3 τομών, το οποίο οι συγγραφείς του άρθρου είχαν λύσει αλλά δεν aποκάλυπταν στο δημοσίευμα. Στη συνέχεια οι δύο συνεργάτες βρήκαν τη λύση για τις περιπτώσεις των 5 και των 7 τομών (οι οποίες έδιναν το ίδιο αποτέλεσμα με τις 3) . Ενθουσιασμένοι με την επιτυχία τους, θεώρη­σαν ότι είχαν βρει την τεχνική που θα έλυνε πλήρως το πρόβλημα. Σε ένα μονό αριθμό τομών, τα aντιδιαμετρικά αντίθετα κομμάτια πηγαίνουν σε διαφορετικούς συνδαιτυμόνες, οπότε συγκρίνοντας κανείς τα μεγέθη τους μπορεί να βρει, για αυτά τα δύο, ποιος παίρνει περισσότερο και πόσο και στη συνέχεια να περάσει στο επόμενο aντιδιαμετρικά αντίθετο «ζευγάρι» .

Παρ ' ότι ακούγεται απλό, στην πράξη απόδεί­χθηκε σχεδόν αδύνατον να βρεθεί μια λύση που να καλύπτει όλους τους πιθανούς μονούς αριθμούς τομών. Οι δύο μαθηματικοί χρησιμοποίησαν ένα γεωμετρικό τέχνασμα για να aπλοποιήσουν τη διαδικασία, εισάγοντας ένα παραλληλεπίπεδο που σχηματίζεται από κάθε τομή και μια παράλληλή της γραμμή η οποία περνάει από το κέντρο της πίτσας. Και πάλι όμως η λύση δεν ήταν ικανοποιητική εφόσον απαιτούσε πολύπλοκους υπολογισμούς. Ήταν, όπως λένε, «άσχημη» .

Ο παριί.γων «κυ0υμ6ς αέρας» Στα χρόνια που ακολούθησαν, ασχολήθηκαν πρόβλημα και τότε όλα άρχισαν να λειτουργούν» .

κατά καιρούς και πάλι με το πρόβλημα, χωρίς όμως Ως τότε οι δύο μαθηματικοί χρησιμοποιούσαν περαιτέρω επιτυχία. Η «φώτιση» ήρθε το 2006, μοντέλα στον ηλεκτρονικό υπολογιστή για τα από­όταν ο κ. Μέιμπρι βρισκόταν για διακοπές στη τελέσματά τους. Μόλις όμως ο κ. Μέιμπρι άφησε Γερμανία. «Ήμουν σε ένα ωραίο ξενοδοχείο, με την τεχνολογία στην άκρη , μπόρεσε να δει το ευχάριστο, δροσερό περιβάλλον και χωρίς πρόβλημα καθαρά. Επιστρέφοντας έβαλε ξανά τον κομπιούτερ»λέει.«Άρχισα να σκέφτομαι ξανά το υπολογιστή να δουλέψει για να βρει όλες τις

ΕΥΚΛΕΙΔΗΣ Β' 75 τ.3/16

Page 20: Ευκλειδης Β 75

------------- ΗΟΜΟ MATHEMAτiCUS -------------

πληροφορίες που χρειάζονταν για να προχωρήσει τις σκέψεις του και τελικά όλα μπήκαν στη θέση τους. Η απόδειξη του θεωρήματος της πίτσας βρέθηκε, λοιπόν, επιτέλους και έχει δημοσιευθεί στην επιθεώρηση Aιηerican Mathematical Monthly (Μάιος 2009). Ποιες πρακτικές εφαρμογές μπορεί να έχει; Προς το παρόν δεν διαφαίνεται καμία, αυτό όμως δεν ανησυχεί τον κ. Μέιμπρι. «Το παράξενο με εμάς τους μαθηματικούς»λέει «είναι ότι συχνά δεν μας ενδιαφέρει αν τα αποτελέσματα έχουν εφαρμογές γιατί τα ίδια τα αποτελέσματα

είναι τόσο όμορφα». Πολλές φορές ωστόσο η χρησιμότητα των λύσεων τέτοιου είδους αφηρημένων μαθηματικών προβλημάτων εμφανίζε­ται με απρόσμενους τρόπους: ένα ξεχασμένο μαθη­ματικό παράδοξο του 1 9ου αιώνα για ένα μορφο­κλασματικό είδος καμπύλης επανήλθε στο προσκή­νιο ως μοντέλο για το σχήμα του ανθρώπινου γονι­διώματος» [πηγή : εφημερίδα "ΤΟ ΒΗ ΜΑ", 1 7/ 1 /20 1 0, © 2009 New

Scientist Magazine, Reed Business Information Ltd]

� \'. Επικαιρ6τητ α : Η πrι.;ιια)σμια μαΟημα τιιιιί ιωι ιγ)τη τα πι: ιιΟι:ί

Στις 5/ 1 0/20 1 0 "έφυγε" ο Israel Moiseeνich Gelfand, ένας από τους πιο επιφανείς μαθηματι­κούς του 20ου αιώνα · πέθανε σε ηλικία 96 ετών.

Ο Gelfand εργάστηκε σε πολλούς τομείς των μαθηματικών, κυρίως στη Συναρτησιακή Ανάλυση, Αρμονική Ανάλυση, Διαφορικές Εξισώσεις και Διακριτά Μαθηματικά. Αργότερα στη ζωή του, άρχισε να ενδιαφέρεται για τη θεωρητική Βιολογία.

Ι I . ,. Η απά ιιτη ση στο: "Α υ η) το ξιiματι:; "

Επιστήμονας της πληροφορικής έσπασε το ρεκόρ υπολογισμού των ψηφίων μιας διάσημης μαθηματικής σταθεράς, του αριθμού π, υπολογίζοντας σχεδόν 2, 7 τρισεκατομμύρια ψηφία που ακολουθούν μετά το 3 , 1 4, κάπου 1 23 δισεκατομμύρια περισσότερα ψηφία σε σχέση με το προηγούμενο ρεκόρ.

Ο Φαμπρίς Μπελάρντ, σύμφωνα με το BBC, χρησιμοποίησε έναν απλό επιτραπέζιο υπολογιστή για να κάνει το νέο υπολογισμό, που του πήρε 1 3 1 μέρες συνολικά. Ο νέος αριθμός-ρεκόρ του π χρειάζεται πάνω από ένα terabyte για να απόθηκευ­τεί σε σκληρό δίσκο .

Τα προηγούμενα ψηφία-ρεκόρ του π είχαν βρεθεί με τη βοήθεια τεράστιων υπερ-υπολογιστών, όμως ο Μπελάρντ υποστηρίζει ότι η δική του μεθο­δος υπολογισμού είναι 20 φορές πιο απότελεσματι­κή .

Το προηγούμενο ρεκόρ με περίπου 2,6 τρισ. ψηφία κατείχε, από τον Αύγουστο του 2009, ο Νταϊσούκε Τακαχάσι του πανεπιστημίου Τσουκού­μπα της Ιαπωνίας και του είχε πάρει 29 ώρες, αλλά με την υποστήριξη ενός σούπερ-κομπιούτερ 2 .000 φορές πιο γρήγορου και χιλιάδες φορές πιο ακριβού από τον κοινό υπολογιστή που χρησιμοποίησε ο Μπελάρντ.

Εκτιμάται ότι αν χρειάζεται περίπου ένα

Εκτός των άλλων ο Gelfand έγραψε περισσότερα από 30 βιβλία, το πιο γνωστό από τα οποία είναι το τετράτομο με τίτλο "Γενικευμένες Συναρτήσεις" .

Για πολλά χρόνια διηύθυνε το περίφημο σεμινάριο Gelfand στη Μόσχα. Μετά την αποδημία του στις ΗΠΑ (αρχές δεκαετίας του ' 90), συνέχισε εκεί τις επιστημονικές του εργασίες

δευτερόλεπτο για να εκφωνηθεί ένας αριθμός, η πλήρης απαρίθμηση φωναχτά όλων των ψηφίων του π θα απαιτούσε πάνω από 49.000 χρόνια !

Ο Μπελάρντ δήλωσε ότι διάβασε το πρώτο του βιβλίο του για τον αριθμό π όταν ήταν 1 4 ετών και έκτοτε παρακολουθούσε ανελλιπώς τις προσπά­θειες υπολογισμού όλο και περισσότερων ψηφίων του.

Όπως είπε, τον ενδιαφέρει ιδιαίτερα η πρακτική πλευρά του ζητήματος, καθώς ορισμένοι από τους αλγόριθμους που απαιτούνται για τον υπολογισμό του π, είναι χρήσιμοι για άλλα πράγμα­τα στους υπολογιστές.

Όπως ανέφερε, σχεδιάζει να δημοσιοποιήσει μια έκδοση του προγράμματος που χρησιμοποίησε για τον υπολογισμό του π, ενώ δεν απέκλεισε να επιμείνει για την ανακάλυψη και άλλων ψηφίων στο μέλλον.

Όπως δήλωσε ο Αιβαρς Πίτερσον, διευθυντής της Μαθηματικής Ένωσης της Αμερικής, το νέο αποτέλεσμα αποτελεί τον τελευταίο κρίκο σε μια μακρά αλυσίδα προσπαθειών να διευρυνθεί το μήκος των γνωστών ψηφίων του «Π» . Μεταξύ άλλων, ο Νεύτων είχε περάσει αρκετό χρόνο προσπαθώντας να βρει και άλλα ψηφία.

[πηγή : εφημερίδα «ΑΓΓΕΛΙΟΦΟΡΟΣ», 8/ 1 /20 1 0]

ΕΥΚΛΕΙΔΗΣ Β' 75 τ.3/17

Page 21: Ευκλειδης Β 75

'I

�ι r' r ]1. ! ('\ ι , I ·' ' - -

1 1 11 (Υ ·π� -� ιr;:1 �- .� ,4\ ' '-rι-' u . . �. -.' f. -� � Ί._ι. � " -- -� - · \_ι �� /j' �\ � ·ι 1

�-- - -�co� � -Ji -��- . . �- �� o � o• · � c--� -� ---

Σύστημα δύο γραμμικών εξισώσεων ε δύο α

Ε. Ευσταθίου - Γ. Βλαχούτσικος

Α ν θεωρήσουμε δύο γραμμικές εξισώσεις με δύο αγνώστους (αυτές παριστάνουν δύο ευθείες (ε ι ) και ( ε2 ) ). Λέμε τότε ότι έχουμε ένα σύστημα δύο γραμμικών εξισώσεων. Το (Σ) : ι 1 -=._ ι . {α χ + β Υ - γ

α2 χ + β2y - γ 2 Επίλυση του (Σ) είναι η διαδικασία εύρεσης ( εφ ' όσον υπάρχουν) των κοινών λύσεων ( χ0 ,y 0 ) των

εξισώσεων και αν υπάρχουν λέμε ότι το (Σ) είναι Συμβιβαστό.

Γραφική επίλυση Αρχικά παριστάνουμε τις ευθείες του συστήματος σε ένα καρτεσιανό σύστημα συντεταγμένων τότε :

Αν οι ευθείες τέμνονται σε ένα σημείο λέμε ότι οι συντεταγμένες ( χ0 ,y0 ) του κοινού σημείου είναι μια λύση του συστήματος.

Αν οι ευθείες είναι παράλληλες το σύστημα είναι αδύνατο, δηλαδή δεν έχει λύση . li Αν οι ευθείες ταυτίζονται το σύστημα έχει άπειρες λύσεις, δηλαδή άπειρα ζεύγη ( χ 0 ,y0 ) που επαληθεύουν τις δύο εξισώσεις.

�- :• Αλγεβρικές μέθοδοι επίλυσης α) Μέθοδος αντικατάστασης

Αρχικά λύνουμε τη μια εξίσωση ως προς τον έναν άγνωστο . " Αντικαθιστούμε τον άγνωστο αυτό στην άλλη εξίσωση οπότε προκύπτει εξίσωση με έναν

άγνωστο τον οποίο και υπολογίζουμε. Κατόπιν τον άγνωστο αυτό τον aντικαθιστούμε στην πρώτη εξίσωση και έτσι βρίσκουμε και τον άλλον άγνωστο.

- τ _ - I · ' � I ,\. j ι: {x - 2y = 1 0 Να λυθεί το σύστημα : (Σ) : . 2x + 3y = -29

Σ <=> <=> <=> <=> <=> χ = -4 -7 . {x = l 0 + 2y {x = l 0 + 2y {x = l 0 + 2y {x = l 0 - 1 4 ( ) 2 ( 1 0 + 2y ) + 3y = -29 7y + 20 = -29 7y = -49 y = -7

( , y) ( '

)

μ :: . : (;( : / 11 ' ' ' 1 ' · ' • ' 1 ( Βρίσκουμε τον ίδιο άγνωστο και από τις δύο εξισώσεις και εξισώνουμε τα αποτελέσματα. {3χ - 2y = 5 Να λυθεί το σύστημα: (Σ) : -4χ + 3y = -7

' ' \. ι, } : :-p -; - { 3χ - 5 { 3χ -5

{ y = -- y = -- 3χ - 5 3χ - 5 3 · 1 - 5

(Σ) <=> 2 <=> 2 <=> {y =-2

-<=> {y =

-2-

<=> y = -

2-

<=> (x , y ) = ( 1 , - 1 ) 4χ - 7 3χ - 5 4χ - 7 y = -- -- = -- 9χ - 1 5 = 8χ - 14 χ = 1 χ = 1

3 2 3 ΕΥΚΛΕΙΔΗΣ Β' 75 τ.3/18

Page 22: Ευκλειδης Β 75

Μαθηματικά για την Α ' Λυκείου

' I] �. ι i '�";:τ i-.'[ ) j- : " ( ιί t _(ι λ/ :(-;ϊ_ ) -Ιi'Τ� :λ�- ; (;-(;ί ;·j) \' Αρχικά πολλαπλασιάζουμε και τα δύο μέλη των εξισώσεων με κατάλληλους αριθμούς ώστε να

προκύψουν στις εξισώσεις αντίθετοι συντελεστές σε ένα άγνωστο . Θα έχουν απόλυτη τιμή το Ε.Κ.Π. αυτών των συντελεστών.

Κατόπιν προσθέτουμε κατά μέλη τις δύο εξισώσεις οπότε προκύπτει νέα εξίσωση με ένα άγνωστο, τον οποίο και υπολογίζουμε.

Την τιμή αυτού του αγνώστου την aντικαθιστούμε σε μια από τις αρχικές εξισώσεις και έτσι υπολογίζουμε τον άλλο άγνωστο . { 2x - y x + y

_

3 Να λυθεί το σύστημα (Σ) : _2

_ _ _ 4

_ _

χ - 2 l - 4y -- + -- = 1 3 6

(Σ) <=> <=> <=> <=> <=> {2 ( 2x - y) - ( x + y) = 1 2 {3x - 3y = l 2 {x - y = 4 2 {2x - 2y = 8 {2y = - 1 2 ( x - 2 ) + ( 1 - 4y) = 6 2x - 4y = 9 2x - 4y = 9 (- 1 ) -2x + 4y = -9 x - y = 4

� (: :(!υ = 4 � {: : :�Ξ � {: : iΞ � (x, y) = (� · -H

δ) Μέθοδος Οριζουσών, Γραμμικό σύστημα 2χ2 ,. Ορίζουσα 2 ης τάξης I ; �ι = αδ - βγ

{αι χ + β ι y = γ ι Επίλυση Γραμμικού Συστήματος 2χ2 (Σ) : _ α2χ + β2y - γ2

Αρχικά υπολογίζουμε τις ορίζουσες : Ο = Ι α, βι I = α1β 2 - α2β 1 , α2 β2 Ι γ ι β ι l l αι γ ι l Οχ = β

= γι β2 - γ2β ι , Οy = = αι γ2 - α2γ ι γ 2 2 α2 γ 2

! .ί Αν Ο * Ο το (Σ) έχει μοναδική λύση την (x , y) = ( � , �) 2 Αν Ο = Ο και Ο, * Ο ή ΟΥ * Ο το σύστημα είναι αδύνατο . .) \ Αν Ο = Ο και Ο , = ΟΥ = Ο ελέγχω το αρχικό σύστημα.

Α ν α1 * Ο ή α2 * Ο ή β 1 * Ο ή β2 * Ο , τότε το σύστημα είναι αόριστο . Αν α1 = α2 = β 1 = β2 = γ1 = γ2 = Ο , τότε το σύστημα είναι ταυτοτικό . Αν α1 = α2 = β 1 = β2 = Ο και γ 1 * Ο ή γ2 * Ο, τότε το σύστημα είναι αδύνατο .

. � Ομογενές Σύστημα 2χ2 (Σ)

: {αχ + βy =

0 και Ο = I

α αι χ + β ι y = Ο αι

_) Το ομογενές σύστημα έχει πάντοτε λύση την (x,y)=(O,O) και ελέγχουμε αν έχει και άλλες λύσεις .

� ll <U OJ ![ (j Αν Ο * Ο το (Σ) έχει μόνο την μηδενική λύση (x,y)=(O,O) .

Αν Ο = Ο το (Σ) έχει άπειρες λύσεις διαφορετικές από την μηδενική (Αόρισρο).

ΕΥΚΛΕΙΔΗΣ Β' 75 τ.3/19

Page 23: Ευκλειδης Β 75

Α l: Κ Η Σ Η {λχ - y = 2λ - 1 Να λύσετε το σύστημα χ + λy = 1 + λ Λύση Αρχικά υπολογίζουμε τις ορίζουσες D, D, , DY D = lλ - 1 Ι = λ2 + 1 , D, = 12λ - 1 - 1 1 = λ(2λ - 1) + 1 + λ άρα D, = 2λ2 - λ + 1 + λ = 2λ2 + 1 και 1 λ ' 1 + λ λ Ι λ 2λ - 1 1 ? ? , , , , Dv = = λ(l + λ) - 1 (2λ - 1 ) = λ + λ- - 2λ + 1 = λ- - λ + 1 . Παρατηρουμε οτι για καθε λ Ε R , ειναι . 1 1 + λ D ο Ε , (Σ) , δ , λ , D 2λ2 + 1 λ2 - λ + 1 =F- • πομενως το εχει μονα ικη υση χ = -' = -7 - και y = --=-7 --D λ- + 1 λ" + 1

Α Σ Κ Η Σ Η

Δίνονται ο ι ευθείες (ε 1 ) : 5χ + λy = 2λ - 1 και (ε2 ) : y = 3 - λχ . Να προσδιορίσετε τις τιμές του λ Ε R ώστε να τέμνονται. ΛίJση {λχ + y = 3 Οι εξισώσεις των ευθειών ορίζουν το σύστημα (Σ) : 5χ + λy = 2λ - 1 Για να τέμνονται οι ευθείες θα πρέπει και αρκεί το (Σ) να έχει μία λύση . Δηλαδή πρέπει και αρκεί D =F- Ο όπου : D = � � �� = ( λ2 - 5 ) , δηλαδή λ =F- .J5 και λ * -.JS . Άρα οι ευθείες τέμνονται μόνον όταν λ � { .J5, -.JS} . ΑΣ Κ Η Σ Η

Δίνεται το γραμμικό σύστημα 2χ2 με αγνώστους x,y αν για τις ορίζουσες Ο, Ο, , ΟΥ ισχύει 40,2 + 40Υ 2 + 902 + 40, - 120Υ - 60 s -1 1 (I) Να βρείτε τη λύση του. ΛίJση

'Εχουμε : (1)� (2D, )2 + 2 · 2D, + 1 - 1 + (2DY )2 - 2 · 2DY · 3 + 32 - 32 + (3D)2 - 2 · 3D · 1 + 1 2 - 1 1 ::::; - 1 1 � ( 2D, + 1 )2 + ( 2DY - 3 )2 + (3D - 1/ - 1 1 ::::; - 1 1 � ( 2D, + 1 )2 + ( 2D), - 3 )2 + (3D - 1 ) 2 ::::; Ο (II) Επειδή το πρώτο μέλος της (Π) είναι άθροισμα μη αρνητικών αριθμών θα ισχύει: ( 2D, + 1)2 = ( 2DY -3)2 = (3D-1)2 =0 Επομένως 2D, + 1 = Ο και 2D), - 3 = Ο και 3D - 1 = Ο � D = _..!._ και D = � και D = .!. έτσι ' 2 Υ 2 3

1 D, -2 3 Χο =D = -1- = -2 ,

3 ΛΣ Κ Η Σ Η

3 Dy 2 Υο =-=-= -3 D 1 2

α) Αν για τους πραγματικούς αριθμούς κ,λ,μ ισχύει: lκ-�2 =lκ-�2 �λ-�2 • Να δείξετε ότι λ=μ ή κ=μ. β) Αν ένα γραμμικό σύστημα 2χ2 έχει μοναδική λύση (x0 , y0 ) η οποία πληρεί την 3χ0 + 2y0 = 4 και για τις ορίζουσες Ο, Ο, , Ο Υ ισχύει η ισότητα ID-0.12 = ID-1:\ 12 �η -1:\ 1 2 (1). Ν α βρείτε την λύση ( χ0 , y 0 ) . Λύση

α ) Από την ισότητα lκ - λ12 = lκ - μl2 + lλ - μl 2 προκύπτει ( κ - λ)2 = ( κ - μ)2 + (λ - μ/ ΕΥΚΛΕΙΔΗΣ Β' 75 τ.3/20

Page 24: Ευκλειδης Β 75

Μ αθηματικά για την Α ' Λυκείου

� κ2 + λ2 - 2κλ = κ2 + μ2 - 2κμ + λ2 + μ2 - 2λμ � 2μ2 - 2κμ - 2λμ + 2κλ = Ο μ2 - κμ - λμ + κλ = Ο ή μ(μ - κ) - λ(μ - κ) = Ο � (μ - κ)(μ - λ) = Ο � μ=κ ή μ=λ β) Επειδή το γραμμικό σύστημα έχει μοναδική λύση θα ισχύει D :;t Ο επίσης από την (Ι) και το (α) ερώτημα προκύπτουν : D = DY ή D, = DY . Επομένως :

• Με D = DY έχουμε y0 = DY = 1 οπότε: 3χ0 + 2y0 = 4 � 3χ0 = 2 � χ0 = � . Άρα ( x0 , y0 ) = (� , 1) D 3 3 • Με D, = Dy και D :;t: O έχουμε x0 = y0 οπότε: 3χ0 +2y0 =4�5χ0 = 4�χ0 =� . Άρα ( x0 , y0 ) = (i ,i) 5 5 5

Τ Ρ Ι Ω Ν Υ Μ Ο ΔΕΥΤΕ ΡΟΥ ΒΑΘΜΟΥ Ορισμός : Τριώνυμο δευτέρου βαθμού μιας μεταβλητής χ λέγεται κάθε πολυώνυμο δευτέρου βαθμού που έχει μορφή : f(χ) = αχ 2 + βχ + γ, α, β , γ Ε R , α :;t: Ο Ορισμός : Π ολυωνυμική συνάρτηση δευτέρου βαθμού είναι η συνάρτηση με τύπο f(x) = αχ 2 + βχ + γ, α, β, γ Ε R, α :;t Ο και πεδίο ορισμού το R. Ορισμός : Ρίζα τριωνύμου f(x) = αχ 2 + β χ + γ, λέγεται κάθε ρίζα της εξίσωσης αχ 2 + β χ + γ = Ο . Ορισμός : Δια κρ ίνουσα ενός τριωνύμου ονομάζουμε την παράσταση Δ = β2 - 4αγ . Το Ε Ι ΔΟΣ των ριζών ενός τριωνύμου εξαρτάται από το πρόσημο της διακρίνουσας , (αντίστοιχα θέματα έχουμε μελετήσει στο είδος ριζών της αχ 2 + βχ + γ = Ο, α, β, γ Ε R, α :;t Ο )

I . Μ ο ρφές τρ ιωνύμου

Παρατηρούμε ότι : f(x) = ax2 + βχ + γ τότε 4αf(χ) = 4ax2 + 4aβχ + β2 - β2 + 4αγ � 4αf(χ) = (2αχ + β)2 - (β2 - 4αγ) � 4αf(χ) = 4α2 (χ + 1._)2 - Δ 2α � 4αf(χ) = 4α2 [ cx + 1.._)2 -�] � f(x) = α [(χ + 1.._)2 -�] ( Ι ) 2α 4α 2α 4α-Από την μελέτη της ( 1 ) προκύπτουν τα επόμενα:

α. Αν Δ<Ο το τριώνυμο γράφεται : f(x) = α [ (χ + t/ - [ �)' ] Στην περίπτωση αυτή το τριώνυμο έχει μορφή γινομένου του α επί ένα άθροισμα δύο τετραγώνων. Έτσι όταν Δ<Ο το Τριώνυμο δεν μηδενίζεται στο R και δεν αναλύεται σε γινόμενο παραγόντων πρώτου βαθμού. β. Αν Δ=Ο το Τριώνυμο γράφεται : f(χ) = α(χ + 1._)2 ή f(χ) = α(χ - ρ)2 αφού ρ 1 = ρο = -1._ = ρ . 2α - 2α Στην περίπτωση αυτή το τριώνυμο έχει Μορφή γινομένου του α επί ένα τετράγωνο πρωτοβάθμιου ως προς χ Διωνύμου.

γ. Αν Δ>Ο το Τριώνυμο γράφεται : f(x) = α [ (χ + :α )' - [:) ' ] = α [[χ + 1._ + JΔ ) [x + l_ - JΔ )] τότε f(χ) = a(χ - ρ 1 )(χ - ρ2 ) , αφού ρ1 2 =

-β ± JΔ . 2α 2α 2α 2α · 2α Στην περίπτωση αυτή το τριώνυμο έχει Μορφή γινομένου του α επί μία διαφορά δύο τετραγώνων. Έτσι όταν Δ>Ο το τριώνυμο αναλύεται σε γινόμενο παραγόντων. 1 1 . Π ρόση μο των τιμών της συνάρτησης : f(x) = αχ 2 + βχ + γ, α, β, γ Ε R , α :;t Ο Θα μελετήσουμε το πρόσημο του f(x) για τις διάφορες τιμές του χ Ε R .Το πρόσημο της Διακρίνουσας επηρεάζει το πρόσημο του γινομένου af(x) όταν χ Ε R .

ΕΥΚΛΕΙΔΗΣ Β' 75 τ.3/21

Page 25: Ευκλειδης Β 75

Μαθη ματικά για την Α ' Λυκείου

Έτσι γνωρίζοντας τις Μορφές του Τριωνύμου διακρίνουμε τις παρακάτω περιπτώσεις :

α ) Αν Δ<Ο προκύπτει : af(x) = a' [ (χ + �)' + �:, ] > Ο για κάθε χ ε R , αφού -Δ>Ο.

Στην περίπτωση αυτή το τριώνυμο για κάθε χ Ε R έχει ίδιο πρόσημο με το α.

11 J Αν Δ=Ο προκύπτει : af(x) = a2 (χ +�)2 > Ο για κάθε χ Ε R - {-:α } .

Στην περίπτωση αυτή το τριώνυμο για κάθε χ Ε R έχει ίδιο πρόσημο με το α, εκτός από την τιμή χ = _l_ (Διπλή ρίζα ) που το μηδενίζει. 2α

Αν Δ>Ο προκύπτει : af(x) = a2 (x - ρ 1 )(χ - ρ2 ) με ρ1 < ρ2 . Στην περίπτωση αυτή διαπιστώνουμε ότι : � ) Αν χ < ρ 1 ή χ > ρ2 .Δηλαδή αν χ Ε (-οο, ρ 1 ) υ (ρυ+οο) έχουμε (χ - ρ 1 )(χ - ρ2 ) > 0 δηλαδή : af(x) > O που σημαίνει ότι το τριώνυμο f(x) έχει ίδιο πρόσημο με το α όταν ο χ παίρνει τιμές μικρότερες της μικρότερης ρίζας ή μεγαλύτερες της μεγαλύτερης ρίζας. u i ) Αν ρ ι < χ < ρ2 δηλαδή αν χ Ε (ρ ι , ρ2 ) τότε (χ - ρ 1 )(χ - ρ2 ) < Ο δηλαδή af(x) < Ο που σημαίνει ότι το τριώνυμο f(x) έχει αντίθετο πρόσημο του α όταν ο χ παίρνει τιμές ανάμεσα στις ρίζες. Εποπτικά : :\ρ •�. σψυ l:υ ιιRπψι!σμυτυ

<[--x__ll__ ___ __ ---tl---------i . ---- -.>

f(x) ομόσημο του d�- ετερόσημο του α 1'Γομόσημο του α

Ι ) Η συνθήκη Δ <Ο εξασφαλίζει ότι για κάθε χ Ε R ισχύει: f (χ) =F Ο 2) Η συνθήκη Δ<Ο εξασφαλίζει ότι για κάθε χ Ε R το τριώνυμο έχει σταθερό πρόσημο. Έτσι: Γίνεται θετικό όταν α>Ο και αρνητικό όταν α<ο.

Δίνεται το τριώνυμο φ ( χ) = (3 - λ) χ2 + 2 ( 1 + λ) χ + λ2 + λ + 2 αν για κάθε λ =F 3 ισχύει φ ( χ) > Ο . Να μελετήσετε την μονοτονία της συνάρτησης f ( χ) = ( 1 - λ) χ + 7 .

Επειδή για κάθε λ Ε R - {3} ισχύει φ ( χ ) > Ο θα είναι: 3 - λ > Ο ( l ) και Δ < Ο (2), 'Εχουμε: ( l ) :::::> λ < 3 . Όμως Δ = [2 ( Ι + λ )Τ - 4 · (3 - λ) (λ2 + λ + 2) = 4 (Λ: - 3λ2 + 7λ - 5 ) (1) . Έτσι (2) :::::> Λ: - 3λ2 + 7λ - 5 < Ο :::::> Λ: - 2λ2 - λ2 + 5λ + 2λ - 5 < Ο :::::> λ2 (λ - Ι ) - 2λ ( λ - Ι ) + 5 (λ - Ι ) < ο :::::> (λ - Ι ) ( λ2 - 2λ + 5 ) < ο :::::> λ - ι < ο :::::> λ < ι ( αφού το λ2 - 2λ + 5 έχει Δ 1 = - ι 6 < Ο , επομένως για κάθε λ Ε R ισχύει 'Κ - 2λ + 5 > Ο ) Άρα: ( ι ) ,(2) :::::> λ < Ι , οπότε η συνάρτηση f ( χ ) = ( Ι - λ) χ + 7 είναι γνησίως αύξουσα.

Αρχικά παραγοντοποιούμε το Ρ ( χ ) , έτσι Ρ ( χ ) = Α (χ ) · Β ( χ ) · Γ ( χ ) · . . . · Φ ( χ ) κατόπιν προσδιορίζουμε το πρόσημο κάθε παράγοντα ξεχωριστά και τέλος το πρόσημο του Ρ ( χ ) . Με όμοιο τρόπο λύνουμε την aνίσωση Α ( χ ) · Β ( χ ) · Γ ( χ ) · . . . · Φ ( χ ) ;:::: Ο ή Α ( χ ) · Β( χ ) · Γ ( χ ) · . . . · Φ ( χ ) � Ο .

Να προσδιορίσετε το πρόσημο της παράστασης: Ρ( χ) = χ4 ( χ2 - 2χ + 1 ) ( χ2 - 3 ) ( χ2 - χ + 1 ) ( χ2 - 5χ + 6) .

Θα προσδιορίσουμε το πρόσημο κάθε παράγοντα του Ρ ( χ ) . χ4 > Ο <=> χ Ε JR * .

ΕΥΚΛΕΙΔΗΣ Β' 75 τ.3/22

Page 26: Ευκλειδης Β 75

Μ αθηματικά για την Α ' Λυκείου

χ 2 - 2 χ + 1 > ο <=> (χ - Ι ) 2 > ο <=> χ * 1 . τ ο χ 2 - 3 > ο <=> (χ + .J3) ( χ - .J3) > ο <=> χ < -.J3 ή χ > .J3 : Το χ 2 - χ + 1 έχει διακρίνουσα Δ = -3 < Ο , άρα χ 2 - χ + Ι > Ο για κάθε χ ε IR . Το χ2 - 5χ + 6 έχει διακρίνουσα Δ = 1 > Ο και ρίζες ρ 1 = 3 , ρ2 = 2 . Έτσι έχουμε : χ2 - 5χ + 6 > 0<::::> χ < 2 ή χ > 3 . τέλος για να βρούμε το πρόσημο του Ρ ( χ ) κατασκευάζουμε τον πίνακα :

χ -CIJ -.J3 ο 2 3 +α:> -_�

-_- -_ �-4 �� -��=� � ___

+ __ ---=----r _ __ill_ + ι __

+ __

--+ __

+ __ L_±_�� -· __ +

__ __ J χ 2 - 2χ + 1 _____

+ _____

+ __ +--__

+ __ -'fCD"----_+_--+- - - _ _ +__ il +

+ I +

+ . --1

ι χ_2 -_

_ 3 + - - - φ + �-x

__

+ __

I _ _ _ __ __

+ __ _:: __

+--f---·-

+---- --

+--- -�--

+--: + :;. -

-ι j-

___

_ x' _

___

-�-�-: )_+ -6

--:----�--)_-_-__ -:_-_-_c-:_D-=_-=_-:_-_-__Jc+-�----_:-=_-=_-:_φ-=_-=_-:_.......1. ��--= -�L_�_:_

---1�,

Α(χ) , Α (χ) ι;. ) Ανισώσεις της μορφής : Β (χ) � Ο (1) η Β (χ) � Ο (2).

Με Β ( χ ) 7= 0 έχουμε : ( l ) <=> A (x ) · B ( x ) � O <::::> . . . (2) <=> Α (χ ) · Β ( χ ) � Ο <::::> . . .

!) \ Κλασματικές Ανισώσεις της μορφής: ��:� � Γ ( χ) ( 1 ) ή ��:� � Γ( χ) (2).

Απαλοιφή παρανομαστών κάνουμε μόνο αν γνωρίζουμε το πρόσημο της παράστασης Β ( χ ) 7: Ο . ' Αν δεν γνωρίζουμε το πρόσημο της παράστασης Β ( χ ) μεταφέρουμε όλους τους όρους της ανίσωσης

στο I ο μέλος, οπότε καταλήγουμε σε ανίσωση της μορφής : � �: � � Ο η � �: � � Ο . /\ :t κ �-α: ι-ι Ν λ θ , , χ - 3 χ + 2 1 Ο (I) α υ ει η ανισωση : -- - -- � ---=-2 --χ + 2 χ - 3 χ - χ - 6

Η · 'ζ · 2 3 τ · (Ι) χ - 3 χ + 2 1 0 ο ανισωση ορι εται μονο για χ 7: - και χ :;t . οτε: <::::>

----- - 2 � χ + 2 χ - 3 χ - χ - 6

( χ - 3 )2 - (χ + 2 )2 - \ Ο χ 2 - 6χ + 9 - χ2 - 4 - 4χ - 1 0χ + 5 <::::> > Ο <::> > Ο <::> > Ο ( χ + 2) ( χ - 3) - ( χ + 2) ( χ - 3) - ( χ + 2) { χ - 3) -

<=> ( - 1 Οχ + 5 ) ( χ + 2 ) ( χ - 3) � Ο (1 1) . Κατασκευάζουμε τον πίνακα προσήμων :

χ -00 -2 Ι/2 3

----

- ---

-- - -- --

- - I I - \ Οχ + 5 I + + CD ' -------i ' I�

χ + 2 - + + t-

+ I

χ - 3 - - - + Ρ ( χ ) + - () + I -' -�- -

- --- ------------- -- -

Άρα: ( Ι ) <::::> χ ε (-οο, -2 ) u [� , 3) .

ΕΥΚΛΕΙΔΗΣ Β' 75 τ.3/23

Page 27: Ευκλειδης Β 75

Μαθηματικά για την Α ' Λυκείου

ΛΣ Κ Η Σ Η

Να μετατρέψετε το τριώνυμο 4ου βαθμού : φ ( χ) = 4a2 x4 - (β2 + 4α2γ 2 )χ2 + β2γ2 , α , β , γ Ε R,α :;e Ο σε γινόμενο πρωτοβαθμίων παραγόντων ως προς χ. Λίιση Αν θέσουμε χ 2 = y 2 0 παίρνουμε το τριώνυμο : φ(y) = 4a2y2 - (β 2 + 4α2γ2 )y + β2γ2 που έχει Διακρίνουσα : Δ = (β2 + 4α2γ2 )2 _ 4 . 4αzβ2γ2 = β4 + 1 6α4γ4 + Sα2β2γ2 _ 1 6α2β2γ2 = β4 + 1 6α4γ4 _ 8α2β2γ2 = (β2 _4α2γ2 )2 2 0

, β2 + 4αzγz ± �(β2 _ 4αzγz )2 βz + 4α2γ2 ± (βz _ 4α2γ2 ) Τοτε Υ ι 2 = 2 ' · 8α 8α-

Δηλαδή Υ ι = β22 , y2 = γ2 άρα φ(y) = 4α2 (y - β2, )(y - γ2 ) = (4α2y - β2 ) ( y - γ2 ) έτσι 4α 4α-

φ(χ) = (4αz χ 2 - βz ) (x z - γ2 ) ή φ(χ) = (2aχ + β)(2aχ - β)(χ + γ)(χ - γ) .

Λ Σ Κ Η Σ Ι-1 Να δείξετε ότι η εξίσωση : χ2 - 2(λ - l)x + λ - 3 = Ο (1) για κάθε λ Ε R έχει ρίζες πραγματικές και άνισες. Κατόπιν να προσδιορίσετε τις τιμές του λ Ε R ώστε η εξίσωση να έχει ρίζες ομόσημες. Λίιση α) Υπολογίζουμε την διακρίνουσα : Δ = 4(λ - 1)2 - 4(λ - 3) = 4 [ λ2 + 1 - 2λ - λ + 3 J = 4(λ2 - 3λ + 4) ,το πρόσημο της Δ προσδιορίζεται από το πρόσημο του τριωνύμου λc - 3λ + 4 που έχει Δ 1 = 9 - 1 6 = -7 < Ο και α = 1 > Ο . Άρα για κάθε λ Ε R , λ2 - 3λ + 4 > Ο οπότε Δ > Ο για κάθε λ Ε R που σημαίνει ότι η ( I ) έχει ρίζες ρ 1 , ρ2 πραγματικές και άνισες. β) Άρα: ρ 1 , ρ 2 ομόσημες <::::> Ρ > Ο <::::> λ - 3 > 0 <::::> λ > Ι <::::> λ > 3

ι\Σ Κ Η Σ Η Να βρείτε τις τιμές του a Ε R ώστε το τριώνυμο: f(x) = (3 - α)χ2 - (7 - 4α)χ + 2 + α να γίνεται θετικό . . \ίιση Έχουμε Δ = [ -(7 - 4α)]2 - 4(3 - α)(2 + α) = 49 + 1 6α2 - 56α - 4(6 + 3α - 2α - α2 ) = 49 + 1 6α2 - 56α - 24 - 1 2α + 8α + 4α2 = 20α2 - 60α + 25 Για να γίνεται το τριώνυμο f(x) > Ο για κάθε χ Ε R πρέπει και αρκεί 3 - α > Ο ( 1 ) και Δ < Ο (2) . Έτσι: ( 1 ) <::::> α < 3 και (2) <::::> 20α2 - 60α + 25 < 0 <::::> 4α2 - 1 2α + 5 < 0 , (3) .

Αλλά: Δ 1 = (- 1 2)2 - 80 = 1 44 - 80 = 64 , α1 , =1 2 ± 8 τότε α1 = 2 , α2 =

_!_ - 8 2 2

Άρα: (3) <::::> 4(α - 2)(α - _!_) < 0 <::::> α Ε (_!_ ,2J . Τελικά: ( 1 ) , (2) <::::> α E (_!_ ,2J n (-oo, 3) <::::> a E (_!_ ,2J . 2 2 2 2 2 2 2 2

Λ Σ Κ Η l: Η

Δίνεται το τριώνυμο f(x) = 3χ2 - (2λ + 3)χ + λ, λ Ε R (Ι) Να βρείτε τις τιμές του λ Ε R ώστε η παράσταση ρ; + ρ; να παίρνει την ελάχιστη τιμή της . Λίιση Για να έχει η (Ι) ρίζες πρέπει και αρκεί Δ 2 Ο , όπου Δ η διακρίνουσα της ' ' 2λ + 3 λ 2 2 ' ( )2 ( 2λ + 3 J 2 λ Τοτε εχουμε ρ 1 + ρ2 = -3- , ρ1 · ρ 2 = 3 . Αν y = ρ 1 + ρ2 τοτε y = ρ 1 + ρ2 - 2ρ 1 · ρ2 = -3-

- 23

ΕΥΚΛΕΙΔΗΣ Β' 75 τ.3/24

Page 28: Ευκλειδης Β 75

Μαθηματικά για την Α ' Λυκείου

= 4λ2 +: + 1 2λ _

23λ =

4λ 2 + 9 : 1 2λ - 6λ , δηλαδή : 4λ2 + 6λ + 9 _ 9y = 0 ( 1 ) .

Για να έχει η (I) ρίζα ως προς λ Ε R πρέπει και εκεί Δ ι � Ο (2) .

'Ε χουμε : (2) <=> 62 - 4 · 4(9 - 9y) � Ο <=> 4y - 3 ;:::: Ο <=> y � � . 4

Επομένως η ελάχιστη τιμή της παράστασης είναι y = � . 4

Για y = � είναι Δ ι = Ο , οπότε από την ( Ι ) έχουμε λ= - __i_ = -� . 4 2 · 4 4

Για λ = -� < Ο έχουμε αγ=3λ <0, οπότε Δ=β2 -4αγ>Ο. Άρα η τιμή λ = -� είναι δεκτή και είναι η τιμή 4 4

για την οποία ελαχιστοποιείται το ρ � + ρ � .

Ά Σ Κ Η Σ Η

Να προσδιορίσετε τις τιμές του κ Ε R ώστε η εξίσωση : (κ - 3)χ2 + (κ - l )x + (κ + 2 ) = Ο να έχει δύο ρίζες αρνητικές. ΛίJ ση Για να έχει δύο ρίζες αρνητικές πρέπει και αρκεί: α.:Ο, Δ ;:::: Ο, Ρ > Ο, S < Ο . Έχουμε: α.:Ο<:::>κ;z:3 . Τότε : Δ � Ο <=> (κ - 1)2 - 4(κ - 3)(κ + 2) ;:::: 0 <=> -3κ2 + 2κ + 25 ;:::: Ο

3 2 2 ? - 0 [ I - 2Ji9 1 + 2Μ ] <:::> Κ - Κ - _ ) :ς <:::> Κ Ε , . 3 3

κ + ? Ρ > Ο <=> --- > Ο <=> (κ + 2)(κ - 3) > Ο <=> κ Ε (-oo, -2) u (3, +oo) .

κ - 3 κ - ]

S < Ο <:::> --- < 0 <:::> (1 - κ)(κ - 3) < 0 <=> (κ - 1 ) ( κ - 3) > 0 <=> K E (-oo, I) u (3, +oo) . κ - 3

Τ ο σύνολο τιμών του κ είναι η τομή των συνόλων λύσεων των τεσσάρων ανισώσεων.

Ά . [ I - 2M _2J

(3 1 + 2Ji9 ] ρα. Κ Ε , υ , .

3 3

ΆΣ Κ Η Σ Η

Ν α βρείτε τα χ Ε R ώστε να ορίζεται το κλάσμα y = 2 χ

χ - 5χ+ 9 Κατόπιν γι' αυτούς τους πραγματικούς αριθμούς να δείξετε ότι το κλάσμα παίρνει τιμές στο διάστημα [- 1\ , 1] .

ΛίJ ση Το τριώνυμο g(x) = χ2 - 5χ + 9 έχει Δ = - 1 1 < Ο άρα για κάθε χ Ε R ισχύει g(x) > Ο . Δηλαδή το κλάσμα ορίζεται για κάθε χ Ε R .

Τότε y = ? χ

<=> yx 2 - 5yx + 9y - x = 0 <=> yx2 - (5y + 1)x + 9y = O ( 1 ) x - - Sx + 9

Αν y = Ο τότε: ( 1 ) <=> Οχ 2 - χ = Ο <=> χ = Ο . Άρα η y = Ο είναι μία τιμή του κλάσματος.

Αν y ,: Ο , επειδή χ Ε R πρέπει Δ ;:::: Ο . Όμως Δ � Ο � [ -(5y + 1) ]2 - 4y9y � Ο

� 25y2 + 1 + l Oy - 36y2 ;:::: Ο � 1 1/ - 1 Oy - 1 ::; Ο (2) . Το τριώνυμο φ(y) του πρώτου μέλους της (2) έχει

διακρίνουσα: Δ = ( - 1 0)2 - 4 · I I · ( - 1 ) = 1 44 > Ο και ρίζες Υ ι = 1, y2 = _ __!_ . 'Ετσι: 1 1

(2 ) <=> _ __!_ ::; y ::; 1 . Άρα το κλάσμα παίρνει τιμές στο διάστημα [-_!_ , Ι] , αφού Ο Ε [-_!_, ι ] . 1 1 1 1 1 1

ΕΥΚΛΕΙΔΗΣ Β' 75 τ.3/25

Page 29: Ευκλειδης Β 75

Μ αθη ματικά για την Α ' Λυκείου

Γ � · � . :- i:ι \, f 1 1 •· ' ;J , · ; i ι ' - · ' ' ' ι ' ' • '

Σπύρος Καρδαμίτσης - Σωτήρης Ε. Λουρίδας

« . . . Γραμμή δε μήκος απλατές. Επιφάνεια εστίν, ό μήκος και πλάτος μόνον έχει. Σχήμα εστίν το υπό τινος ή τινών όρων περιεχόμενον . . . »

Δίνεται κύκλος κέντρου Ο. Έστω ΑΒ χορδή του κύκλου αυτού και Ι το μέσο της . Θεωρούμε Γ Δ, ΕΖ τυχαίες χορδές του κύκλου αυτού που διέρχονται από το Ι . Α ν Μ, Ν είναι τα σημεία τομής των ΓΖ, ΕΔ με την ΑΒ αντίστοιχα, να αποδειχθεί ότι lM = ΙΝ

Λ Λ Λ Λ

Είναι Ζ = Δ και Γ = Ε ως εγγεγραμμένες σε ίσα τόξα γωνίες, συνεπώς τα τρίγωνα ΖΓΙ και ΙΕΔ

, , , ΓΖ ΕΔ ( I) ειναι ομοια αρα - = -ΓΙ ΕΙ

Φέρνουμε τα aποστήματα ΟΚ και ΟΛ των χορδών ΖΓ και ΔΕ, τότε είναι :

ΓΖ ΕΔ ΓΚ = - και ΕΛ = - (2) 2 2

Τ α τρίγωνα Κ Γ Ι και ΙΕΛ έχουν τις γωνίες Γ και Ε ίσες και από τις σχέσεις ( 1 ) και (2) έχουμε ότι:

ΓΖ ΕΔ ΓΚ ΕΛ - = - :::? - = -ΓΙ ΕΙ ΓΙ ΕΙ

άρα είναι όμοια οπότε ω = φ. Από τα εγγράψιμα τετράπλευρα ΚΟΙΜ και ΟΛΝΙ

Λ Λ Λ

έχουμε ω = ωι και φ = Ψι Λ Λ

επομένως είναι: ω1 = φ1 , οπότε τα τρίγωνα ΟΙΜ και ΟΙΝ είναι ίσα, άρα ΜΙ = ΙΝ.

(Ευκλείδης 365 π.Χ.) του κύκλου με τις πλευρές ΒΓ, ΑΓ και ΑΒ αντίστοιχα και Ρ η τομή της ΙΔ με την ΕΖ, να δείξετε ότι η ΑΡ διέρχεται από το μέσο της ΒΓ

Θεωρούμε την παράλληλη ΚΛ από το σημείο Ρ στην πλευρά ΒΓ του τριγώνου (Κ σημείο της ΑΒ, Λ της ΑΓ) , επομένως αρκεί να αποδείξουμε ότι το σημείο Ρ είναι το μέσο του ΚΛ ή το τρίγωνο ΙΚΛ είναι ισοσκελές. Προς τούτο αρκεί να

Λ Λ

δείξουμε με βάση σχήμα που ακολουθεί, ότι χ = y

Το τετράπλευρο ΙΡΚΖ είναι εγράψιμο αφού Λ Λ

ΙΖΚ = ΙΡΚ = 90° . Επίσης το τετράπλευρο ΙΛΕΡ Λ

είναι εγραψιμμο αφού IRL = ΙΕΛ = 90" Λ Λ Λ Λ

Άρα χ = Χ ι και Υ = Υ ι Όμως το τρίγωνο ΙΕΖ είναι ισοσκελές οπότε

Λ Λ Λ Δ Χ ι = y 1 άρα και χ = y δηλαδή το τρίγωνο Ι Κ Λ είναι ισοσκελές.

1 Ό Λ

Δίνεται ορθογώνιο τρίγωνο ΑΒΓ ( Α =90°) και ο εγγεγραμμένος κύκλος του. Α ν Δ, Ε, Ζ είναι τα σημεία επαφής του κύκλου με τις πλευρές ΒΓ, ΑΓ και ΑΒ αντίστοιχα και ΕΡ η κάθετη στην ΖΔ, να αποδείξετε ότι: α) ΑΡΕ =45° β) Η ευθεία ΡΓ είναι μεσοκάθετος της ΔΕ γ) ΑΡΓ =90°

Δίνεται τρίγωνο ΑΒΓ και ο εγγεγραμμένος α) Τα ΑΖ και ΑΕ είναι εφαπτόμενα στο κύκλο του κύκλος ( Ι, ρ). Α ν Δ, Ε, Ζ τα σημεία επαφής

ΕΥΚΛΕΙΔΗΣ Β' 75 τ.3/26

Page 30: Ευκλειδης Β 75

Μ αθηματικά για την Α ' Λυκείου

τμήματα, συνεπώς είναι ίσα, άρα το τρίγωνο ΑΖΕ είναι ορθογώνιο ισοσκελές. Επομένως ΑΕΖ = ΕΖΑ = 45" .

Το τετράπλευρο ΑΕΡΖ είναι εγγράψιμο γιατί Λ Λ Λ Λ Λ

Α+ Ρ = 90" + 90" = 1 80° άρα ω = ΑΡΕ = ΑΖΕ = 45°

Λ

β) Είναι ΑΕΖ = Ε.2 = 45 '' (από το θεώρημα χορδής και εφαπτομέγης) . συνεπώς το τρίγωνο ΔΡΕ είναι ορθογι;η·ιο και ισοσκελές, άρα ΡΕ = Ρ Δ. Ισχύει ότι ΓΕ = Γ ..1 ( σαν εφαπτόμενα τμήματα από το σημείο Γ στο" ίδιο κύκλο), επομένως τα σημεία Ρ και Γ ισαπέι.ου" από τα άκρα του τμήματος ΕΔ, πράγμα που σημαίνει ότι η ευθεία ΡΓ είναι μεσοκάθετο; του ευθύγραμμου τμήματος ΕΔ. γ) Επειδ1Ί το τρίγωνο ΔΡΕ είναι ορθογώνιο και ισοσκελές η ΡΓ ως μεσοκάθετος είναι και

Λ Λ

διι.οτόμος της γωνίας ΕΡΔ άρα ΕΡΓ =45°

Δίνεται τρίγωνο ΑΒΓ εγγεγραμμένο σε κύκλο. Οι εφαπτόμενες του κύκλου στα σημεία Β και Γ τέμνονται στο Η. Από το σημείο Η φέρνουμε ΗΕ .l ΑΒ, ΗΖ .l ΑΓ και ΗΔ .l ΒΓ. Να αποδείξετε ότι το τετράπλευρο ΕΗΖΔ είναι παραλληλόγραμμο.

Λ Λ

Σ το τετράπλευρο ΔΓΖΗ είναι Δ+ Ζ= ιχ1' +'Χ1' = 1 80°,

Λ Λ

άρα είναι εγγράψιμο, επομένως Γ ι = Ζι . Αλλά Λ Λ

Γι = Α ( θεώρημα χορδής και εφαπτομένης) Λ Λ

επομένως Ζ ι = Α ( 1 ) Λ Λ

Στο τετράπλευρο ΑΕΗΖ είναι Ε+ Ζ 90" + 90" = 1 80", άρα είναι εγγράψιμο, οπότε

Λ Λ

ΕΗΖ+ Α = 1 80° και λόγω της ( I ) έχουμε Λ Λ

ΕΗΖ+ Ζ ι = 1 80" άρα ΔΖ // ΕΗ. Όμοια αποδεικνύεται ότι ΔΕ // ΗΖ, επομένως το τετράπλευρο ΕΗΖΔ είναι παραλληλόγραμμο.

·= i T �·) I

Λ Λ Θεωρούμε τρίγωνο ΑΒΓ ,ώστε Β- Γ = 90° . Α ν το ύψος ΑΔ του τριγώνου ΑΒΓ τέμνει στο σημείο Ε την εφαπτομένη του περιγεγραμμένου κύκλου του τριγώνου αυτού στην κορυφή Γ, να συγκριθούν τα ευθύγραμμα τμήματα ΕΑ και ΕΓ.

Προφανώς το τρίγωνο ΑΒΓ είναι αμβλυγώνιο στην κορυφή Β . Έστω Ζ σημείο της πλευράς ΑΓ τέτοιο ώστε ΒΖ.lΒΓ.

Λ Λ Λ

Παρατηρούμε ότι: ΔΑΒ = ΑΒΖ = Γ . Αυτό σημαίνει ότι η ΕΑ είναι εφαπτομένη στον κύκλο (γνωστή πρόταση ), επομένως ΕΑ ΕΓ. (Εφαπτόμενες αγόμενες από το σημείο Γ στον ίδιοι κύκλο).

Θεωρούμε κυρτή γωνία xOy και σημεία Α, Γ της πλευράς Οχ διαφορετικά της κορυφής Ο με ΟΑ < ΟΓ και Β, Δ της Oy διαφορετικά της κορυφής Ο με ΟΔ < 08. Έστω Ε το σημείο τομής των ΑΒ και Γ Δ. α) Να αποδειχθεί ότι οι περιγεγραμμένοι κύκλοι στα τρίγωνα ΕΑΓ και ΕΔΒ δεν είναι δυνατό να εφάπτονται.

ΕΥΚΛΕΙΔΗΣ Β' 75 τ.3/27

Page 31: Ευκλειδης Β 75

Μ αθη ματικά για την Α ' Λυκείου

β) Αν Μ το δεύτερο σημείο τομής τους , αποδείξατε ότι το τετράπλευρο α ΓΜΔ είναι εγγράψιμο. Λύση : α) Αν οι κύκλοι εφάπτονταν τότε στο σημείο επαφής τους Ε θα θεωρούσαμε την κοινή τους εξωτερική εφαπτομένη ΖΕΗ (Η προς το μέρος του 0), οπότε θα είχαμε ότι οι ευθείες ΑΓ και ΔΒ είναι παράλληλες (γιατί;) πράγμα άτοπο, αφού τέμνονται στο σημείο Ο.

ο ·: '

'

Η z

- - -

β) Έστω Μ το δεύτερο σημείο τομής των περιγεγραμμένων κύκλων στα τρίγωνα ΕΑΓ και ΕΔΒ. Θεωρούμε την κοινή χορδή τους ΕΜ οπότε

Λ Λ Λ παίρνουμε : ΑΓΜ = ΜΕΒ = ΜΔΒ , ισότητα που οδηγεί στο ότι το τετράπλευρο ΟΓΜΔ είναι εγγράψιμο σε κύκλο καθότι μία εσωτερική του γωνία ισούται με την απέναντι από αυτή εξωτερική του . ΣΧΟΛ Ι Ο I : Το σημείο Μ ονομάζεται σημείο του Miquel . Τελικά οι περιγεγραμμένοι κύκλοι στα

Δ Δ Δ Δ τρίγωνα Ε Α Γ, Ε Β Δ, Ο Γ Δ, Ο Α Β διέρχονται από το ίδιο σημείο . ΣΧΟΛΙ Ο 2 : Χρησιμοποιήθηκαν τα θεωρήματα i) Μία ικανή και αναγκαία συνθήκη, ώστε το

κυρτό τετράπλευρο να είναι εγγράψιμο σε κύκλο είναι μία εσωτερική του γωνία να ισούται με την απέναντι εξωτερική του.

ii) Μία ικανή και αναγκαία συνθήκη , ώστε ένα

κυρτό τετράπλευρο είναι εγγράψιμο, είναι η εξής: Δύο γωνίες που οι κορυφές τους είναι δύο

διαδοχικές κορυφές του τετραπλεύρου και σχηματίζονται από τις δύο διαγωνίους και από τις δύο απέναντι πλευρές είναι ίσες.

Άσκη ση 7'' Θεωρούμε ισοσκελές τρίγωνο ΑΒΓ

(ΑΒ=ΑΓ) και τον περιγεγραμμένο του κύκλο (0, ρ). Έστω Η σημείο της πλευράς του ΑΒ ώστε OH .l AB.

Στην προέκταση της ΟΗ παίρνουμε ΗΔ=ΟΗ. Θεωρούμε σημείο Ε της πλευράς ΒΓ, ώστε ΔΕ παράλληλη στην ΑΓ. Να αποδειχθεί

Λ ότι: ΔΟΕ = 90° Λύση :

Παρατηρούμε ότι το τετράπλευρο ΑΔΒΟ είναι ρόμβος οπότε η ΒΔ είναι ίση και παράλληλη προς την ΑΟ, γεγονός που οδηγεί και στην ισότητα

Λ ΔΒΓ = 90° ( 1 ) Θεωρούμε τώρα την διάμετρο ΒΟΖ οπότε

Λ ΖΑΒ = 90° πράγμα που σημαίνει ότι οι ΑΖ και

Λ Λ ΔΟ είναι παράλληλες και επομένως ΖΑΓ = ΟΔΕ (οξείες γωνίες με πλευρές παράλληλες)

Λ Λ Λ Λ Λ Όμως Γ ΑΖ = ΓΒΖ � ΕΔΟ = ΓΒΖ = ΕΒΟ οπότε το τετράπλευρο ΔΟΕΒ είναι εγγράψιμο σε κύκλο

Λ Λ ,άρα ΔΟΕ = ΔΒΕ = 1 80° (2).

Από τις σχέσεις ( 1 ) και (2) παίρνουμε Λ

ΔΟΕ = 90ο

Σχόλιο : Παρατηρήσαμε ότι ,από το ισοσκελές τρίγωνο ΑΒΓ έχουμε : ΑΟ κάθετη στην ΒΓ, οπότε ΔΒ κάθετη στην ΒΓ σαν παράλληλη στην ΑΟ.

ΕΥΚΛΕΙΔΗΣ Β' 75 τ.3/28

Page 32: Ευκλειδης Β 75

Μαθηματ ι κά

γ ια τη Β ' τάξ η του Λυκε ίου

Εκθετική-Λογαριθμ ική συνάρτηση

Ο Ρ Ι Σ Μ ΟΙ-ΧΡΗΣ Ι Μ ΕΣ Ε Π ΙΣΗ ΜΑΝΣ Ε Ι Σ Ι .ΟΡΙΣΜΟΣ Εκθετική συνάρτηση με βάση το θε­τικό αριθμό α, με α * 1 ονομάζουμε τη συνάρτηση f : IR � IR με τύπο f(χ)=α χ .Προφανώς έχει σύνο­λο τιμών το (Ο,+ οο ) . 2 . Η γραφική παράσταση της εκθετικής συνάρτη­σης f(x) = αχ είναι:

Ο < "· < I

-2 - 1 .5 - 1 - 0 . 5

2 . 5

1 . 5

0 . 5

υ > Ι

0 . 5 1 . 5

3.Αν α> 1 , τότε η f(x)= α χ είναι γνησίως αύξουσα, δηλαδή για χ ι < χ2 ισχύει f(x ι ) < f(x2 ) , ενώ αν Ο<α :;e 1 η f(x)= α χ είναι γνησίως φθίνουσα δηλα­δή για χ ι < χ2 ισχύει f(x ι ) > f(x 2 ) .

4.Μια εξίσωση της μορφής ( f(x) )g( x ) = 1 επαλη­θεύεται για εκείνα τα χ για τα οποία ισχύει:

• f(x)= 1 ή • g(x)=O και f(x) * Ο ή • f( χ )=-1 και g( χ )=άρτιος

Κυριακοπούλου Κωνσταντίνα προς την y=x (διχοτόμος 1 ης και 3ης γωνίας αξό­νων)

5. ΟΡΙΣΜΟΣ Λογαριθμική συνάρτηση με βάση 8.Για Ο<α * Ι και θ>Ο ,θ ι >Ο,θ 2 >Οκαι κ ε IR ισχύ­το θετικό αριθμό α με α * Ι ονομάζουμε τη συνάρ- συν τα παρακάτω: τη ση f : (0,+ οο ) � IR με τύπο f(x)= logα χ :Εχει δε σύνολο τιμών το IR .

6. Αν α> Ι , τότε η f(x)= logα χ είναι γνησίως αύ­ξουσα, δηλαδή για Χ ι < Χ2 ισχύει f(x ι ) < f(x 2 ) , ενώ αν Ο<α :;t: 1 η f(x)= logα χ είναι γνησίως φθί­νουσα δηλαδή για χ ι < χ2 ισχύει f(x ι ) > f(x2 ) . 7 . Οι γραφικές παραστάσεις των συναρτήσεων f(x) = αχ και g(x)= logα χ είναι συμμετρικές ως

logα αχ = χ αιοg. θ = θ logα Ι = Ο logα α = Ι logα (θ ι · θ2 ) = logα θ ι + logα θ2

logα ( :� ) = logα θ ι - logα θ2

logα θκ = κ · logα θ

ΕΥΚΛΕΙΔΗΣ Β' 75 τ.3/29

Page 33: Ευκλειδης Β 75

Μαθη ματικά για την Β ' Λυκείου

9 ,Παρατηρήστε ότι: Αν χ · y>Ο,τότε logα (χ . y) = logα I χ . Υ Ι= logα I χ I + logα I Υ I καθώς

και logα [�) = logα � = logα I χ I - logα I Υ I . Υ I Y l

Άρα logx 2 =log lx l+log lx l=2loglx Ι . Γενικά για κάθε ν Ε Ζ ισχύει : logα X 2v = 2v logα I χ I ,με X ::f. Ο 1 Ο "Δεν πρέπει να ξεχνάμε τους περιορισμούς, όπου αυτοί απαιτούνται , κατά την λύση των λογαριθμι­κών εξισώσεων και ανισώσεων.

Δίνονται οι συναρτήσεις f(x) = 5' -(�)' + 2010 ,

h(x) = - - 3' και g(χ)= Ιn � ( 1 J' [ 2' 5) 3 e' + 1 α) Να αποδείξετε ότι η f(x) είναι γνησίως αύ­ξουσα . β) Να αποδείξετε ότι η h(x) είναι περιττή. γ) Να συγκριθούν οι αριθμοί � +-1- και 510 +-1-1024 512 δ) Να βρεθεί το σημείο τομής της γραφικής πα­ράστασης της g(x) με τον χ 'χ.

α)Έστω χ 1 , χ 2 Ε JR , με χ ι < χ2 .Τότε αφού 5> 1 και

I θ , - < 1 α ισχυουν 5' ' < 5' ' ( 1 )'' ( 1 )'' και - > -2 2 2 ( I ) ' ' ( I )' ' ( Ι )'' ( Ι )'' => - 2 < - 2 => - 2 +2010<- 2 +2010

( 1 ) ' ' ( I ) ' ' Άρα 5' ' - 2 + 20 1 0 < 5' ' - 2 + 20 1 0

Δηλαδή f(x ι ) < f(x 2 ) ,οπότε f(x) γνησίως αύξουσα. β) Το πεδίο ορισμού της h(x) είναι το JR . Άρα για κάθε χ Ε JR -χ Ε JR και

h(-x)= (l)-x - 3-' = 3' - (l)' =-h(χ),άρα περιττή .

γ)Αποδείξαμε ότι η f(x) είναι γνησίως αύξουσα. Άρα: 9< 1 0 => f(9)<f( I O) => ( 1 )9 ( I ) ι ο 59 - 2 + 20 1 0 < 5 ι ο _ 2 + 20 1 0 ::::>

59 _ _ Ι_ < 5 ι ο _ _ Ι _ => 59 + _1 _ < 5 ι ο + -1-

5 1 2 1 024 1 024 5 1 2 δ)Ένας αριθμός αριθμός χ Ε JR ανήκει στο σύνολο {e' + 1 ::f. Ο ορισμού της g αν , και μόνο αν : ez ' _ 5 (I ) . -- > 0 e' + Ι

(1) <=> e2 ' - 5 > Ο <=> e2 ' > 5 <=> χ > !η 5 .Η τετμη-2

μένη του κοινού σημείου της γραφικής παράστα­σης της g και του άξονα χ ' χ είναι η λύση της εξί-

σωσης g(χ)=Ο .Με χ > lη 5 έχουμε: 2

[e2x -5) e2 x - 5 g(x)=O <=> Ιη -- =Ιη 1 <=> . 1 <=> e' + 1 e ' + 1

e2 ' - e' - 6 = Ο <=> ( e' + 2 ) · ( e' - 3 ) =Ο <=>

e' =-2(αδύνατη) ή e' =3 <=> x=ln3 , η οποία είναι δεκτή αφού ικανοποιεί τον περιορισμό ,γιατί 21η3=1η9>1η5 .Άρα η γραφική παράσταση της g τέ­μνει τον χ 'χ στο Κ(lη3 ,0) . ?\ σκηση 1 1 1 α) Να βρείτε για ποιες τιμές του α Ε JR η συνάρ-τηση f(x)=(S-αJ' είναι γνησίως αύξουσα στο JR • α-2 β)Αν α='!.. να λυθεί η εξίσωση : f(2x)=� -6 -� -4'

2 2 2

α)Η συνάρτηση f είναι γνησίως αύξουσα στο JR

όταν και μόνο όταν {� : � ( Ι ) . -- > 1 α - 2

( 1 ) <=> {� : � <=> {�;-22α -- - 1 > 0 > 0

α - 2 α - 2 {α ::f. 2

{α ::f. 2 <=> 2(5 - α) <=> <=> 2 < α < 5 --'------'- > 0 2(5 - α)(α - 2) > 0

α - 2

β)Αν α=2 ,τότε f(x)= 3' .'Εχουμε: 2

f(2x) = � · 6' -� · 4' <=> 32' = � · 6 ' -� · 4' <=> 2 2 2 2

5 3 9' 5 6' 3 9' = - · 6' - - · 4' <=> - = - · - - -2 2 4' 2 4' 2

<=> � = � . (i) ' - � <=> (�)lx - � . (�)' + � = ο 22 ' 2 4 2 2 2 2 2

[( 3 ) ' ] [( 3 ) ' 3 ] ( 3 ) ' <=> 2 - ] . 2 - 2 = Ο <=> 2 - 1 = 0

( 3)' 3 ( 3 )' (3)' 3 (3)' (3)0

ή 2 -2 =ο <=> 2 = 1 ή 2 =2 <=> 2 = 2

ή (%)' = (%} <=:> χ=Ο ή χ= Ι

λ σ •η� -c�η J ' � Να λυθούν οι εξισώσεις :

ΕΥΚΛΕΙΔΗΣ Β' 75 τ.3/30

Page 34: Ευκλειδης Β 75

Μ αθηματικά για την Β ' Λυκείου ( ) 2 x- l α) 52>+ 1 •

215 = 625 β) 13 ' - 14Ji3' + 13 = Ο

γ) 8' + 1 = συνχ δ) (χ2 - 7Χ + 1 1)3 '-6 = 1

α) 52 x+ l . ( 215 ) 2 x - l = 625 <=> 52 x+ l . 5-2( 2 x - l ) = 54

<::::> 52x+l . �χ+2 =� <::::> )2χ+3 =� <::::> -2Χ+ 3=4 <::::> χ�..!. 2

β) 1 3' - 14J13' + 1 3 = 0 <::::> J132' - 14Jl3' + 1 3 = 0 ν 1 3 = ω {m' =ω { r;::; '

<=> (m' )2 - 14Jl3' + 1 3 = 0 <=> ω2 - 14ω + 1 3 = 0

<::::> {Jl3' =ω <=> Jl3' = 1 ή J13' = 1 3 <=> χ=Ο ή χ=2 ω=1ήυ=13 γ) 8' + 1 = συνχ <=> 8' = συνχ - 1 Για κάθε χ ε IR είναι 8 ' >Ο ενώ συνχ- 1 � Ο ,άρα η εξίσωση είναι αδύνατη . δ)Η εξίσωση ( x c - 7χ + ι ι )3 ' -6 = ι επαληθεύεται για εκείνα τα χ για τα οποία :

. ,

χ2 -7χ+ 1 1 = ι <::::> x 2 - 7x + l 0 = 0 <::::> χ=2 ή χ=5 3χ-6=0 και χ 2 - 7χ + 1 1 =ι. Ο .Έχουμε 3χ-6=0 <=> χ=2 .Η τιμή αυτή είναι δεκτή αφού 22 - 7 . 2 + 1 1 = 4 - ι 4 + 1 1 = ι * ο . χ 2 - 7χ + ι ι = - 1 και (3χ-6) άρτιος. Έχουμε: χ2 -7χ + 1 1 =-1 <=> χ2 -7χ+ ι2=0 <=> χ=3 ή χ=4. Για χ=3 είναι: 3χ-6=3 · 3-6=9-6=3 περιττός άρα η τιμή χ=3 απορρίπτεται Για χ=4 είναι 3χ-6=3 · 4-6=6 άρτιος άρα η χ=4 δεκτή. Τελικά η ε­ξίσωση έχει λύσεις τις χ=2 ή χ=5 ή χ=4.

Να λυθούν οι ανισώσεις : x-s ( 1 )Ιχ+91-Ι l α) 81 χ+4 < 33'-8 β) - > -4 256

γ) 24'� <{48J6)2,_9

δ) log ( tog ( x2 - 1 9χ + 1 00)} < 0

-3(χ+2)(χ-2)(χ+4)<0 <=> χ ε ( -4, -2) υ (2, +οο) ( 1 ) Ι χ +9 Ι- Ι 1 ( 1 ) Ι χ +9 Ι- Ι ( 1 )4 β) - > - <=> - > -4 256 4 4 Επειδή � < 1 , η

(�) χ είναι γνησίως φθίνουσα ο­

πότε θα είναι Ιχ+9 1- 1 <4 <=> Ιχ+9 1<5 <=> -5<χ+9<5 <=> -ι 4<χ<-4 3 γ) 24χ-� < (48-!6γχ-9 <=> [(2J6)2T2 <[(2J6)3τ-9 <=> ( 2-!6)l x-J < ( 2-!6)δχ-lΊ . Επειδή 2)6 > 1 η

( 2-!6 ) ' είναι γνησίως αύξουσα ,οπότε θα είναι 2χ-3<6χ-27 <=> 4χ>24 <=> χ>6 . δ) 'Εχουμε : \og ( Iog (x 2 - 1 9χ + 1 00) ) < 0 <=>

χ2 - 1 9x + l 00 > 0 <=> log (x 2 - 1 9χ + 1 00) > 0

log ( Iog ( x 2 - \ 9χ + 1 00) ) < 0 χ 2 - 1 9χ + ι οο > ο

<=> log ( χ 2 - 1 9χ + 1 00) > log 1 log ( log ( χ 2 - 1 9χ + 1 00) ) < log 1 {χ2 - ι9χ+100>0 J

J {χ- -19χ+100>1 <=> χ- -19χ+100>1 <=> ( 7 ) lcg(x2 -19x+100) <1qs10 lqs χ- -19χ + 100 <1

<=> <=> <=> χ ε (9, 1 0) {χ2 -19χ+99>0 {χ ε.ΙR χ2 -19χ+<χ><Ο (χ -9)(χ - 10) <0

Να βρεθούν τα πεδία ορισμού των συναρτήσε­ων: α) f(x) = .J-52' + 6 · 5' - 5

β) g(x)�Ιn(ln(x-1)) γ) h(x) = ( logj (ιogj (χ+ 9) J J x-s δ) λ( χ)= 5χ + 13

α)Η ανίσωση 8 1 χ+4 < 33 ' -8 αληθεύει για χ =ι. -4 . Έ 3 χ-5 χ -5 4(χ-5)

χουμε : 8 1'+4 <33'-8 <::::> (34 ) ' +4 < 33 '-8 <::::> 3 χ+4 <Υ'-8 . Επειδή 3> 1 η 3 ' είναι γνησίως αύξουσα, οπότε θα ' 4(χ-5) 3 8 4χ - 20 - (3χ - 8)(χ + 4)

ειναι --< χ- <=> < 0 χ+4 χ + 4 4χ - 20 - 3χ 2 - 12χ + 8χ + 32 ο <=> < <=> χ + 4

-3χ 2 + 1 2 --- < ο <=> -3(χ 2 - 4)(χ + 4) < ο <=> χ + 4

Ιοg(ημχ + -) 2

α)Για να ορίζεται η συνάρτηση f(x) πρέπει και αρ­κεί: -52' + 6 · 5 ' - 5 2 0 ( 1 ) . ( 1 ) <=> {5' � Υ <=> {5' = Υ <=> 1 � 5 ' � 5 -y- + 6y - 5 2 0 1 � y � 5 <=> 5 ° � 5' � 5 1 <=> Ο � χ � 1 . Επομένως το πεδίο ορισμού της συνάρτησης f είναι το Α=[Ο, 1 ] .

ΕΥΚΛΕΙΔΗΣ Β' 75 τ.3/31

Page 35: Ευκλειδης Β 75

Μ αθηματικά για την Β ' Λυκείου

β) Για να ορίζεται η συνάρτηση g(x) πρέπει και {χ - Ι > ο αρκεί: (Σ) In(x - Ι) > Ο (Σ) <::::> {χ > Ι <::::> {χ > Ι <::::> χ > 2 . Επομένως το χ - Ι > Ι χ > 2 πεδίο ορισμού της συνάρτησης g είναι το Β=(2, +οο ) . γ) Για να ορίζεται η συνάρτηση h(x) πρέπει και {χ + 9 > ο αρκεί: log � (χ + 9) > ο

(Σ) .

(Σ) ς;{�:j�: + 9) > ιοgj ι ς; {: : ;: ι

<::::>{χ > -9 <::::> -9 < χ < -8 . Επομένως το πεδίο ορι­χ < -8 σμού της συνάρτησης h είναι το Γ=(-9,-8) . δ) Για να ορίζεται η συνάρτηση λ(χ) πρέπει και

!ημχ + � > 0 αρκεί: ( 2 3 ) (Σ) . log ημχ + - :;t: O 2 {ημχ +� > ο {ημχ > -�

(Σ) <::::> � <::::> � <::::> ημχ :;t: η μ(-�)

ημχ +- :;t: 1 ημχ :;t: --2 2 Επομένως το πεδίο ορισμού της συνάρτησης λ είναι

π 7π το Δ= {χ ε IR / x :;t: 2κπ-- και χ :;t: 2κπ+- ,κ ε Ζ } 6 6 Α σκη ση 6 '1 Να υπολογιστεί η τιμή των παραστάσεων:

1 log 3 - log 27 + - log 243 + log 9 Α= 5

log 180 - 2 1ogJ6 '•(τ- ::g, :) J7'og, 4 • 3 'og ι οο

Β= log e g, Γ=-----,-----,-----3 ' log7 ( log3 37 ) · 61og• 54 Λ \J ση : Ι log 3 - log 27 + - log 243 + log 9 α) Α= 5

log 1 80 - 2 logJ6 Ι log 3 - log 33 + - log 35 + log 32 5

log 1 80 - log J62 I log 3 - 3 log 3 + - · 5 log 3 + 2 log 3 5

log 1 80 - log 6

= log 3 ι 1 80 og-6

log 3 ιοg 30

ιοg 3 = log 3 ιοg 1 0 + ιοg 3 Ι + ιοg 3

β) Β= ιοg3 e " τ ιοg, 9 = log3 .!2_ - ιοg2 3 =

1 ( ' 9 log, J ) ( ) 2 ιοg2 9

_ _ ι ( 1 9 ιοg2 3 ) - ι ( 1 9 ιοg2 3

) _ - - og3 -- - og3 - - -2 ιοg2 32 2 2 log2 3

I - log7 4 2 7 2 • 3

ιοg7 ( 7 ιοg3 3) · 54 7 log, J4 · 9 J4 · 9 1 8 1 --=- =-ιοg, 7 . 54 54 54 3

Ά σ κη ση 7'1 Δίνεται η συνάρτηση f(x) = log(� - χ) α)Να βρεθεί το σύνολο ορισμού της f(x). β)Να δείξετε ότι η f(x) είναι περιττή. γ)Να βρεθούν τα σημεία τομής της γραφικής παράστασης f(x) με την ευθεία y=2. Λ\J ση : α) Ένας αριθμός χ ε IR ανήκει στο σύνολο ορισμού {χ 2 + Ι ;::: Ο της f αν , και μόνο αν : � (Σ). ν χ2 + 1 - χ > Ο (Σ) <::::> � > χ <::::> χ ε IR , γιατί για κάθε χ ε IR έχουμε : J Χ2 + I > R � χ ι;::: χ � .J χ 2 + I > χ .Άρα το σύνολο ορισμού της f είναι το IR . β) Για κάθε χ ε IR έχουμε (-χ) ε IR και επίσης :

( Jx2 + 1 +χ) · ( Jx2 + Ι -χ) f( -χ)= log(�( -χ )2 + I +χ) = ιοg-'----�2 �---'­�-χ I � =ιοg = - log(ν x - + 1 - x) =-f(x). �-χ

Άρα η f είναι περιττή . γ) Οι τετμημένες των κοινών σημείων της γραφι­κής παράστασης της f και της ευθείας y=2 είναι οι λύσεις της εξίσωσης: f(χ)=2 .Με χ ε IR έχουμε : f(x)=2 <::::> ιοg<)χ2 + l -x) = 2 <::::> ιog(Jx2 + 1 - χ) = ιοg ιΟΟ <::::> .J χ 2 + Ι - χ = 1 00 <::::> � = 1 00 + χ <::::> {χ + 1 00 > ο {χ>-100 , <::::> ? 2 <::::> χ 2 + 1 = ( χ + 1 00γ χ- +Ι=χ +200x+ lO.αxJ {χ>-ΙΟΟ 9.999 9.m <=> χ =---χ=-- 200

200

ΕΥΚΛΕΙΔΗΣ Β' 75 τ.3/32

Page 36: Ευκλειδης Β 75

Μαθη ματικά για την Β ' Λυκείου

Ά . . . Μ ( 9.999 2) ρα το σημειο τομης ειναι το --- , . 200 Άσκηση sιι

Ν α βρεθούν οι τιμές του χ ε (Ο, ; ) έτσι ώστε

( ) I 1 + συν2χ οι αριθμοί log ημ2χ , -2 1og 2 και og 2 να αποτελούν με τη σειρά που δίνονται διαδοχι­κούς όρους αριθμητικής προόδου. ΛίJση : Για να αποτελούν οι αριθμοί log (ημ2x ) , -2 \og 2 ,

ι 1 + συν2χ og----2 διαδοχικούς όρους αριθμητικής προόδου πρέπει και αρκεί :

( ο ) 1 + συν2χ 2( -2 \og 2 )= \og ημ- χ + log ( 1 ) . 2 Για να ορίζεται η παραπάνω σχέση πρέπει και αρ-{ημ2χ > ο κεί: 1 + συν2χ (Σ ) . ---- > 0 2

{ημχ "#- ο {ημχ "#- ο (Σ) <=> ο <=> ,που ισχύουν για συν-χ > Ο συνχ 7:- Ο

κάθε χ Ε (ο, Ξ} 'Εχουμε :

( 1 ) <=> -4 1og 2 = \οg ημ2χ + Ιοg συν2χ <=> logT4 = Ιοg(ημ2χ · συν2χ )

I 1 <=> - = ημ-χ · συν-χ <=> - = 4ημ-χ · συν-χ <=> 1 6 4 1 ' 2 2 1 ' 2 I Α ' ' - = ημ- χ <=> ημ χ=- η ημ χ=-- . πο αυτες 4 2 2 δεκτή 2 1 ' ημ χ=- αφου 2 είναι μόνο η

Ο<χ<� <=> 0<2χ<π ,οπότε ημ2χ>Ο. 2 'Ε χουμε ημ2χ=..!.. <=> ημ2χ=ημ � <=> 2χ=2κπ+ � ή 2 6 6

5π π ' 5π 2χ=2κπ+- Κ Ε 2. <::::> χ=κπ+- η χ=κπ+ - Κ Ε 2. . 6 ' 1 2 12 ' ' π π π π 5π Όμως Ο<χ<- <=> Ο<κπ+- <- <=> -- <κπ<-2 1 2 2 1 2 1 2

Ι 5 <=> --<κ<- <=> κ=Ο επειδή κ Ε Ζ. . Οπότε για 1 2 1 2 '

κ= Ο έχουμε π χ=- . Ι 2 Επίσης π Ο<χ<- <=> Ο< 2 5π π 5π π 5 Ι κπ+-<- <=> -- <κπ<- <=> -- <κ<- <::::> k=O Ι2 2 12 1 2 Ι 2 Ι 2 '

δ ' '77 ο ' ο ' 5π επει η κ ε !L.J . ποτε για κ= εχουμε χ=- . Ι 2 Άσκηση 9'� Σε μία αριθμητική πρόοδο είναι α 1 =ln4 και α 2 =Ιη32.Να δειχθεί τι το άθροισμα των ν πρώ­των όρων της δίνεται από τον τύπο

ν sν = 2 · (3ν + 1) · 1η 2 . β)Αν χ>Ο και Ιοgχ=9,να υπολογιστεί η τιμή της

�χ4Μ παράστασης: log I ν χ�χ4� γ)Να λυθεί η εξίσωση : Ιοgω ( 3χ: 15 ) = 1 ΛίJση :

32 α)Είναι ω=α -α =Ιη32-Ιη4=\η-=Ιη8=Ιη2 3 =3 \η2 2 I 4 και sν =�[2α, +(ν- 1)ω] =� [2 · lη 4 + (ν - Ι ) · 3 · 1η 2] =

ν ν =- ( 4 \η 2 + 3 ν \η 2 - 3 \η 2 ) =- ( 3 ν + Ι ) · \η 2 2 2 1x 4 �xif;. 1χ4 ��χ · χ3 β)Εχουμε : log ν = Iog ν = �χ� �χ��

�·χ-4 1-�=χ=4 J� χ 4 . χ 60 ρ = log = log = log-- = Jχιρ J�X I 3 . Χ Ι 5 3ρ 64 χ3ο" � 36 36 =log- =Iogx 30 =- logx=- · 9 = 1 0 8 28 30 30 ' χ 30

x + 2 7:- l γ) Πρέπει και αρκεί: χ + 2 > Ο

3χ + Ι 5 > Ο 4 δηλαδή χ E (-2, - I) u (- I , +oo) . Επομένως για χ Ε (-2, - Ι ) υ (- 1 , +οο) έχουμε:

Ιοgω (3χ: Ι5)=1 <=> lc>g,ω ( 3χ: I5)=1ogx+2 ( χ +2) <=>

3χ + Ι 5 2 7 ' δ ' --- = χ + <=> χ= που ειναι εκτη . 4 Άσκηση ι ο ιι Να λυθούν τα συστήματα: {χ 1" Υ = � {'% (χ-2) - Ι% (Υ-3) =0 α) e β)

2 ι2+ Ιn χ-' = S'" Y log(x+ 1)+ log(y-2)= 31og2

ΕΥΚΛΕΙΔΗΣ Β' 75 τ.3/33

Page 37: Ευκλειδης Β 75

Μαθη ματικά για την Β ' Λυκείου

{3χ · 7Υ = 147 γ) 3y . γ = 63 .\1>ση : α) Αρχικά πρέπει και αρκεί να είναι χ>Ο και y>O και το σύστημα ισοδύναμα γράφεται : { lη x · lη y = lη e-4 { ln x · lη y = -4 2Ι 2+3 Ιn χ = 23 Ιn y <:::::> ln x = ln y - 4 <=> {( Ιη y - 4) · 1η y = -4 <=> { lη 2 y - 4 \η y + 4 = 0

!η χ = !η Υ - 4 !η χ = !η Υ - 4 {( Ιη y - 2)2 = 0 {ln y = 2 {lny=2 <:::::> !η χ = !η y - 4 <:::::> Ιη χ = !η y - 4 <:::::> lnx=-2 {y = e2

<:::::> 1 X =­e2 χ - 2 > 0 y - 3 > 0 β) Αρχικά πρέπει και αρκεί να είναι (Σ). χ + 1 > ο y - 2 > 0

(Σ) <=>{χ > 2 και το σύστημα ισοδύναμα γράφεται : y > 3 {IO&(X -2) = IO&(y-3) {χ -2 = y-3 log[(x+ 1) · ( y-2)]= logi <:::::> ( x + 1) · ( y-2) = 8

<:::::> {χ = Υ - 1 {χ =Υ - 1 xy-2x + y- 1 0=0 <:::::> y(y- 1) - 2(y- 1) +y- 1 0=0 <:::::> {χ = Υ - 1 {χ = Υ - l y2 - 2y - 8 = Ο <=> y = 4ήy = -2

Η τιμή y=--2 απορρίπτεται αφού πρέπει να ισχύει y>3 άρα y=4 οπότε χ=4-1=3 , η οποία επαληθεύει τον περιορισμό χ>2 .Άρα η λύση του συστήματος είναι η (x,y)=(4,3)

γ) <:::::> 3y . γ = 63 {3χ . γ = 1 47 3y . γ = 63

{3χ ·γ = 1 47

3' . γ . }Υ . γ = 1 47 · 63 {3χ · Τ =147 {3' . γ = 147 {3' . γ = 147

<:::::> 3y . 7χ =63 <:::::> 3y . γ = 63 <:::::> 3y . γ = 63 3x+y . 7x+y =9261 2 1x+y = 2 13 x + y= 3

{3χ . 73-χ = 147

<:::::> 3y . γ = 63 <:::::> <:::::> x + y = 3

3 7

}Υ · γ = 63 <:::::> x + y = 3

{�Υ� � χ = 63 <:::::> {Χ = 1 y = 2 x + y = 3 Ά σ κη ση 1 1 '� Δίνεται η συνάρτηση f(x) = Ιη(ημχ + συνχ + 5) α) Να δείξετε ότι ln(S - .J2) � f(x) � ln(S + .J2) β) Αν α ν είναι γεωμετρική πρόοδος με α 1 = f(O)

λ 1 β θ , , , Ι η 1 2 96 και = '2 να ρε ει ποιος ορος της ει ναι ο 1 6 γ) Να λυθεί η εξίσωση : logif;. + �log x = log x .\ί>ση : α) Επειδή για κάθε χ Ε � ημχ+συνχ +5>0 η συ­νάρτηση ορίζεται για κάθε χ Ε � .

'Ε χουμε :ημχ+συνχ= J2 · η μ (χ + �) . Άρα

f(x)= ln [ J2ημ(χ + �) + 5] και

- I � ημ( χ + �) � Ι => -J2 � J2ημ( χ + �) � J2

=> 5 -J2 �J2ημ( x + �) + 5 � 5 +J2 Η συνάρτηση Ιηχ είναι γνησίως άυξουσα , οπότε Ιη(5 -J2) � !η [ J2ημ( χ + �) + 5] :;:; Ιη(5 + J2) => Ιη(5 -J2) � f(x) � lη(5 + J2) β) α 1 =f(O)=lη6 . Είναι

λv- 1 \η 1 296 J 6 ( 1 )ν- Ι α = α · => = η · - => ν I J 6 2 Ιη 64 = Ιη 6 =>

4 1η 6 = Ιη 6 => 2v- Ι = 4 1 6 2ν- Ι 1 6 2ν- Ι => ν-1 =2=> ν=3 {χ > 0 γ) Πρέπει και αρκεί να είναι : ( 1 ) . log χ 2: Ο

{χ > 0 (Ι) <:::::> <:::::> χ 2: 1 . Με χ 2: 1 έχουμε : χ 2: I log�+J!ogx = logx <:::::> .!. ιog x + �log x = log x 3

rι::::-:; {�log χ = ω <:::::> 2 log χ - 3ν ιu� λ = ο <:::::> ο <:::::> 2ω- - 3ω = Ο {�log x = ω 3 <:::::> logx=O ή ω = Οήω = -2

9 x= J 04

9 logx=- <:::::> χ= 1 4 ή

ΕΥΚΛΕΙΔΗΣ Β' 75 τ.3/34

Page 38: Ευκλειδης Β 75

Κανονι κά Πολύγωνα - Μέτρηση Κύκλου

Στα πλαίσια αυτού του άρθρου, είναι δύσκολο να μιλήσουμε για την τεράστια αξία της Ευκλείδειας Γεωμετρίας. Πέρα από τη χρησιμότητά της σε τε­χνικές εφαρμογές, από την Αρχαιότητα μέχρι σή­μερα, επιπλέον προάγει την κριτική σκέψη και ε­ξοικειώνει κάποιον στην αποδεικτική διαδικασία. Παρόλα αυτά, τα τελευταία χρόνια παρατηρείται μία τάση υποτίμησης της Γεωμετρίας στα σχολεία και ιδιαίτερα στη Β ' Λυκείου, με τη λογική ότι, οι προαπαιτούμενες γνώσεις της, για τα Μαθηματικά κατεύθυνσης της Γ ' Λυκείου, είναι "περιορισμέ­νες" . Όμως πολλά θέματα Πανελληνίων εξετάσεων, που έχουν τεθεί κατά καιρούς, απαιτούν και γνώσεις Γεωμετρίας. Αλλά εκτός αυτού, δεν θα πρέπει να υπάρχει αδιαφορία από τους μαθητές για το συ­γκεκριμένο μάθημα, γιατί είναι ένα εργαλείο για τη συνέχιση των σπουδών τους σε πολλές σχολές, καθώς και χρήσιμο στην καθημερινή τους ζωή . I . Τ Υ Π Ο Ι ΣΤΑ K A � O !\ I K A Π Ο Λ Υ ΓΩ 'Ι Α

' ' 360° κεντρικη γωνια ων = --ν γωνία

• Περίμετρος Ρ ν = ν · λν 1 Εν = 2Ρν · αν

ισχύει

και εμβαδόν

λ2 α� + -;f = R 2 όπου αν: το απόστημα και λv: η πλευρά του.

Θανάσης Τσιούμας

• Δύο κανονικά πολύγωνα με τον ίδιο αριθμό πλευρών είναι όμοια, οπότε ισχύει η σχέση :

� =!._ =� = Ρν και � = [�) 2 λ: R' α: Ρ� Ε: λ: • Στις εγγραφές (ισόπλευρου - τετραγώνου και

καν. εξαγώνου) έχουμε : R α3 = Rημ30° = 2 λ3 = 2Rσυν30° = R J3 R..fi λ4 = 2Rσυν45° = R .J2 α4 = Rημ45° = --2 RJ3 α6 = Rημ60° = -2- λ6 = 2Rσυν60° = R

�χόλιο

Για να εγγράψουμε ένα κανονικό πολύγωνο με ν πλευρές σ' ένα κύκλο, θα πρέπει να τον χωρίσουμε σε ν ίσα τόξα και να ενώσουμε τα αντίστοιχα ση­μεία. Ο χωρισμός όμως του κύκλου, με κανόνα και δια­βήτη , σε ν ίσα τόξα δεν είναι πάντοτε δυνατός. Ο Gauss απέδειξε ότι αυτό μπορεί να γίνει όταν: ν = 2" ρ 1 · ρ2 . . . ρν όπου ρ ι , ρ2, . . . , ρν φυσικοί πρώτοι αριθμοί της μορφής: 22'' + Ι, όπου κ, μ ε Ν *

1 1 . Τ Υ Π Ο Ι Σ Τ Ο !\ Κ Υ Κ Λ Ο

Τόξο σε μοίρες Τόξο σε rad Μήκος Μήκος τόξου Μήκος τόξου κύκλου

πRμ0 L = 2πR f =-- f = α · R 1 80° Εμβαδόν Εμβαδόν κυκλι- Εμβαδόν κυκλι-κυκλικού δίσκου κού τομέα κού τομέα

(Ο.ΑΒ) = πR 2μο (Ο.ΑΒ) = ..!.. αR2

Ε = πR2 2 360° (όπου α είναι α-

ριθμός rad)

ΕΥΚΛΕΙΔΗΣ Β' 75 τ.3/35

Page 39: Ευκλειδης Β 75

Μαθη ματικά για την Β ' Λυκείου

Ση μείωση � 1 2 Επειδή S = α · R και (Ο.ΑΒ) = -α · R θα έχουμε 2

� 1 ότι (Ο.ΑΒ) = -S · R 2

Π αρ{ιδειγμα

ο

s

Στο σχήμα είναι: ΑΟΒ = 90° και Τι = Τ2 να α­ποδείξετε ότι ΑΟΒ = 1 rad

Α ΛίJση

Δ � Ισχύει: Τ1 + Τ = Τ2 + Τ � (ΟΑΒ) = (Ο.ΑΓ) � ..!_ S · R = ..!_ · R · R οπότε S = R που σημαίνει ότι η 2 2 AOB = I rad

Σχόλιο

Στο βιβλίο κατεύθυνσης Γ Λυκείου υπάρχει η πα­ρακάτω άσκηση (στα μέγιστα - ελάχιστα) που α-1 παιτείται η χρήση του τύπου Εκ. τ. = -αR 2 , α σε 2 rad, όμως οι πιο πολλοί μαθητές τον αγνοούν ! !

Άσκηση 4 σελ. 29 1 ( Γενικές παραγώγων) ι Β

Με σύρμα μήκους 20 m να περιφράξουμε ανθό­κηπο σχήματος κυκλικού τομέα. Να βρείτε την ,....-..._ ακτίνα r ώστε (Ο.ΑΒ) = max . ΛίJση

Θα πρέπει να εκφράσουμε το εμβαδόν του κυκλι­κού τομέα ως συνάρτηση του r. Το μήκος τόξου θ rad είναι L = r · θ ενώ

� I 7 (Ο.ΑΒ) = -Γ · θ 2 Η περίμετρος του κυκλικού τομέα θα είναι

. 20 - 2r 2r + rθ = 20, οποτε θ = , Ο < r < I Ο r . ,---..... 1 7 20- 2r 7 Οποτε (Ο.ΑΒ) = Ε(r) =-Γ-- = 10r -Γ , r ε (Ο, 10) 2 2

Επειδή -1 < Ο άρα το τριώνυμο E(r) = -r2 + 1 Or για β 1 0 r = -- = ---= 5 παρουσιάζει max το 2α 2( - 1)

Eιnax=25 . Βέβαια το μέγιστο στη Γ Λυκείου το βρίσκουμε κυρίως με τη βοήθεια των παραγώγων.

Λυμένες Α σκι]σεις

I . Να αποδείξετε ότι μια αυλή, μπορεί να κα­λυφτεί μόνο με τρία είδη πλακιδίων, που είναι ίσα μεταξύ τους και έχουν σχήμα κανονικού πο­λυγώνου.

Λύση

Έστω από ένα σημείο Ρ είναι τοποθετημένα κ ίδια πλακίδια σχήματος κανονικού ν-γώνου Α ν φν η γωνία κανονικού ν-γωνου τότε θα ισχύει: κ · Ψv = 360° <::::>κ · (180"- 360")=360" <::::>κ=2+_±_ ν ν-2 Όμως κ φυσικός αριθμός οπότε θα πρέπει το ν - 2 να διαιρεί το 4, άρα ν = 3 , ν = 4, ν = 6.

Επομένως, μπορούν να τοποθετηθούν: • Για ν = 3 , κ = 6 πλακίδια σχήματος ισόπλευ­

ρου τριγώνου • Για ν = 4, κ = 4 πλακίδια σχήματος τετραγώ­

νου και • Για ν = 6, κ = 3 πλακίδια σχήματος κανονικού

εξαγώνου ο

2 . Να υπολογίσετε την πλευρά, το απόστημα ΕΥΚΛΕΙΔΗΣ Β' 75 τ.3/36

Page 40: Ευκλειδης Β 75

Μ αθηματικά για την Β ' Λυκείου

και το εμβαδόν του περιγεγραμμένου τριγώνου σε κύκλο (Κ. R) ως συνάρτηση της ακτίνας R.

Λί)ση

Έστω Δ, Ε, Ζ τα σημεία επαφής του τριγώνου με τον κύκλο.

Α

Έχουμε: ΚΔ ΑΔ R και εφ60° =- άρα R ΑΔ = Rεφ60° = RJ3 (1)

Α8 , Α8 Είναι ΑΔ =- οποτε 2 2ΑΔ δηλαδή Α8 = 2RJ3. Το εμβαδόν

1 1 � ? J3 ? (Α8Γ) = -Α8 · ΑΓ · ημ60° = - (2Rν3 )-- = 3R-2 2 2

3 . Δίνεται κανονικό πεντάγωνο ΑΒΓΔΕ εγγε­γραμμένο σε κύκλο (Κ, R). Έστω Μ το μέσο του ,..--... τόξου ΒΓ. Να αποδειχτεί ότι: i) ΜΑ - ΜΒ = R ii) MA · MB = R2

Λύση

Έστω ότι η ΑΜ τέμνει την Κ8 στο Σ. � � ο 360ο 1 oso Ε δ , i) Η γωνία Α8Γ = φ5 = 1 80 --5- = . πει η

Κ8 η διχοτόμος της γωνίας Α8Γ τότε η 82 = 54°, επίσης

� 360° � (8Γ) = --= 72° και Μ το μέσο του 8Γ. 5

διότι

Ε

� � � � � (ΑΒ) 72 ο Άρα Σ8Μ = 8ι + 82 = 72° και Μι =--=-= 36 2 2

Επομένως η γωνία Σ ι = 1 80° - (72° + 36°) = 72° οπότε το τρίγωνο ΜΣ8 είναι ισοσκελές δηλαδή

ΜΣ = Μ8 ( 1 ) . Η γωνία Σ2 = Σ ι = 72° και Κ = 72° άρα και το τρίγωνο ΚΑΣ είναι ισοσκελές δηλαδή

ΑΣ = ΑΚ = R (2) Από (1 ), (2) έχουμε

ΜΑ - Μ8 = ΜΑ - ΜΣ = ΣΑ = ΚΑ = R ii) Τα ισοσκελή τρίγωνα ΚΑΜ και ΣΚΜ είναι ό-

� � ΜΑ ΜΚ μοια διότι ΑΚΜ = ΚΣΜ = 144° άρα ΜΚ = ΜΣ , ΜΑ R , 2 και από ( 1 ) , (2) ειναι R = Μ8 η ΜΑ · Μ8 = R .

4 . Σε κύκλο (0, R) είναι εγγεγραμμένο κανονι­κό εξάγωνο με εμβαδόν Ε6• Περιγράφουμε, στον ίδιο κύκλο, κανονικό εξάγωνο με εμβαδόν Ε ' 6·

Ε' Να υπολογίσετε το λόγο -6 • Ε6

Λί)ση

Για το εγγεγραμμένο κανονικό εξάγωνο έχουμε RJ3 , , λ = R a6 = -- . Το περιγεγραμμενο κανονικο 6 '

2 εξάγωνο θα έχει πλευρά λ'6 και aπόστημα α'6 = R. Επειδή τα δύο κανονικά εξάγωνα είναι όμοια με λόγο ομοιότητας λ' ' λ' R λ' 2 _6 = α6 <=> _6 =----τ;; <=> -6 = � θα έχουμε : λ6 a6 λ6 Rν3 λ6 ν3

2 �: = (�: ) ' � �: = (�) ' � :: = � ΕΥΚΛΕΙΔΗΣ Β' 75 τ.3/37

Page 41: Ευκλειδης Β 75

Μαθηματικά για την Β ' Λυκείου

5. Από σημείο Σ που απέχει απόσταση 2R από κλο. το κέντρο Ο κύκλου (0, R) φέρουμε εφαπτόμε- Ρ

νο τμήμα ΣΑ του κύκλου. Α ν Β είναι το σημείο τομής του ευθύγραμμου ΟΣ με τον κύκλο, να υπολογίσετε σε συνάρτηση του R, την περίμε-τρο και το εμβαδόν του μεικτόγραμμου τριγώ-νου ΑΒΣ.

Σ

ΛiΗΗ] Έχουμε Α = 90°. Άρα το τρίγωνο ΟΑΣ είναι ορ­θογώνιο . Ε , Σ ΟΑ R 1 , Σ� 30ο ιναι ημ = ΟΣ = 2R = '2 οποτε =

Επομένως Ο = 60°. , 2RJ3 r:; Εχουμε ΑΣ = 2R συν30° = -- = Rν3 και 2 ΣΒ = 2R - R = R @ Έστω S η περίμετρος του μεικτόγραμμου τρι­

γώνου ΣΑΒ, τότε r:; πR60ο S = (ΑΣ) + τ� + (ΒΣ) = Rν.J + -- + R ΑΒ 1 800

= RJ3 + πR + R 3 άρα S = R (3J3 + π + 3) . 3 Αν Τ είναι το εμβαδόν του μεικτόγραμμου τρι­γώνου ΣΑΒ, τότε Τ = Ε(ΣΟΑ ) - Εκ τομ ( ΑΟΒ) =�R · 2Rημ60°

, R2J3 πR2 R2 αρα Τ = -- - - = -(3J3 - π) 2 6 6

πR2 60° 360°

6. Στο σχήμα η ΑΒ = 2R είναι διάμετρος, Α = 30° και η ΒΡ είναι εφαπτόμενη του κύκλου. ί) Ν α αποδείξετε ότι ΡΓ = .!_ ΑΓ 3 ίί) Να βρείτε το εμβαδόν Τ του γραμμοσκια­σμένου χωρίου ίίί) Να βρείτε τη δύναμη του Ρ ως προς τον κύ-

i) Στο ορθογώνιο τρίγωνο ΓΡΒ η γωνία ΓΒΡ = Α = 30° (χορδή - εφαπτομένη) οπότε ΓΡ = ΡΒ . Θέτουμε ΓΡ = χ οπότε ΡΒ = 2χ. Ισχύει 2 ΡΓ · ΡΑ = ΡΒ2 <=> χ · ΡΑ = (2χ)2 <=> ΡΑ = 4χ άρα ΓΑ = 3χ = 3ΡΓ.

Δ i i) Η ΒΟΓ = 60° άρα το Ο Γ Β είναι ισόπλευρο

Δ R2 J3 με εμβαδόν (Ο Γ Β) = -- . Από το ορθογώνιο 4 τρίγωνο ΒΑΡ έχουμε

' ' I RJ3 1 ΒΓ- = ΓΡ · Γ Α <=> R- = 3χ · χ <=> χ = -3-Δ Επομένως το εμβαδόν του Γ Β Ρ είναι

Δ 1 1 Rfi ' Δ R 2J3 (ΓBP) =- x · R =- ·-· R η (Γ Β Ρ) = -- ( l ) 2 2 3 6

Το εμβαδόν Τ = (O.rn) - (O rB) = 60ο πR2 - R2J3 = I 360° 4

= πR2 - R2 J3 (2) 6 4 Επομένως

Δ ( I ) R2J3 πR2 R2J3 R2 Τ = (ΓΒΡ) - Τ = -- - - +-- = - (5f3 - π) I ( 2 ) 6 6 4 6 i i i) Είναι

? Ρ 2 2 2 Ρ 2 2 4R-

�ιo. R J = ΟΡ -R = ΡΒ <=>�ω. R J = (2χ) = 4χ =-3-

7. Το σχήμα δείχνει δύο ημικύκλια. Το μεγα­λύτερο έχει κέντρο Ο και ακτίνα 4 cm, ενώ το μικρότερο είναι το (Κ, 3cm) και διέρχεται από το Ο. Να υπολογίσετε:

ΕΥΚΛΕΙΔΗΣ Β' 75 τ.3/38

Page 42: Ευκλειδης Β 75

Μ αθηματικά για την Β ' Λυκείου

Α Δ

α) Το εμβαδόν (Ο ΡΚ) � �

β) Τις γωνίες ω και Κ = φ ------.

γ) Το εμβαδόν του κυκλικού τομέα (Κ.ΟΡ) �

δ) Το εμβαδόν (Ο.ΡΒ) ε) Το εμβαδόν του γραμμοσκιασμένου χωρίου S

α) Παρατηρούμε ότι ΟΚ = ΚΡ = 3cm ως ακτίνες του μικρού ημικυκλίου και ΟΡ = 4. Από τον τύπο του Ήρωνα: Ε = �τ( τ - α)( τ - β )( τ - γ) όπου τ = 3 + 3 + 4 = 5 2 προκύπτει

Δ (Ο Ρ Κ) = �5(5 - 3)(5 - 3)(5 - 4) = #5 = 2JS cm2

Δ ι (α) r; β) Επειδή (Ο Ρ Κ) = - · 3 · 3 · ημφ = 2ν5 έχουμε 2

4J5 ημφ = -- και από τους τριγωνομετρικούς πίνα-9

κες βρίσκουμε φ = 84°. Το τρίγωνο ΟΡΚ είναι ι-� �

σοσκελές άρα 2ω + 84° = 1 80° <=> ω = 48° .

γ) Το εμβαδόν (K.Or) = 840 π · 32 = 2, ι Οπ cm2 360°

' � 48° 7 7 δ) Επισης (Ο.ΡΒ) = --π · 4- = 2, 1 3π cm-3600 ε) Το ζητούμενο εμβαδόν του γραμμοσκιασμένου χωρίου S θα είναι :

� � Δ S = Ε Ι)μικ < Κ . 3 Ι - [(Ο .ΡΒ) + (Κ.ΟΡ) - (Ο Ρ Κ)] =

π · 3 2 r; r; = -- - (2, ι 3π + 2, 1 0π - 2ν 5 ) = 0, 37π + 2ν5 cm2 2

i'ι Στο διπλανό σχήμα είναι: ΑΓ = λ4, ΑΒ = λι; και Ε6 - Ε4 = 3J3 - 4 (1). Να βρείτε το εμβαδόν του γραμμοσκιασμένου χωρίου Τ.

ΛιΊση Θα πρέπει να υπολογίσουμε την ακτίνα R του κύ­κλου . Είναι Ε = 6α6λ6 = 6 · R.J3 . R = 3R

2.J3 (2) 6 2 2 · 2 2 Επίσης Ε4 =

4α;λ4 = 2R2J2 . R J2 = 2R z (3)

Από ( 1 ) , (2), (3) έχουμε : 3R2.J3 - 2R2 = 3ν'3 - 4 <=> 3R2.J3 - 4R2 = 2(3.J3 - 4) 2 <=:>R2 (3ν'3 -4) = 2(3J3 -4) οπότε R2 = 2 <=> 1R = 21

Επειδή ΑΒ = λ6 η κεντρική γωνία ΑΟΒ = � rad 3 � Δ ι ι Άρα Τ = (Ο.ΑΒ) - (ΟΑ Β) = -R 2 · θ - - R · R · ημθ 2 2

ι 7 π 1 , π ι π ι .J3 = -R- · - - -R- · ημ- = - · 2 · - -- · 2 · -2 3 2 3 2 3 2 2 π .J3 2π - 3.J3 = - -- = 3 2 6

9 . Δίνεται το τεταρτοκύκλιο ΟΑΒΟ ακτίνας R και το ημικύκλιο C. Α ν η κάθετη στο μέσο Μ της ΟΑ τέμνει το ημικύκλιο C στο Δ και το τε­ταρτοκύκλιο στο Γ. Να βρείτε: ί) Το εμβαδόν του κυκλικού τμήματος Τ1 της χορδής ΑΓ ίί) Το εμβαδόν Τ του μεικτόγραμμου τραπεζίου ΟΒΓΔΟ . \ ί1ση

Β

i) Η ΓΜ είναι μεσοκάθετη του ΟΑ, άρα Γ Α =

ΕΥΚΛΕΙΔΗΣ Β' 75 τ.3/39

Page 43: Ευκλειδης Β 75

Μαθη ματικά για την Β ' Λυκείου

ΓΟ = R δηλαδή το τρίγωνο ΟΓ Α είναι ισόπλευρο RJ3 Δ R2J3

οπότε ΓΜ = -- και (ΓΟΑ) = -- (1) 2 4

� Δ 600 R2J3

Επομένως 'Γ.1 = (Ο.ΑΓ) - (ΓΟΑ) =-πR2 --- = 360° 4

= πR2 _ R

2 J3 (2) 6 4

ii) Το ζητούμενο εμβαδόν Τ προκύπτει αν από το τεταρτοκύκλιο ΟΑΒΟ εμβαδού

πR2 (Ο.ΑΒ) = - (3) αφαιρέσουμε: 4

• Ο εμβαδόν του τομέα ΜΟΔΜ δηλαδή

(Μ.6Δ) = 90ο . π ( R ) 2 = πR2 ( 4)

360° 2 1 6

' Δ 1 R RJ3 R2 J3 • Το εμβαδον (Μ Γ Α) = - · - · -- = --

• Το Τι 2 2 2 8

Επομένως από ( 1 ) , (2) , (3) , (4) έχουμε :

Τ = πR 2 _ πR 2 _ ( πR 3 _ R 2 J3] _ R 2 J3 = 4 1 6 6 4 8

Ι Ο . Στο διπλανό σχήμα ΓΣ είναι διάμετρος με �

ΓΣ = 2R, η γωνία φ = 3ω και το εμβαδόν του κυκλικού τομέα (Ο.ΑΣ8) = !πR2 (1) 3

Γ

Σ α) Να υπολογιστούν οι γωνίες ω, φ

β) Ν α βρεθεί ο λόγος !ι όπου τ I ' τ 2 τα γραμ­τ2 μοσκιασμένα κυκλικά τμήματα.

α) Έχουμε άρα

� � 2π � �

ω + φ = - και φ = 3ω οπότε 3

� � 2π � 2π � π � π ω + 3ω = - <=> 4ω = - άρα ω = - και φ = -3 3 6 2

� � π 5π β) Η γωνία Α ΟΓ = π - ω = π - - = -6 6

------. Δ Επομένως Τ1 = (Ο.ΑΜΓ) - (Ο Α Γ) =

1 5π 2 1 5π = - · - · R - - R · R · ημ- =

2 6 2 6

= 5π R2 _ ..!_R2 = ( 5π - 3 )R2 (2) 1 2 4 1 2

.....----. Δ 1 1 Είναι Τ, = (Ο.ΒΝΓ) - (Ο Γ Β) = -πR 2 - - · R · R = - 4 2

= ( π� 2 ) R 2 (3)

Από (2) , (3) προκύπτει ότι: Ιι = 5π - 3 Τ2 3π - 6

Ερωτήσης Σ - Λ Να κυκλώσετε το Σωστό Σ ή το Λάθος Λ των πα­ρακάτω ισχυρισμών: i) Η κεντρική γωνία ενός κανονικού ν-γώνου εί­ναι ίση με την εξωτερική γωνία του ν-γώνου . � . \ i i ) Το απόστημα ενός κανονικού ν-γώνου περιγε­γραμμένου σε κύκλο (0, R) να είναι μεγαλύτερο από την ακτίνα. Σ Λ ii i) Η γωνία ενός κανονικού ν-γώνου είναι πάντα μεγαλύτερη από την κεντρική του γωνία. Σ Λ ίν) Σε κανονικό ν-γωνο μπορεί να είναι αv=R. l: Λ ν) Σ' έναν κύκλο (0, R) μπορούμε με κανόνα και διαβήτη να εγγράψουμε κάθε κανονικό ν-γωνο.Σ Λ

νί) Ένα τρίγωνο έχει μήκη πλευρών λ3 , λ4 και RJ5 τότε το τρίγωνο είναι αμβλυγώνιο . Σ Λ νίί) Αν τόξο ακτίνας R = 2cm έχει μήκος R = 3π

' ' ξ θ ' 3π d τοτε το το ο α ειναι - ra . 2

Σ Λ νίί ί) Αν ο λόγος των εμβαδών Ε και Ε ' δύο κύ-

λ ' 1 6 ' ' λ ' κ ων ειναι - τοτε ο αντιστοιχος ογος των μη-25

' L L ' θ ' 4 κων τους και α ειναι -

5 Σ Λ

ix) Αν το εμβαδόν ενός κυκλικού τομέα κύκλου

(0, 2cm) είναι 5π cm2 τότε το μήκος του τόξου θα 4 ΕΥΚΛΕΙΔΗΣ Β' 75 τ.3/40

Page 44: Ευκλειδης Β 75

Μαθηματικά για την Β ' Λυκείου

' 5π ειναι - cm 2 Σ Λ

χ) Αν σε κύκλο (0, R) έχουμε τα σημεία του Α, � 1 � R2

Β έτσι ώστε ΑΟΒ = - rad τότε (ΟΑΒ) = - Σ Λ 2 4

Π ροη:ινό μενες Ασκ1Ίσεις για λί>ση

I . Να αποδείξετε ότι οι τομές των διαγωνίων κα­νονικού πενταγώνου, είναι επίσης κορυφές κανο­νικού πενταγώνου .

2 . Δίνεται κανονικό εξάγωνο ΑΒΓ ΔΕΖ. Α ν οι δι­αγώνιες ΑΕ και ΓΖ τέμνονται στο Ρ και η ευθεία ΒΡ τέμνει την ΕΖ στο Σ, να αποδείξετε ότι:

1 ΣΖ 1 α) ΡΖ = - ΓΖ β) - = -

4 ΣΕ 2

3 . Σε κύκλο (0, R) είναι εγγεγραμμένο κανονικό ν-γωνο με εμβαδόν Ε και περιγεγραμμένο κανονι-

, β δ ' Ε ' , , Ε 3 κο ν-γωνο με εμ α ον τετοια ωστε - = - . Ε ' 4

Να βρείτε το ν. (Απ. : ν = 6)

4. Στο διπλανό σχήμα η Γ Δ είναι παράλληλη με την ΑΒ και ΓΟΔ = 90° . (Ο είναι το κέντρο του η­μικυκλίου ΑΒ). Να αποδείξετε ότι Ε = Ε ι + Ε2 .

πR2 (Απ. : Είναι Ει = Ε2 = μ. Επίσης Τ + 2μ = --4

πR2 καθώς και Τ + Ε = -- ) 4 5. Στο σχήμα δίνονται: ΑΒ=4α, ΟΓ=2χ . Να υπο­λογιστεί το χ σε συνάρτηση του α, ώστε το γραμ­μοσκιασμένο χωρίο να έχει εμβαδόν Ε ίσο με το 1

β δ , α - του εμ α ου του κύκλου. (Απ. : χ = - ) 4 2

Α

6. Δίνεται τρίγωνο ΑΒΓ εγγεγραμμένο σε κύκλο (0, R) . Αν γ = �α2 + β2 - αβ , να υπολογίσετε το

εμβαδόν των κυκλικών τμημάτων Τ ι , Τ 2 ΠΟυ η χορδή ΑΒ χωρίζει τον κύκλο.

(Απ. Είναι συνΓ = .!. οπότε ΑΒ = λ3 = RJ3 ) 2 .

7. Δίνεται κύκλος (0, R) και η διάμετρος του ΑΒ. Γράφουμε τον κύκλο (Α, λι) . Να βρείτε το εμβαδόν του κοινού μέρους των δύο κύκλων. (Απ . : λ4 = R.Ji και το ζητούμενο εμβαδόν είναι Ε = (π - l)R2)

Γ

Δ

8. Τρεις κύκλοι με ακτίνες R 1 = J3 - \ , R 2 = J3 + 1 και R3 = 3 - .J3 εφάπτο-

νται ανά δύο εξωτερικά. Να βρείτε το εμβαδόν του καμπυλόγραμμου τριγώνου, που ορίζεται από τα σημεία επαφής των κύκλων.

(Απ. : 2J3 - π (l0 - 4J3) ) 3

Απαντήσεις στις ερωτήσεις Σ - Λ

Σ - Λ - Λ - Λ - Λ - Σ - Σ - Σ - Λ - Σ

ΕΥΚΛΕΙΔΗΣ Β' 75 τ.3/41

Page 45: Ευκλειδης Β 75

Αντ. Κυριακόπουλος - Χρυστ. Κυβερνήτου - Αθαν. Μαλαφέκας

! . l<. '\/ Κ \ ΟΣ � . n . Η εξίσωση του κύκλου C με κέντρο το σημείο

Κ( α, β) και ακτίνα ρ>Ο είναι: (χ _ α)2 + (y _ β)2 = ρ2 .

Υ ε

c Μ(Χο,�ο) \

Κ(α,β) ) ο χ

k . 2 . Η εξίσωση της εφαπτομένης ε ενός κύκλου C : (x - α)z + (y - β)z = ρz

σ' ένα σημείο του Μ(χο, Υ ο) είναι: (χ - α)(χο - α) + (y - β)(Υο - β) = ρ2 .

� . 3 . Κάθε κύκλος στο επίπεδο έχει μία εξίσωση της

ο

μορφής: χ 2 + y2 + Αχ + By + Γ

= Ο (Ι) Az + Bz

(στην οποία ισχύει: Γ

> Ο ) . 4 Λ νηστ σ Ιiφως Μία εξίσωση της μορφής ( I )

έναν κύκλο μόνο αν : παριστάνει Α2 + Bz

4 Γ

> Ο . Τότε, το κέντρο του κύκλου

. Κ( Α Β ) . . ειναι -- ,-- και η ακτινα του ειναι 2 2 �Α2 + Β2 ρ = - Γ . 4

Az + Β2 Α ν

Γ = Ο, τότε η εξίσωση (Ι) παρι-4

. . Κ( Α Β ) στανει το σημειο --,-- . 2 2

Αν Α2 + Β2 - Γ < Ο, τότε η εξίσωση (I) είναι 4

αδύνατη .

. \ l: �� li n.T i Σ 1 . Να βρείτε την εξίσωση του κύκλου που το κέντρο του είναι στην πρώτη γωνία των αξόνων και εφάπτεται στην ευθεία

ε : 3χ - 4y - 1 2 = Ο και στους άξονες χ'χ και y'y . .\ ίJ(iJ] . Ένας κύκλος με κέντρο Κ( α, β) και ακτίνα ρ είναι ζητούμενος αν, και μόνο αν, α>Ο, β>Ο και

ρ = d(K , x 'x ) = d(K , y 'y) = d(Κ, ε) �

� ρ = [β[ = [α [ = [3α -

,4β -

?1 2 [

.)3- + 4-{ ρ = α, β = α {ρ = α β = α � [3α-4β- 1 2 [ � ι ' ι � α = α + 1 2 = 5α

5 { α = 3 (αφού α > Ο) {(α + 1 2 = 5α ή α + 1 2 = -5α) � β = 3 β = α, ρ = α ρ = 3 Άρα: Κ(3 ,3) και ρ=3 . Συνεπώς, η εξίσωση του ζη­τούμενου κύκλου, είναι:

(χ - 3) 2 + (y - 3)2 = 32 . 2. Να δείξετε ότι οι κύκλοι:

C : χ2 + y 2 - 4χ - 2y + 3 = Ο και C' : χ2 + y 2 - 10χ - 4y + 21 = Ο

τέμνονται ορθογώνια, δηλαδή τέμνονται και οι εφαπτόμενες αυτών στα κοινά τους σημεία είναι κάθετες. ΛiJση , Ο κύκλος C έχει κέντρο το Κ(2, Ι ) και ακτίνα:

ρ = )Ι 6; 4 _ 3 = Jl.

Ο κύκλος c · έχει κέντρο το Λ(5,2) και ακτίνα:

R = )l 004+ 1 6 2 I = J8.

Δύο κύκλοι C(Κ,ρ) και C '(Λ,R) τέμνονται ορ­θογώνια αν, και μόνο αν :

(ΚΛ)2 = ρ2 + R 2 Έχουμε : (ΚΛ) 2 = (5 - 2)2 + (2 - 1) 2 = 1 0 = p2 + R 2 . Άρα, οι κύκλοι C και C · τέμνονται ορθογώνια . 3. Να βρείτε τις εξισώσεις των ευθειών, οι ο­ποίες διέρχονται από το σημείο M(l ,-2), τέ­μνουν τον κύκλο C : χ2 + y 2 - 14χ + 10y + 49 = Ο

ΕΥΚΛΕΙΔΗΣ Β' 73 τ.3/42

Page 46: Ευκλειδης Β 75

Μαθηματικά για την Β ' Λυκείου

σε δύο σημεία Ε και Ζ και το μήκος της χορδής 4. Θεωρούμε τον κύκλο C : (χ - 1)2 + y2 = 42 ΕΖ είναι 8. και την ευθεία ε : y = 2χ + 3.

Η εξίσωση του κύκλου C είναι της μορφής: χ 2 + y2 + Αχ + By + Γ = Ο με Α = - 1 4, Β = 1 0 και Γ = 49 . Έτσι το κέντρο του C είναι Κ(7 , -5)

και η ακτίνα του είναι:

ρ =�Α2 :Bz

- Γ = 1 42 : 1 02- 49 = 5 .

Έστω ότι μία ευθεία ε διέρχεται από το σημείο Μ και τέμνει τον κύκλο C στα σημεία Ε και Ζ.

ε / - -, � : · Η./',

Et .. �>,.\ \ C �- - \ � ι - - - .... Μ(7,-2) \ 5 Κ(Ί ,-5)

Ονομάζουμε Η την προβολή του Κ στην ε, οπότε ΕΗ=ΗΖ . Έτσι. έχουμε : (ΕΖ) = 8 <=> (ΕΗ) = 4 <=> �52 - (ΚΗ)2 = 4 <=>

<=> (ΚΗ) = 3 <=> d(Κ, ε) = 3 .

Ο ι εξισώσεις των ευθειών που διέρχονται από το Μ, είναι: χ= 1 και

y+2=λ(χ-1) <::::>λχ-y-λ-2=0 (λε!R) (1).

Η ευθεία με εξίσωση : x = l δεν τέμνει τον κύκλο C (γιατί;) και άρα δεν είναι ζητούμενη . Μία ευθεία ε : λχ-y-λ-2=0 είναι ζητούμενη αν,

και μόνο αν : l7λ + 5 - λ - 21 r:::;-: d(Κ,ε) = 3 <=> Jλ2;Ϊ = 3 <=> 12λ + 1 1 = ν λ2 + I <=> λ- + 1

ο Ί Ί ( , 4) 4λ- +4λ+ 1 =λ- + 1 <::::> 3λ- +4λ = 0 <=> λ = Ο η λ =-3

Από την ( I ) : c Με λ=Ο, βρίσκουμε:

-y - 2 = 0 <=> y = -2 (2).

ο Με λ = -� , βρίσκουμε : 3

4 4 -- x - y + - - 2 = 0 <=> 4x + 3y + 2 = 0 (3) 3 3

Άρα υπάρχουν δύο τέτοιες ευθείες. Οι εξισώ­σεις τους είναι οι (2) και (3) .

1 ) Να δείξετε ότι ο κύκλος C και η ευθεία ε δεν έχουν κοινό σημείο. 2) Από ένα σημείο Μ της ευθείας ε φέρνουμε τις εφαπτόμενες στον κύκλο C και ονομάζουμε Α και Β τα σημεία επαφής. Να δείξετε ότι, όταν το σημείο Μ διαγράφει την ευθεία ε, η ευθεία ΑΒ διέρχεται από ένα σταθερό σημείο.

Οι συντεταγμένες των κοινών σημείων του κύ­κλου C και της ευθείας ε, αν έχουν, είναι οι λύσεις του συστήματος: { (χ - 1)2 + y2 = 4

<=>{ (χ - 1)2 + (2χ + 3)2 = 4

<=> y = 2χ + 3 y = 2χ + 3

<=>{5x 2 + l 0x + 6 = 0 ( 1 )

y = 2 χ + 3

Η διακρίνουσα της εξίσωσης ( 1 ) είναι Δ=-20<0. Άρα, το σύστημα είναι αδύνατο και συνεπώς ο κύ­κλος C και η ευθεία ε δεν έχουν κοινό σημείο . 2. ) Έστω ότι (λ, μ) είναι οι συντεταγμένες ενός ση­μείου Μ της ε, οπότε μ = 2λ + 3 και άρα: Μ(λ, 2λ + 3) . Έστω ακόμα ότι Α(χ 1 , y 1 ) και Β(χυ y2 ) . Η εξίσωση της ευθείας ΜΑ, είναι: (χ - l )(x 1 - I) + yy1 = 4 και επειδή αυτή διέρχεται

από το Μ, έχουμε: (λ - Ι)(χ 1 - 1) + (2λ +3)y1 = 4. (2)

Όμοια βρίσκουμε ότι: (λ - 1)(χ 2 - 1) + (2λ + 3)y2 = 4 . (3)

Θεωρούμε τώρα την ευθεία με εξίσωση : (λ - Ι )(χ - 1) + (2λ + 3)y = 4. (4)

Η ευθεία αυτή, λόγω της (2) διέρχεται από το Α και λόγω της (3) διέρχεται από το Β . Άρα, η εξί­σωση της ευθείας ΑΒ είναι η (4). Έχουμε:

(4) <=> λ(x + 2y - l ) - x + 3y - 3 = 0.

Έτσι, η ευθεία ΑΒ διέρχεται από το σημείο του οποίου οι συντεταγμένες είναι λύση του συστήμα­τος:

{ x + 2y - 1 = 0 { x + 2y = l {χ = -� -χ + 3y - 3 = Ο

<=> -χ + 3y = 3

<=> Υ = �

Άρα, η ευθεία ΑΒ διέρχεται από το (σταθερό) ση-

ΕΥΚΛΕΙΔΗΣ Β' 73 τ.3/43

Page 47: Ευκλειδης Β 75

Μαθηματικά για την Β ' Λυκείου

. ( 3 4 ) μειο -5 '5 .

5. Θεωρούμε την ευθεία ε : 2χ - y - 3 = Ο και τον κύκλο C : χ2 + y2 - χ + y - 2 = Ο. α) Να δείξετε ότι η ευθεία ε και ο κύκλος C τέ­μνονται. Ονομάζουμε Μ και Ν τα σημεία τομής. β) Να δείξετε ότι η εξίσωση :

χ2 + y2 - χ + y - 2 + λ(2χ - y - 3) = Ο (1) για Κάθε λ Ε JR, παριστάνει ένα ΚύΚλΟ Cλ , Ο Ο-ποίος διέρχεται από τα σημεία Μ και Ν. γ) Να βρείτε το γεωμετρικό τόπο των κέντρων των κύκλων Cλ , όταν το λ διατρέχει το JR •

Λί)ση

α) Λύνουμε το σύστημα των εξισώσεων των γραμμών ε και C : { 2x-y-3=0 { y=2x-3

χ2 +i -x+y-2=0 <=>

χ2 +(2χ-3)2 -χ+2χ-3-2=0 { y = 2x - 3 <=> 5χ 2 - 1 1χ + 4 = 0

Το σύστημα αυτό έχει δύο λύσεις, γιατί η διακρί­νουσα της εξίσωσης: 5 χ 2 - 1 1 χ + 4 = Ο είναι

Δ = 1 12 - 4 · 5 · 4 = 4 1 > 0 . Άρα, η ευθεία ε και ο κύκλος C τέμνονται. β) Έχουμε: (1) <=> χ2 + y2 + (2λ - 1)χ - (λ - 1)y - (3y + 2) = 0 (2) Αυτή είναι της μορφής : χ 2 + y2 + Αχ + By + Γ = Ο με : Α = 2λ - 1 , Β = -(λ - 1) και Γ = -(3λ + 2) . Για κάθε λ ε JR, έχουμε: Α2 +Er

-Γ (2λ-1)2 +(λ-1)2 +3λ+2 5λ2 +6λ+10 >Ο

4 4 4 ,

γιατί η διακρίνουσα του τριωνύμου : 5λ 2 + 6λ + 1 Ο είναι αρνητική . Άρα, για κάθε λ ε JR, η εξίσωση (2), επομένως και η ( 1 ) παριστάνει ένα κύκλο cλ .

Επειδή τα σημεία Μ και Ν είναι σημεία και των δύο γραμμών ε και C, οι συντεταγμένες τους επαληθεύουν τις εξισώσεις και των δύο αυτών γραμμών και άρα επαληθεύουν και την εξίσωση ( 1 ), για κάθε λ ε JR.

Άρα, όλοι οι κύκλοι Cλ διέρχονται από τα ση-

μεία Μ και Ν. γ) Το κέντρο ενός κύκλου Cλ είναι:

(_ Α _ Β )=(-� �)

. 2

, 2 2

, 2

Ένα σημείο M(x,y) ανήκει στο ζητούμενο γεωμε­τρικό τόπο αν, και μόνο αν, υπάρχει λ ε JR με: {

χ = -2\- 1 {2χ = -2λ + 1 { λ = 2y + 1

Υ = λ� 1 <=>

2y = λ - 1 <=>

2χ = -2λ + Ι

Αυτό συμβαίνει αν, και μόνο αν : 2χ = -2(2y + 1) + 1 <=> 2χ + 4y + 1 = ο .

Άρα, ο ζητούμενος γεωμετρικός τόπος είναι η ευ­θεία με εξίσωση : 2χ + 4y + 1 = Ο .

2 . Π Α ΡΆ ΒΟΛ Η

2 . 1 . Ορισμός Σ' ένα επίπεδο θεωρούμε μία ευθεία δ και ένα σημείο Ε εκτός της δ. Ο γεωμετρικός τό­πος των σημείων του επιπέδου αυτού, τα οποία ισαπέχουν από το Ε και τη δ, ονομάζεται παραβο­λή με εστία το σημείο Ε και διευθετούσα την ευ­θεία δ .

δ

Ρ χ=- -2

Υ c

Ε( f ,ο) 2

χ

2 .2 Η εξίσωση της παραβολής C με εστία Ε(Ε. , ο) 2

και διευθετούσα δ : χ = _Ε. (p =F- Ο) , είναι : 2

y2 = 2px.

• Η εξίσωση της παραβολής με εστία Ε(Ο,Ε.) 2

και διευθετούσα δ : y = _Ε. (p =F- Ο) , είναι 2

χ 2 = 2py. 2 .3 Η εφαπτομένη της παραβολής C : y2 = 2px στο σημείο της Α(χο , Υο ) έχει εξίσωση :

ΥΥο = p(x + X0 ) . • Η εφαπτομένη της παραβολής C : χ 2 = 2py

στο σημείο της Α( χ ο , y ο ) έχει εξίσωση :

ΧΧο = p(y + yo ) .

ΕΥΚΛΕΙΔΗΣ Β' 73 τ.3/44

Page 48: Ευκλειδης Β 75

Μαθηματικά για την Β ' Λυκείου

Α Σ Κ Η 2: Ε ΙΣ

1 . Να βρείτε την εξίσωση της παραβολής C που έχει κορυφή την αρχή των αξόνων, την ε­στία της στον άξονα χ · χ και εφάπτεται στην ευ­θεία ε : y = 2χ + 1 . Λ\>ση

Η εξίσωση της C είναι της μορφής: / = 2 px . Η

εφαπτομένη της C στην κορυφή της 0(0,0) είναι η ευθεία y 'y, διαφορετική από την ε.

Έστω Μ ( χ ο , y ο ) ένα σημείο της C διαφο-ρετικό από την κορυφή της, οπότε χ ο y ο :f:. Ο και

Υ ο� = 2pX0 • ( l )

Η εξίσωση της εφαπτομένης της C στο Μ είναι: ( I ) 2

ΥΥο = p(x + Χο )<=>ΥΥο = p(x +�) <=> Υ =_Ε_ Χ +�. 2p Υο 2

Έτσι, η ε εφάπτεται στη C αν, και μόνο αν, υπάρ­χει Υ ο Ε IR με:

Υ Υ = 2 ο <=> ο !_Ε_ = 2

Υ; � ι {p � 2y,

Αυτό συμβαίνει αν, και μόνο αν : p = 2 · 2 <=> p = 4.

Άρα, η ζητούμενη παραβολή είναι C : y2 = 8χ.

2. Θεωρούμε την παραβολή C : y z = 4χ και την ευθεία ε : 4χ - 3y + 1 2 = Ο . Να βρείτε τις συντεταγμένες του σημείου της C, του οποίου η απόσταση από την ευθεία ε είναι η ελάχιστη δυνατή. Ποια είναι η ελάχιστη αυτή απόσταση ; λi>ση Η απόσταση d ενός σημείου M(x,y) της C από την ε, είναι:

l 4x - 3y + 1 2 l Il I d = ι = - 4x - 3y + 1 2 .

ν 42 + 32 5

'Εχουμε y2 = 4χ και συνεπώς χ = ..!_ y2 . 'Ετσι, έ-4

χουμε : 1 1 1 2 I 1 1 2 I d = 5 44 y - 3y + 1 2 = 5 y - 3y + 1 2 ,

όπου y Ε IR. Το τριώνυμο y2 - 3y + 1 2 έχει Διακρίνουσα αρνη­

τική και συνεπώς y2 - 3 y + 1 2 > Ο , για κά-

1 2 θε y ε JR. . Έτσι έχουμε : d = - (y - 3y + 12) . Το 5

τριώνυμο : / - 3 y + 1 2 έχει ελάχιστη τιμή για 3 . 1 ? 1 9 9 y = - . Τοτε: x = - y- = - · - = - . 2 4 4 4 1 6

Άρα, οι συντεταγμένες του ζητούμενου σημείου . ( 9 3 ) Η . . .

ει ναι - ,- . υποψη αποσταση ειναι: 1 6 2

Ι 9 3 39 d ιηin = 5 (4 - 3 ·}2 + 1 2) = 20

.

3. Έστω μία παραβολή C : yz = 2px (p>O) με εστία Ε και ένα σημείο της Μ. Να δείξετε ότι ο κύκλος c · με διάμετρο το ευθ. τμήμα ΕΜ εφά­πτεται στον άξονα y 'y. Λ\>ση Έστω M(x0 , y0 ) , οπότε: ( 1 )

Α c ·

χ

Επειδή Ε(Ε. , ο) , οι συντεταγμένες του μέσου, έ-2

στω Κ, του ευθύγραμμου τμήματος ΕΜ, είναι:

( χ2ο +� , �ο ) . Έτσι, έχουμε:

(ΚΜ)2 = (ΚΕ)2 = ( � +�-Ξ)2 +(; J2 = ( � -�J2 + � ( I ) Χ� px o p2 2pxo Χ� pxo p2 =---+ - + -- = - + - + - =

4 4 1 6 4 4 4 1 6

= ( χ20 + � J 2 => (ΚΜ)=(ΚΕ) = I� +� I = � +� ( 1 ) ,

γιατί p > Ο και Χ 0 � Ο.

Έστω τώρα Α η προβολή του Κ στον άξονα y 'y. Έχουμε: Α(Ο, 2'2..) και συνεπώς:

2

(ΚΑ) = ��+E. I = �+Ε. �(ΚΜ) = (ΚΕ). 2 4 2 4

Άρα, ο κύκλος C · εφάπτεται στον άξονα y · y.

ΕΥΚΛΕΙΔΗΣ Β' 73 τ.3/45

Page 49: Ευκλειδης Β 75

Μ αθηματικά για την Β ' Λυκείου

4. Να βρείτε τις εξισώσεις των εφαπτομένων και σταθερό άθροισμα 2α . της παραβολής C : y 2 = 8χ, οι οποίες σχηματί- y ζουν με την ευθεία ε : 3χ - y + 5 = Ο γωνία 45° .

. \ i : <Ό\1 Έστω Μ ( χ ο , y ο ) ένα σημείο της παραβολής C, οπότε :

Υ� = 8χο ( 1 )

Η εξίσωση της εφαπτομένης ε ' της C στο Μ , είναι: ΥΥο = 4(χ + Χο ) <=> 4χ - ΥΥο + 4Χο = Ο (2) .

Γνωρίζουμε ότι: δ = ( 1 , 3 ) 1 1 ε και � = (y0 , 4) 1 1 ε ' . Λ

Θέτουμε : θ = (δ:�) , οπότε: Ο � θ :::; 1 80° . Οι ευ-

θείες ε και ε ' σχηματίζουν γωνία 45° αν, και μόνο αν : (θ = 45° ή θ = 1 80° - 45° ) <::::> <=> ( συνθ = συν45° ή συνθ = -συν45° )

<=> jσυνθj = J2 <=> �� = J2 <=> I Yo + I 2j =-Ι

2 ι δι ι�ι 2 JW )y� + 16 J2 ( y0 + I 2 )2 1 2 <=> ? = - <=> Υ - 6y - Ι 6 = Ο <::::> I O · (y� + Ι 6) 2 ο ο

<=> (Υο = 8 ή Υο = -2 ) . Από την ( 1 ) με Υο = 8 βρίσκουμε Χ0 = 8 και με

1 Υ ο = -2 βρίσκουμε Χο =2 .

Από τη (2) : Με Χο = 8 και Υο = 8 βρίσκουμε :

4x-8y+4 · 8=0<::::>x -2y+8=0 (3) .

1 Με Χο =2 και Υο =-2 , βρίσκουμε :

4x +2y+2 = 0 G2x + y + l =0 (4) .

Άρα, υπάρχουν δύο τέτοιες εφαπτόμενες. Οι εξι­σώσεις τους είναι οι (3) και ( 4 ) .

ι . : . � Θεωρούμε ένα αριθμό α>Ο και σ ' ένα επίπεδο δύο σημεία Ε ' και Ε με (ΕΈ) < 2α . Ο

γεωμετρικός τόπος των σημείων Μ του επιπέδου αυτού, για τα οποία ισχύει: (ΜΕ ') + (ΜΕ) = 2α , ονομάζεται έλλειψη με εστίες τα σημεία Ε ' και Ε

Α '/ _ - " "

Ε ' Ο I Ε / χ

. L . Η εξίσωση της έλλειψης με εστίες Ε '( -γ, Ο), Ε(γ, Ο) (γ 2 Ο) και σταθερό άθροισμα 2α (προφανώς γ<α ) είναι:

2 2 χ Υ , β � 2 2 -, + -7 = 1 , οπου = α - γ . α- β-Ισχύουν : α z β και γ = Jα2 - β2 •

Η εξίσωση της Ε '(Ο, -γ), Ε(Ο, γ)

έλλειψης με εστίες και σταθερό άθροισμα

2α είναι: ? ? χ- y- 1 ' β � 2 2 -, + -, = , οπου = α - γ . β- α-

χ 2 y2 Εκκεντρότητα μιας έλλειψης C : -, + -, = Ι , α- β-ονομάζεται ο αριθμός e = 1_ . α

Ισχύουν: Ο � e < Ι και l = .J1 - e2 • α ? ?

· ; · ' Η εφαπτομένη της έλλειψης C : χ� + Υ: = Ι σ' α- β-ένα σημείο της Ρ(χο , Υο ) έχει εξίσωση :

χχο + ΥΥσ = I α2 β2 ·

? ? ο Η εφαπτομένη της έλλειψης C : χ: + Υ: = 1 σ ' β- α-

ένα σημείο της P(x0 , y0 ) έχει εξίσωση :

1 . θεωρούμε χ2 y 2

C : -2 + -2 = 1 α β

ΧΧο + ΥΥο = 1 βz α2 ·

μία έλλειψη ( α>β>Ο) και ένα σημείο της Ρ

διαφορετικό από τις κορυφές Β και Β Ό Οι ευ­θείες ΡΒ και ΡΒ ' τέμνουν τον άξονα χ ' χ στα ση­μεία Μ και Ν, αντιστοίχως. Να δείξετε ότι, όταν το Ρ διαγράφει τη C (εκτός των κορυφών Β και Β '), το γινόμενο (ΟΜ) · (ΟΝ) είναι σταθερό.

ΕΥΚΛΕΙΔΗΣ Β' 73 τ.3/46

Page 50: Ευκλειδης Β 75

Μαθη ματικά για την Β ' Λυκείου

( I )

Τ ε · Μ χ

Β '

Επειδή Β(Ο, β) και Β '(Ο, -β) , έχουμε: Υο - β _ Υο + β λ ΡΒ = -- Και λ ΡΒ' --- .

χο χο Η εξίσωση της ευθείας ΡΒ είναι:

Υ - β = Υ ο - β χ . (2) χ ο

Η εξίσωση της ευθείας ΡΒ · είναι :

Υ + β = Υ ο + β χ . (3) χ ο

-β χ Από τη (2) με y = Ο , έχουμε: χ = __ ο . Υο - β

-β χ Άρα: Μ (--'' . 0 ) . Υ,, - β

Από την (3) με y = Ο . έχουμε : χ = � . Υο + β

Άρα: Ν(� , Ο) . Υο + β

Έτσι έχουμε : ' ' -βχ βχ β-χ-(ΟΜ) · (ΟΝ) = __ ο · _ο_ = , σ ,

Υσ - β Υο + β Υ� - β-

( I ) Χ 2 ο 2 ( θ ' ) = --7 = α στα ερο .

-χ� αz

2. Από το σημείο Μ( -3, 4) φέρνουμε τις εφα­πτόμενες της παραβολής C : y2 = 4χ και ονομά­ζουμε Κ και Λ τα σημεία επαφής. Να δείξετε ότι η ευθεία ΚΛ εφάπτεται στην έλλειψη

χ2 2 2 C' : - + _r_ = l . 3 3

. \ ίJση Η εξίσωση της ευθείας ΚΛ, όπως γνωρίζουμε, εί­ναι:

y · 4 = 2(χ - 3) <=> χ - 2y - 3 = Ο (I ) . Βρίσκουμε τα κοινά σημεία της ευθείας ΚΛ και της έλλειψης C' . Προς τούτο, λύνουμε το σύστη-!χ-2y-3=0i y=� (x-3) 1 y=�(x-3) μα: xz 2y 2 - +- =1 7 1 ? 3 ' 2 Ι Ο 3 3 χ- +24(χ-3)- = χ- - χ+ =

<=> { x = l y = - 1

Άρα, η ευθεία ΚΛ και η έλλειψη C' έχουν ένα μο­ναδικό κοινό σημείο, το Ρ( Ι , - 1 ) . Η εξίσωση της εφαπτομένης της C' στο σημείο της Ρ( 1 , - 1 ) , είναι:

� + 2 y · (- 1) = Ι <=> χ - 2y - 3 = Ο (2) . 3 3

Η (2) όμως είναι η εξίσωση ( 1 ) της ευθείας ΚΛ .

Άρα, η ευθεία ΚΛ εφάπτεται στην έλλειψη C' .

3. Να βρείτε την εξίσωση της έλλειψης C που έχει εστίες E'(-J6,0), E(J6, 0) και εφάπτεται στην ευθεία ε : χ + 2y - 4 = Ο .

ι 1)-ση Έστω ότι μία έλλειψη C πληροί τις δοσμένες συν­θήκες. Η εξίσωση της C θα είναι της μορφής:

' ?

:� + �: = 1 (α>β>Ο) . ( 1 )

Θα έχουμε γ = J6 , οπότε : α2 - β2 = 6 . (2)

Έστω ότι η C εφάπτεται στην ευθεία ε στο σημείο M(X0 , y0 ) , οπότε:

2 2 � + � = ] αz β z .

Έτσι, η εξίσωση της ε είναι:

(3)

χχ,ο + Yf,o = 1 . (4) α- β-

Άρα, η εξίσωση : χ + 2y - 4 = Ο και η εξίσωση ( 4)

παριστάνουν την ίδια ευθεία ε. Α ν y ο = Ο , τότε το

προηγούμενο προφανώς δεν συμβαίνει. Άρα y ο =f:. Ο . Έτσι, έχουμε:

ΕΥΚΛΕΙΔΗΣ Β' 73 τ.3/47

Page 51: Ευκλειδης Β 75

Μαθη ματικά για την Β ' Λυκείου

Αντικαθιστώντας στην (3) βρίσκουμε:

_;_[�)2 + �[f_) 2 = 1 <=:> α2 + 4β2 = 1 6 (5) . α- 4 β- 2

Από τις (2) και (5) βρίσκουμε : α2 = 8 και β2 = 2 .

Αντικαθιστώντας στην ( 1 ) βρίσκουμε ότι η εξίσω­χ 2 y2

ση της C είναι: - + - = 1 . 8 2

Λ ντιστρόφως

Δείχνουμε (όπως στην προηγούμενη άσκηση) ότι η έλλειψη

χ 2 y2 C : - + - = 1

8 2 εφάπτεται στην ευθεία

ε : χ + 2y - 4 = 0

Άρα, η μοναδική ζητούμενη έλλειψη είναι η χ 2 y2

C : - + - = 1 .

4. Θεωρούμε

8 2

χ2 y2 C : -2 + -2 = 1 (α>β>Ο) α β

μία έλλειψη και τις εστίες της

Ε'(-γ, Ο) και Ε(γ , Ο) . Η κάθετη στον άξονα χ'χ από την εστία Ε, τέμνει τη C στο σημείο Μ. Η εφαπτομένη ε της C στο σημείο της Μ, τέμνει τον άξονα y'y στο σημείο Ν. Να δείξετε ότι η κάθετη ε' της ε στο Ν διέρχεται από την εστία Ε '. Λ\J ση Η τετμημένη του Μ είναι γ. Έστω Υ ο η τεταγμένη

του . Επειδή Μ ε C , έχουμε :

2 2 α2 _ 2 2 β4 1_2 + Υ2

ο = 1 � Υ2 = β2 2 γ � Υ ο = -2 ( 1 ) . α β ο α α

χ

Β '

Η εξίσωση της εφαπτομένης ε, είναι:

!]__ + yy ο = 1 Από αυτή με χ=Ο βρίσκουμε: α2 β2 ·

β' β2 Υ = -

-. Άρα: Ν(Ο,-) .

Υο Υο β ? ?

'Ε λ -γ , λ

α-yο χουμε : ε = --2 - και αρα ε' = -, - . α Υο β-γ

β ' ? Έτσι, έχουμε ε' : y - -

-=

α -,Υ ο χ . (2) .

Υο β-γ

Τώρα, αρκεί να δείξουμε ότι η εξίσωση (2) επαλη­θεύεται από τις συντεταγμένες (-γ, Ο) του Ε' . Δηλαδή , αρκεί να δείξουμε ότι:

β 2 αyο ( ) δ λ δ , , -- = -- -γ , η α η αρκει : Υο β2γ

ισχύει λόγω της ( 1 ) .

4 . V ll E P BOΛ H

4. 1 Ορισμός : Θεωρούμε ένα αριθμό α>Ο και σ ' ένα επίπεδο δύο σημεία Ε ' και Ε με (ΕΈ) > 2α . Ο γεωμετρικός τόπος των ση-

μείων Μ του επιπέδου αυτού, για τα οποία ισχύει: I CME ') - (ME)I = 2α , ονομάζεται υ-

περβολή με εστίες τα σημεία Ε ' και Ε και σταθερή απόλυτη διαφορά 2α .

Υ Μ

Ε ' χ

4.2 Η εξίσωση της υπερβολής με εστίες Ε '( -γ, Ο), Ε(γ, Ο) (γ>Ο) και σταθερή απόλυ-

τη διαφορά 2α (προφανώς γ>α) είναι:

ΕΥΚΛΕΙΔΗΣ Β' 73 τ.3/48

Page 52: Ευκλειδης Β 75

Μαθηματικά για την Β ' Λυκείου

4.3

χ2 y2 r---- -- = I , όπου β = �γ2 - α2 • αz βz

Ισχύουν: γ > β και γ = �α2 + β2 . • Η εξίσωση της υπερβολής με εστίες Ε '(Ο, -γ), Ε(Ο, γ) και σταθερή απόλυτη δια-φορά 2α είναι:

2 2 Υ χ • Ι 2 2 -0 - -0 = Ι , οπου β = ν γ - α . α- β-

Οι aσύμπτωτες της υπερβολής χ2 y2 C : - --0 = Ι είναι οι ευθείες με εξισώ­α2 β-

σεις: y = Ι χ και y = -Ι χ . α α 2 2

Οι aσύμπτωτες της υπερβολής C : �2 - ;2 = Ι

είναι οι ευθείες με εξίσωση : y = Ι χ και α y = -I x . α

χ2 y2 ..ι ...ι Εκκεντρότητα μιας υπερβολής C : -2 - -0 = Ι α β-

ονομάζεται ο αριθμός: e = l . α Ισχύουν: e > I και Ι = � . α

4.5 . Η εφαπτομένη της υπερβολής χ 2 /

C : -0 --2 = Ι σ' ένα σημείο της P(x0 , y0 ) α- β έχει εξίσωση : χχ

0° - �ο = Ι . α- β-? ?

• Η εφαπτομένη της υπερβολής C : Υ� - χ: = 1 α- β-σ ' ένα σημείο της P(x0 , y0 ) έχει εξίσωση :

Λl: Κ Η Σ Ε Ι Σ

ΥΥο _ χχο = I α 2 β2 .

1 . Να βρείτε τις υπερβολές που έχουν κέντρο την αρχή των αξόνων, τις εστίες τους σε έναν από τους άξονες των συντεταγμένων, aσύμπτω­τες y = ±J3x και διέρχονται από το σημείο Μ(2,3). ΛίJση

α) Έστω ότι μία τέτοια υπερβολή έχει τις εστίες

της στον άξονα χ 'χ . Τότε θα έχει μία εξίσωση της ' χ 2 y2

μορφης: - - - = I ( α>Ο, β>Ο) . ( 1 ) α2 βz

Οι δοσμένες συνθήκες πληρούνται αν, και μόνο

αν: ��� :� I α2 β2 Αντικαθιστώντας στην ( Ι ) βρίσκουμε :

2 x 2 - L = I . (2) 3

β) Έστω ότι μία τέτοια υπερβολή έχει τις εστίες της στον άξονα y'y . Τότε θα έχει μία εξίσωση της / χ 2 μορφής - - - = 1 α2 β2 .

Οι δοσμένες συνθήκες πληρούνται αν, και μόνο αν:

<=> 9 4 <=> β αδύνατο. { α = β/3 {α =

fj

3β2 - β2 = Ι β2 = -1

- Άρα, υπάρχει μία μόνο τέτοια υπερβολή και η εξίσωσή της είναι η (2).

2. Ν α δείξετε ότι υπάρχουν δύο εφαπτόμενες 2

της υπερβολής C : χ2 - L = 1 , οι οποίες είναι 3 κάθετες στην ευθεία ε : χ + 2y - 7 = Ο . Μετά, να βρείτε τη μεταξύ τους απόσταση. ΛίJση Έστω Ρ( χ ο , y ο ) ένα σημείο της υπερβολής C, οπό-

τε: ( 1 )

Η εξίσωση της εφαπτομένης ε' της C στο Ρ , είναι: ΧΧ0 - ΥΥο = I <::::> 3ΧΧ 0 - ΥΥο - 3 = 0 (2) . 3

Γνωρίζουμε ότι: δ = (2,- Ι) 1 1 ε και δ' = (y0 , 3Χ0 ) 1 1 εΌ Έτσι, έχουμε: ε l_ ε' <=> δ' l_ δ <=> δ' ο δ = ο <=> 2yo - 3χο = ο (3) . Λύνουμε το σύστημα των εξισώσεων ( 1 ) και (3) και βρίσκουμε τις λύσεις : ( Χ 0 = 2, y ο = 3 ) και ( χο = -2, y0 = -3 ) . Αντικαθιστώντας στη (2) και aπλοποιώντας, βρίσκουμε αντιστοίχως τι; ε�ισc:)-

ΕΥΚΛΕΙΔΗΣ Β' 73 τ.3/49

Page 53: Ευκλειδης Β 75

Μαθηματικά για την Β ' Λυκείου

σεις : 2x - y - 1 = 0 (4) και 2x - y + l = O (5) . Άρα, υπάρχουν δύο τέτοιες εφαπτόμενες. Οι εξι­σώσεις τους είναι οι (4) και (5) . Τις ονομάζουμε ε 1 και ε2 , αντιστοίχως. Από την ( 4) με χ=Ο βρίσκουμε y = -I . Άρα, το σημείο Μ(Ο,- 1 ) ανήκει στην ε1 • Συνεπώς:

1 2 · 0 - (- I) + I i 2 2JS d(ε 1 , ε ? ) = d(Μ, ε? ) = � = r; = -- - ν 4 + 1 ν 5 5

χ2 y 2 3. Έστω μία υπερβολή C : -2 - -2 = 1 . Ονο-α β μάζουμε Κ την προβολή μιας εστίας της C σε μία aσύμπτωτη αυτής. Να δείξετε ότι η από­σταση της αρχής των αξόνων Ο από το Κ, είναι ίση με την απόσταση του Ο από μία κορυφή της c.

Έστω ότι Κ είναι η προβολή της εστίας Ε(γ,Ο) στην ασύμπτωτη ζ : y = Ι χ .

β y= α χ

α

'Εχουμε : y = Ι χ <:::> βχ - αy = Ο . α

ζ

'Ετσι, έχουμε : (ΕΚ) = d(Ε, ζ) = �γ Ι ? = βγ

? = β . �β- + α- Ν Από το ορθογώνιο τρίγωνο ΟΚΕ έχουμε :

(ΟΚ)2 = (ΟΕ)2 - (ΕΚ)2 = γ2 - β2 = α2 => (ΟΚ) = α = (ΟΑ) = (ΟΑ') .

Όμοια, αν το Κ είναι προβολή της Ε στην ασύ­μπτωτη ζ' : y = _Ι χ ή αν το Κ είναι προβολή της α εστίας Ε '(-γ, Ο) σε μία από τις ασύμπτωτες ζ ή ζΌ 4 . Η εκκεντρότητα μιας υπερβολής

χ2 y2 C : -- - = 1 (α>Ο, β>Ο) είναι η e = 2 . Να α2 β 2

βρείτε τη γωνία των ασυμπτώτων της C, στην οποία βρίσκεται ο δεξιός κλάδος της.

Οι ασύμπτωτες της C είναι ε 1 : y = Ι χ και α ε2 : y = _Ι χ . Η εφαπτομένη της C στην κορυφή α Α(α,Ο) έχει εξίσωση : χ = α . Η εφαπτομένη αυτή, όπως βρίσκουμε εύκολα, τέμνει την ασύμπτωτη ε 1 στο σημείο Γ( α, β) και την ασύμπτωτη ε2 στο ση-μείο Δ( α, -β) . Η γωνία των ασυμπτώτων της C, στην οποία βρί­σκεται ο δεξιός κλάδος της, είναι η γωνία θ των διανυσμάτων: ΟΓ = (α, β) και ΟΔ = (α, -β) . Έχουμε :

- -ΟΓ · ΟΔ συνθ ΙοrΙ · ΙοΔΙ

, ? ? ? 2α2 - γ2 (γιατι : β- = γ- - α- ) = ?

2 - e2 2 - 4 I -- = -- = --

e2 4 2 Άρα:

γ-

Ε χ

β y= - α

1 2π 2π ' συνθ = -- <:::>συνθ =συν- <::>θ=- (γιατι Ο ::;θ ::;π). 2 3 3

1 . Θεωρούμε δύο θετικούς αριθμούς α και β. Να δείξετε ότι, για κάθε ν ε Ν * , ισχύει:

να ν+ I + β ν+ I � α ν β( ν + 1) • (1)

ΕΥΚΛΕΙΔΗΣ Β' 73 τ.3/50

Page 54: Ευκλειδης Β 75

Μαθηματικά για την Β ' Λυκείου

Επαγωγικά. "' Για ν= Ι , η ( Ι ) γίνεται: α 2 + β2 � 2αβ . Για να δείξουμε αυτή , αρκεί να

δείξουμε ότι: α2 + β2 - 2αβ � Ο , αρκεί:

(α - β γ � Ο , ισχύει.

Έστω ότι η ( Ι ) ισχύει για ν = κ (κ Ε Ν* ) , δηλ.

ότι; Κακ+ Ι + βκ+ Ι � α κβ(Κ + 1 ) . (2)

Θα δείξουμε ότι ισχύει και για ν = κ + 1 , δηλ. ότι: (κ + Ι )α Κ+ 2 + βκ+l � α'+ 1 β(κ + 2) .

Προς τούτο, αρκεί να δείξουμε ότι: βΚ+2 � ακ+ Ι β(κ + 2) - (κ + Ι )α κ+l (3 )

Από τη (2 ) έπεται: βΚ+2 � ακβ2 (κ + Ι) - κακ+ Ι β . Έτσι, για να δείξουμε την (3) , αρκεί να δείξουμε ότι : ακβ2 (κ + ι) - κακ+Ιβ � ακ+Ιβ(κ + 2) - (κ + ι)α'+2 ' αρκεί : β2 (κ + Ι) - καβ - αβ(κ + 2) + (κ + Ι )α2 � 0 , αρκεί : β2 (κ + I) - 2αβ(κ + I ) + (κ + Ι)α2 � Ο , αρκεί: (κ + I )(α - β)2 � Ο , ισχύει.

2. Ο αριθμός α2 + 2β , όπου α, β Ε Ζ , είναι τε­τράγωνο ακέραιου αριθμού. Να δείξετε ότι ο αριθμός α2 + β είναι άθροισμα δύο τετραγώνων ακέραιων αριθμών.

Έστω ότι α2 + 2β = κ2 , όπου κ Ε Ζ . Έχουμε

κ2 - α2 = 2β και άρα ο αριθμός κ 2 - α2 είναι άρ­

τιος. Α ν ο ένας από τους αριθμούς κ και α είναι άρτιος και ο άλλος περιττός, τότε ο κ 2 - α2 είναι περιττός, άτοπο . Άρα, οι αριθμοί κ και α είναι και οι δύο άρτιοι ή και οι δύο περιττοί. Συνεπώς, ο α­ριθμός κ - α είναι άρτιος. Θέτουμε κ - α = 2λ , όπου λ Ε Ζ , οπότε κ = α + 2λ . Έτσι, έχουμε : α2 + 2β = κ2 => α2 + 2β = (α + 2λ)2 => => α 2 + 2β = α 2 + 4αλ + 4λ 2 => 2β = 4αλ + 4λ 2 => β = 2αλ + 2λ 2 => => α 2 + β = α 2 + 2αλ + λ 2 + λ 2 => α2 + β = (α + λ)2 + λ2 •

3. Να βρείτε τους φυσικούς αριθμούς χ, y και ω , για τους οποίους ισχύουν:

{χ + 5y + 1 15ω = 933 χ < 5, y < 23

Έχουμε : 933 = 5(y + 23ω) + χ . ( Ι ) Επειδή Ο � χ < 5 , η ( Ι ) είναι η ισότητα της διαίρε­σης 933 : 5 . Εκτελούμε τη διαίρεση αυτή και βρί­σκουμε πηλίκο Ι 86 και υπόλοιπο 3 . Άρα χ = 3 και y + 23ω = ι 86 , οπότε 1 86 = 23ω + y (2) . Επειδή Ο � y < 23 , η (2) είναι η ισότητα της διαί­ρεσης ι 86 :23 , οπότε y = 2 και ω = 8 . Άρα, χ = 3 , y = 2 και ω = 8 . Όπως βρίσκουμε εύ-

κολα οι αριθμοί αυτοί επαληθεύουν τις δοσμένες σχέσεις και άρα είναι οι μοναδικοί ζητούμενοι.

4. Να βρείτε τους φυσικούς θετικούς αριθμούς α και β για τους οποίους ισχύει:

(7α + 3β - 16)(1 7β + 1 5) = 32. (1 )

Έστω ότι για δύο αριθμούς α, β Ε Ν * η ( Ι ) ισχύει. Τότε : Ι Ίβ + I 5 l32=> ι Ίβ+ Ι 5 � 32 =>ΙΊβ � Ι Ί =>β� Ι και επειδή β � ι , έπεται ότι β = Ι . Αντικαθιστώντας στην ( Ι ) , βρίσκουμε : (Ία + 3β - 1 6)(1 Ί + 1 5) = 32 => Ία - 1 3 = I => α = 2 . Άρα, τότε α = 2 και β = Ι .

Με α = 2 και β = Ι βρίσκουμε εύ­κολα ότι η ( Ι ) ισχύει. Άρα: α = 2 και β = ι .

ΕΥΚΛΕΙΔΗΣ Β' 73 τ.3/51

Page 55: Ευκλειδης Β 75

Μαθηματ ι κά Γεν ι κής Πα ιδε ίας

γ ια τη Γ ' τάξη του Λυκε ίου

lΎΧΑΙ Ο -Π ΙΘΑΝΟ -B E BAI O Παναγιώτης Π. Χριστόπουλος - Θανάσης Π. Χριστόπουλος

Καθημερινά χρησιμοποιούμε λέξεις όπως τυχαίο, πιθανό , βέβαιο που συνδέονται με τη θε­ωρία των πιθανοτήτων. Οι πιθανότητες αρχικά μπήκαν στη ζωή μας από τους Pascal και Fer­mat το 1 654 για να αναλύσουν τυχερά παιχνίδια. Η πρώτη αξιωματική θεμελίωση έγινε το 1 933 από τον Ρώσο μαθηματικό Kolmogorov. Οι πιθανότητες αποτελούν βασικό εργαλείο όλων των επιστημών. Η πιθανότητα να συμβεί ένα γεγονός (ενδεχόμενο) μπορεί να εκφραστεί είτε ως κλά­σμα μικρότερο της μονάδας, είτε ως δεκαδικός από [0- 1 ] είτε επί τοις εκατό.

Ι στορική Αναδ ρομή στην έννοια τη ς Π ιθανότητας

Ο Αριστοτέλης (384-322 π.Χ.) διατύπωσε τη διάκριση μεταξύ των λέξεων γνώση και γνώ­

μη . Θεώρησε δηλαδή ότι η γνώση αφορά σε κάτι που είναι σωστό ή λάθος, ενώ η γνώμη σε κάτι που μπορεί να είναι σωστό ή λάθος. Έδωσε επίσης τις έννοιες του τυχαίου, του aπροσδόκητου και της σχετικής συχνότητας. Θεωρούσε όμως ότι το τυχαίο δεν είναι επιστημονική έννοια, ο­φείλεται στη δική μας αδυναμία να ερμηνεύσουμε τα φαινόμενα και έδωσε το παράδειγμα: Ο­ποιαδήποτε ανακοίνωση για το αποτέλεσμα μιας ναυμαχίας που θα γίνει την επόμενη μέρα θα είναι σωστή ή λάθος μετά το τέλος της ναυμαχίας. Πριν τη ναυμαχία καμία ανακοίνωση δε μπο­ρεί να είναι αληθής.

Ο Καρνεάδης (21 4- 129 π.Χ. ), που έζησε στα ελληνιστικά χρόνια, έδωσε μια πρώτη έννοια της πιθανότητας ως μορφής γνώσης, αρνούμενος την ύπαρξη κριτη ρίου της αλήθειας. Όπως γράφει ο Σέξτος ο Εμπειρικός, ο Καρνεάδης διέκρινε τρεις βαθμούς πιθανότητας (πιθανής γνώ­σης) . Ο πρώτος βαθμός πιθανότητας, η πιθανή φαντασία, χρησιμοποιείται όταν ασχολούμαστε με κοινά πράγματα ή όταν δεν έχουμε καιρό, π.χ. κάποιος κυνηγημένος φτάνοντας σε ένα χα­ντάκι φαντάζεται ότι μέσα στο χαντάκι είναι κρυμμένοι οι κυνηγοί του, οπότε χωρίς να το ξα­νασκεφθεί αλλάζει κατεύθυνση και φεύγει από το χαντάκι. Ο δεύτερος βαθμός πιθανότητας, η

απερ ίσπαστος φαντασία, χρησιμοποιείται για σπουδαιότερα πράγματα όταν κάποια παράστα­ση που μας δημιουργείται δεν έρχεται σε αντίφαση με άλλες παραστάσεις του ίδιου λογικού πλαισίου, π.χ. αν κινούμενοι σε σκοτεινό δωμάτιο δούμε ένα σκοινί στριμμένο αμέσως πηδάμε πάνω από αυτό γιατί το φανταζόμαστε ότι είναι φίδι, αλλά καθώς ξανακοιτάμε πίσω το σκοινί να είναι ακόμη ακίνητο, αποφασίζουμε ότι δεν είναι φίδι. Ο τρίτος βαθμός πιθανότητας, η διε­

ξωδευ μένη φαντασία, απαιτεί ένα ολόκληρο σύστημα από παραστάσεις να αποδειχθεί ότι έχει εσωτερική αλληλουχία και δεν αντιφάσκει με την εμπειρία, π.χ. στο τελευταίο παράδειγμα με

ΕΥΚΛΕΙΔΗΣ Β' 75 τ.3/52

Page 56: Ευκλειδης Β 75

------------- Μαθηματικά για την Γ Λυκείου -----------­

το σκοινί, αφού δούμε ότι είναι ακίνητο, σκεφτόμαστε ότι ενδέχεται να είναι ακίνητο λόγω του χειμερινού κρύου, γι' αυτό παίρνουμε ένα ραβδί και το κουνάμε. Εφόσον ούτε και τώρα βλέ­πουμε το σκοινί να κινείται καταλήγουμε να αποκλείσουμε ότι είναι φίδι. Ούτε όμως ο Καρνεά­δης ούτε και κανείς άλλος στην αρχαιότητα, όρισε ποσοτική έννο ια της πιθανότητας. Ο Spinoza ( 1 632- 1 677) πίστευε ότι η άγνοια της πραγματικότητας μας οδηγεί να αποδίδουμε στην τύχη ορισμένα γεγονότα.

Η θεωρία Πιθανοτήτων αναπτύχθηκε από την ανάγκη να αντιμετωπισθούν πρακτικά προ­βλήματα. Ο 1 7°ς αιώνας χαρακτηρίζεται από την ανάπτυξη του διεθνούς εμπορίου και την πλη­ρωμή ασφαλίστρων, όπου έπρεπε να ληφθούν υπόψη τα ατυχήματα κατά τη μεταφορά.

Επίσης η οργάνωση του κράτους με τα νέα δεδομένα απαιτούσε υπολογισμούς εσόδων και εξόδων. Γνωστοί μαθηματικοί συμβούλευαν τους ηγεμόνες για το ποσό που αναμένεται να συ­γκεντρωθεί από φόρους, για το πλήθος των κατοίκων της χώρας ή του στρατού κλπ. Α ναφέρε­ται ότι ο Leonard Euler ( 1 707- 1 783) έδωσε συμβουλές στο βασιλιά Frederick της Πρωσίας το 1 754 και το 1 763 για την τιμή πώλησης των κρατικών λαχείων.

Η ανάγκη για μια αξιωματική θεμελίωση της θεωρίας πιθανοτήτων με μαθηματική αυστη­ρότητα παρουσιάσθηκε από τον D. Hilbert στον κατάλογο των σπουδαίων άλυτων προβλημά­των που έδωσε το 1 900. Η σημερινή αξιωματική θεμελίωση οφείλεται στον Α.Ν. Kolmogoroν το 1 933 που παρουσίασε τις πιθανότητες ως ειδική περίπτωση της θεωρ ίας μέτρου .

Π Ι ΝΑ ΚΑΣ 1

ΣΤΗΛΗ Α ΣΤΗΛΗ Β ΣΤΗΛΗ Γ ΔιατύπωσJΙ Συμβολισμός Διάγραμμα Venn

Πραγματοποιείται A n B � Το Α και το Β �/

Τουλάχιστον ένα εκ των δύο (το A u B ιcS?� Α ή το Β )

� Α '

Δεν πραγματοποιείται το Α Α ' \ Α ) I -----------

Α - Β .� Ναι το Α και όχι το Β ___./) ή A n B '

Μόνο ένα εκ των Α, Β (Α-Β) υ (Β-Α)

ICQ) .

ή (Α n B ') u (B n A ')

ΕΥΚ\ΕΗΗΣ Β ' 7 5 τ.3/53

Page 57: Ευκλειδης Β 75

------------ Μαθηματικά για την Γ Λυκείου -----------

� Β

Ούτε το Α ούτε το Β (Α υ Β) ' u� ή (Α' 11 Β ') �·

Το πολύ ένα εκ των Α, Β (A n B) ' ή Α' υ Β '

Πραγματοποιείται το Α ή δεν Α υ Β '

ή (Α' 11 Β) ' πραγματοποιείται το Β

ή (8-Α) '

A n 8= 0 Ειδικές περιπτώσεις Ν(Α υ Β)=Ν(Α)+Ν(Β) Ι . Α, Β ασυμβίβαστα Ρ( Α υ Β)=Ρ(Α)+Ρ(Β)

Α � Β

2 . Α υποσύνολο Β Α Η=Α Α υ Β=Β

B c A

3 . Β υποσύνολο Α Α -,8=8 Α υ Β=Α

Γενική μορφή Α, Β ςΩ

ΕΠΙ ΤΟΥ ΤΥΠΟΓΡΑΦΕΙΟΥ . . . ΓΙΑ ΤΟ ΤΕΥΧΟΣ 74 . . .

1 . Στην άσκηση 4 ερώτημα (γ) της Άλγεβρας Α ' Λυκείου εκ παραδρομής αντί να γρα-

φεί R γράφτηκε � .

2 . Στην Άσκηση 6 της Κατεύθυνσης Γ Λυ­

κείου, ο ισχυρισμός lim f- 1 ( y) = f- 1 ( Ι ) = Ι , y -> 1

δηλαδή με ελεύθερη γραφή : χ ---1- Ι όταν

y ---1- Ι ισχύει, αν η f είναι συνεχής οπότε

και η Γ1 συνεχής (πρόταση που θέλει από­

δειξη)

Ο συνάδελφος Κ. Σερίφης έδωσε παρά­δειγμα όπου y ---1- Ι ενώ χ ---1- -ω (προφα-

νώς για μη συνεχή συνάρτηση) .

ΕΥΚΛΕΙΔΗΣ Β ' 7 5 τ.3/54

Page 58: Ευκλειδης Β 75

----------- Μαθηματικά για την Γ ' Λυκείου -----------

Ισχύουν τα παρακάτω:

Είναι Α Β c Α ή Α Β c Β άρα

Ρ(Α Β)::; Ρ(Α) ή Ρ(Α Β)::; Ρ(Β) οπότε

Ρ( Α Β)::; min {Ρ( Α), Ρ(Β) } Ακόμη Ο::; Ρ(Α - Β) επειδή ακόμη

Ρ(Α Β) ::::; 1 ή Ρ(Α) + Ρ(Β)- Ρ(Α Β) ::::; 1 άρα Ρ(Α) + Ρ(Β)- 1 ::::; Ρ(Α Β) αν Ρ(Α) + Ρ(Β)- 1 είναι αρνητικό τότε γράφουμε Ο:::;Ρ (Α Β) δηλαδή max {O, Ρ(Α) + Ρ(Β)-1 } ::::; Ρ(Α Β) .

Είναι Α ς Α Β ή B c A Β άρα Ρ(Α) ::::; Ρ(Α Β ) ή Ρ(Β) ::::; Ρ(Α _ Β) οπότε

max { P(A) , Ρ(Β) } ::::; Ρ(Α Β) Ακόμη Ρ( Α Β) � Ο αλλά Ρ( Α _ Β) = Ρ(Α) + Ρ(Β)- Ρ(Α Β) ή Ρ(Α Β) = Ρ(Α) + Ρ(Β)- Ρ(Α Β) δηλα­

δή Ρ(Α) + Ρ(Β)- Ρ(Α Β) � Ο άρα Ρ(Α Β ) ::::; Ρ(Α) + Ρ(Β) αν το Ρ(Α) + Ρ (Β ) είναι μεγαλύτερο της μονάδας τότε γράφουμε Ρ(Α Β) ::; ι .

Είναι Α Β ' c A ή Α Β ' cB ' άρα

Ρ(Α Β ')::; Ρ(Α) ή Ρ(Α Β ')::; Ρ(Β ')= 1- Ρ(Β) Έχουμε P(A)+P(B ')- l :::;P(A Β ') ή Ρ(Α) - Ρ(Β) ::::; Ρ(Α Β ') δηλαδή max {O,

Ρ(Α)-Ρ(Β) }:::;P(A-B):::;min {Ρ( Α), 1 -Ρ(Β) } .

i) (Α Β)' = Α' Β' Δεν πραγματοποιείται

ούτε το Α ούτε το Β , αλλά πραγματοποι­

ούνται ταυτόχρονα τα Α' και Β ' . ii) (Α Β)' = Α' Β' Δεν πραγματοποιού-

νται ταυτόχρονα τα Α και Β, αλλά πραγ­

ματοποιείται το Α' ή το Β Ό

Ρ(Α ' Β)=Ρ(Α ') +Ρ(Β)- Ρ(Α Β)= 1-Ρ(Α)+Ρ(Β)-Ρ(Β-Α)=

= 1-Ρ(Α)+Ρ(Β)-[Ρ(Β)- Ρ(Α Β)]= = 1-Ρ(Α)+Ρ(Β)-Ρ(Β)+ Ρ(Α . · Β)= = 1-Ρ(Α))+ Ρ(Α - Β) άλλος τρόπος με τους • . ι · · .

είναι: Α' _ Β=Α' (Β ') '=( Α Β ') '= (Α-Β) ' και

Ρ(Α' Β)= Ρ((Α · B) ')= l-P(A-B) = 1 -Ρ(Α)+Ρ(Α -Β).

,. ., , . και πως αυτός αξιοποιείται στις ασκήσεις και τα προβλήματα.

1 ) Αν Ω={ 1 ,2,3} 1

Ρ( 1 )=2Ρ(2)- -4 και Ρ2

(2)+ Ρ(Ι )

=Ρ(3) ποια είναι η πιθανότητα για 3

κάθε απλό ενδεχόμενο;

'Εστω Ρ(2)=χ τότε Ρ( 1 )=2χ-� και 4

2 1 1 Ρ(3)=χ + - (2χ-- ) 3 4

Ισχύει Ρ( 1 )+Ρ(2)+Ρ(3)= 1 1 2 1 ι 2χ- - +χ+χ +- (2χ-- )= 1 4 3 4

χ2+2χ+χ+ � χ-�--1 - 1=0 1 2χ2+44χ-1 6=0 3 4 1 2

3χ2+ 1 1 χ-4=0 - 1 1 - 1 3 ' χ 1 = =-4 απορριπτεται

6

χ2 = - 1 1 + 1 3 1 6 3

Οπότε Ρ ( Ι )= � -� = 2_ 3 4 1 2

,

Ι 1 5 9 3 Ρ(3)=- + - . - = - =-9 3 1 2 36 1 2

1 Ρ(2)= 3 ,

2) Αν Ω={1 ,2 ,3,4,5,6} είναι ο δειγματικός χώ­ρος που προκύπτει από τη ρίψη ενός μη α­μερόληπτου ζαριού και ισχύει Ρ(ί)=z-ϊ-ι για

1 i � 3 και Ρ(ί)= -.- για i � S.

2ι - 4 ς � Να βρεθούν οι πιθανότητες των α­

πλών ενδεχομένων.

ΕΥΚΛΕΙΔΗΣ Β' 75 τ.3/55

Page 59: Ευκλειδης Β 75

------------ Μαθηματικά για την Γ Λυκείου ------------

β) Αν Α={λ ε Ω I η f είναι γνησίως αύ­ξουσα στο R} όπου

f(x)=(λ-l)x3 -λχ2+2χ+λ

Λύση

α) Αν i ::; 3 P( l )=T1- 1 =T2= _!_ , 4

P(2)=T2- 1 =T3=i , Ρ(3)=Τ3- 1 =2-4= 1�

αν i � 5 Ρ(5)= 1

= _!_ 2.5 - 4 6

1 1 Ρ(6)- -

2 .6 - 4 8 6

επίσης ισχύει Σ Ρ(ί) = 1 άρα i = l

1 1 1 I 1 1 3 - + - + - +Ρ(4)+ - + - = 1 Ρ(4)= - . 4 8 1 6 6 8 48

β) f ' (χ)=3(λ-1 )χ2-2λχ+2 για να είναι η f

γνησίως αύξουσα στο R πρέπει f ' (χ)� Ο .

Αν λ= 1 τότε f ' (χ)=-2χ+2 μή δεκτή τιμή

του λ γιατί η Γ αλλάζει πρόσημο στο Χ0 = 1

οπότε η f αλλάζει μονοτονία.

Αν ληt: 1 τότε η Γ(χ) είναι τριώνυμο και για

να είναι αυτό μεγαλύτερο ή ίσο από μηδέν για

κάθε χ ε R πρέπει η διακρίνουσά του να είναι

μικρότερη ή ίση του μηδενός και το α=3(λ-1 )>0. Αλλά Δ=4λ2 -24(λ-1 )=

4λ2 -24λ+24 = 4(λ2 -6λ+6) οπότε Δ::;Ο<::::>λ2 -6λ+6::;0 .

' : - r. : - � 7 ' Δ =36-24= 1 2 και λ1 ,2=�=----. - οποτε

λ ι=3- J3 και λ2=3+ J3 •

3 - J3

- -

ο 2 3 4 5 6 - -ι - - -� - -

Πρέπει λ 1::;λ�Ξλ2 και λ> 1 επομένως

δεκτές τιμές λ=2, λ=3 και λ=4 δηλαδή

Α= {2,3 ,4 } και

p( Α )=p(2 )+p(3) +ρ( 4 )= _!_ + __!_ + � = _!_!_ 8 1 6 48 24

Τα προβλή ματα πιθανοτήτων και η α­

ντ ιμπώπισή τους Αφού μελετήσουμε προσεκτικά το πρό­

βλημα φροντίζουμε να θέτουμε ως ενδεχόμενα

Α και Β κάποια που περιγράφονται στο πρό­

βλημα πιθανόν έμμεσα αποφεύγουμε για τα

Α ,Β προτάσεις σίJνθετες ή προτάσε ις με άρ­νη ση επίσης αν θέσουμε ως Α κάποιο ενδεχό­

μενο πρέπει να προσέξουμε να μη θέσουμε ως

Β το συμπλήρωμά του π.χ. αν Α τεθεί το ενδε­

χόμενο «είναι άνδρας» τότε δεν ονομάζουμε Β

το ενδεχόμενο «είναι γυναίκα», αλλά Α ' . Για

σύνθετες προτάσεις συμβουλευτείτε τον πίνα­

κα 1 .

Ασκή σεις 1 . Σε ένα σχολείο το 25% των μαθητών δεν έ­

χει ηλεκτρονικό υπολογιστή και το 15% έχει κινητό αλλά όχι ηλεκτρονικό υπολογιστή. α) Ποια η πιθανότητα αν επιλέξουμε τυχαία ένα μαθητή να μην έχει ούτε κινητό ούτε Η/Υ β) να μην έχει κινητό ή να έχει Η/Υ.

ΛίJση Θέτω Α: «έχει Η/Υ», Β : «έχει κινητό»

άρα Ρ(Α ' ) = 0,25 και Ρ(Α)= 1 - 0,25=0,75 Ρ(Β-Α)=Ο, 1 5 ή Ρ(Β)- Ρ(Α-ιΒ) = Ο, 1 5

α) «ούτε κινητό ούτε Η/Υ»: (Α _ ιΒ) '

Ρ(Α_ ιΒ) '= 1 -Ρ(Α _ ΙΒ)= 1 -[Ρ(Α)+Ρ(Β)-Ρ(Α rB)]

= 1 -[0, 75+0, 1 5 ]= 1 -0,90=0, 1 ο β) Ρ(Β Έ.r .�. )=Ρ(Β ' )+Ρ(Α)-Ρ(Α - ιΒ ' )=

= 1 -Ρ(Β)+Ρ(Α)-Ρ(Α)+Ρ(Α - rB)=

= 1 -[Ρ(Β)-Ρ(Α - ιΒ)]= 1 -0, 1 5=0,85 .

2. Ένα Λύκειο έχει 80 μαθητές στη Γ τάξη εκ των οποίων 30 έχουν επιλέξει Θεωρητική κατεύθυνση και 45 μαθητές έχουν επιλέξει μαθηματικά γενικής παιδείας(ΜΓΠ) ως πα­νελλαδικά εξεταζόμενο. Οι μαθητές της θε­τικής και τεχνολογικής κατεύθυνσης που δεν έχουν επιλέξει μαθηματικά γενικής παι­δείας είναι 1 Ο. Α ν επιλέξουμε τυχαία ένα μαθητή. α) να βρεθεί η πιθανότητα να είναι της θεω­ρητικής κατεύθυνσης ή να έχει επιλέξει μα­θηματικά γενικής παιδείας, β) την πιθανότητα να είναι της θεωρητικής κατεύθυνσης και να έχει επιλέξει ΜΓΠ. Πό-σοι είναι αυτοί οι μαθητές; Λύση Α : «είναι μαθητής θεωρητικής κατεύθυνσης»

Β : «ο μαθητής έχει επιλέξει ΜΓΠ»

ΕΥΚΛΕΙΔΗΣ Β' 75 τ.3/56

Page 60: Ευκλειδης Β 75

------------ Μαθηματικά για την Γ Λυκείου ------------

Ρ( Α)= Ν( Α)

= 30 =�

Ν(Ω) 80 8

Ρ(Β)= Ν(Β) = 45 = _2_

Ν(Ω) 80 1 6

α) Ρ(Α ' -Β ')= Ρ(Α _ 8) '= 1 -Ρ(Α._ Β) άρα

Ρ( Α . - Β ' )= !Q_ = _!_ = 1 -Ρ(Α - · Β) 80 8

ή Ρ(Α _Β)= 7_ και 8

β) Ρ(

Α·-Β)=Ρ(Α)+Ρ(Β)-Ρ(Α·-Β)=

15 ___2 =__!_ 16 8 16

N(A n B) , Ρ(Α -Β)= οποτε

Ν(Ω)

Ν(Α -Β)= -

1 80=5 μαθητές.

1 6

γ) Τα ευνοϊκά αποτελέσματα είναι

Β= { αα, αβαα, αβαβα, β β, βαββ, βαβαβ }

είναι Ν(Β)=6

άρα Ρ(Β)= ���� =

1� ή Ρ(Α)=60%.

4. Σε ένα pet-shop υπάρχουν λιγότερα από 50 άσπρα και μαύρα γατάκια. Επιλέγοντας τυ­χαία ένα γατάκι η πιθανότητα να είναι αρ­σενικό-μαύρο είναι 30% ενώ η πιθανότητα

' ' ' 1 να ειναι ασπρο ειναι - . 3

α ) Ποιά η πιθανότητα να είναι μαύρο­θηλυκό; β) Πόσα είναι συνολικά τα γατάκια; Λί!ση Έστω Α: «άσπρο γατάκι» οπότε Α ' : «μαύρο

3. Στα play off ενός πρωταθλήματος 2 ομάδες γατάκι» φτάνουν στον τελικό και παίζουν μεταξύ Και Β : «αρσενικό γατάκι» οπότε Β ' : «θηλυκό τους τόσους αγώνες ώστε κάποια από τις δύο να πετύχει 2 συνεχόμενες ή 3 συνολικά νίκες ώστε να ανακηρυχθεί πρωταθλήτρια. (Κάθε ομάδα έχει ίδια πιθανότητα νίκης σε κάθε αγώνα). α ) πόσοι το πολύ αγώνες θα γίνουν; β) Ποιά η πιθανότητα να ανακηρυχθεί πρωταθλή­τρια μια ομάδα μετά από 3 αγώνες; γ) Επί­σης ποιά η πιθανότητα η ομάδα που θα κερ­δίσει στον πρώτο αγώνα να είναι πρωτα­θλήτρια. ΛίJση Συμβολίζου με α νίκη της πρώτης ομάδας

και β νίκη της δεύτερης.

_ .. -· ...

. ζ-{ .:::.:.=_- ,-. __ _ ...-.-

- · (:{' -==-�---

μ..:: __ _

----β _/ .... __

β β

(3 -:::=-

_

--- ��{

---

-

.

·

..

_

--

---._

-

-

-

-

1:::{" - · ι,Λ

Ω= {αα, αβαα, αβαβα, αβαββ, αββ, ββ, βαββ, βαβαβ, βαβαα, βαα} Ν(Ω)= 1 0

α) Θα χρειαστούν 5 το πολύ αγώνες

β ) Α= {αββ, βαα} Ν(Α)=2

Ρ( Α)= Ν(Α)

= � ή Ρ(Α)=20%. Ν(Ω) 1 0

γατάκι»

Δίνεται Ρ(Α)= _!_ αρσενικό-μαύρο: Β - λ 3

Ρ( -. - . ' ) 3001' ' 3 �· . . "". = /0 η -1 0

και

α) μαύρο-θηλυκό : A 'nB ' ή (AuB) ' οπότε Ρ(ΑυΒ) '= 1 -Ρ(ΑυΒ) ( Ι ) αλλά δίνεται

Ρ( ., .-. . . . ) 3 Ρ( ,, ., ) 3 ;. ..... = - <=> ..ι - ... = - <=>

1 0 1 0

<::::> Ρ( Β ·ι-Ρ(ΑΠΒ) = � . 1 0

οπότε Ρ(ΑυΒ)= Ρ(Α)+ [Ρ( Ε ι -Ρ(ΑΠΒ)]= 1 3 1 9 = - + - = -3 1 0 30

επομένως από σχέση ( 1 ) έχουμε ' 1 9 1 1

P(AuB) = 1-P(AuB)= I -- = -30 30

β) ' Ρ(Α Β) Ν(Α υ Β) ' Ν(Α Β) ειναι u = αρα υ = Ν(Ω)

Ρ(ΑυΒ) ·Ν(Ω) δηλαδή Ν(ΑυΒ)= !i ·Ν(Ω) αφού 30

Ν(ΑυΒ) είναι φυσικός αριθμός πρέπει li ·Ν(Ω) 30

να είναι φυσικός και αυτό συμβαίνει αν Ν(Ω) είναι πολλαπλάσιο του 30, αλλά Ν(Ω)<50 οπότε Ν(Ω)=30

ΕΥΚΛΕΙΔΗΣ Β ' 75 τ.3/57

Page 61: Ευκλειδης Β 75

------------ Μαθηματικά για την Γ Λυκείου ----------­

1 . Να βρεθεί η ελάχιστη τιμή της παράστασης : Π= Ρ2 (Α) ·Ρ(Α ')-Ρ(Α) 'Ρ(Α ')+Ρ(Ω) , όπου Α ενδεχόμενο ενός δειγματικού χώρου Ω ενός πειράματος τύχης.

Θέτω p(A)=x οπότε 0::Ξ:χ::Ξ: 1 και p(A ')= 1-x επίσης είναι Ρ(Ω)= 1 επομένως

Π( χ)= x2( 1-x)-x( l-x)+ 1 = -χ3+2χ2 -χ+ 1 Π ' (χ)= -3χ2+4χ-1 Π ' (χ)=Ο<=:>-3χ2+4χ- 1=0 <=> 3χ2-4χ+ 1 =0,

Δ= 1 6-1 2=4 και χ 1 ,2

χ ---οο ο 1 - 1 +οο 3

Π ' (Χ) \\\\\\\ I - ο + ο \\\\\\\\\\\ ι

Π(Χ) \\\\\\\ ! i I ' 1 3 1 2 1

Οποτε Πιnin=-( - ) +2( - ) -- + 1 = 3 3 3 1 2 1 23 =- - +- - - + 1= -

27 9 3 27

\\\\\\\\\\\\

2. Α ν Ας Β και A:;t:0 όπου Α, Β ενδεχόμενα ενός δειγματικού χώρου Ω και Ρ(Α), Ρ(Β) είναι ρίζες της εξίσωσης: χ3+λ(λ+1 )χ2+λ3 χ=Ο , λεR ί) να βρεθούν συναρτήσει του λ οι πιθανότη­τες Ρ(Α), Ρ(Β) ίί) αν 5λ2-13λ+6=0 να υπολογίσετε τις πιθα­νότητες P(AuB), Ρ(Β-Α).

i ) χ3+λ(λ+ l )χ2+λ3 χ=Ο <=>χ[χ2+λ(λ+ Ι )χ+λ3 ]=Ο <:::>χ=Ο (μη δεκτή αφού A:;t:0) ή χ2+λ(λ + I )χ+λ3 =Ο είναι

Δ=λ2(λ + 1 )2 --4λ3=(λ2 -λ)2 οπότε

_ λ ( λ + Ι ) ± (λ2 - λ) ΧΙ ?-· - 2 και τελικά χ=λ ή χ=λ2 . Αφού λε (Ο, I ) επο­μένως λ2<λ ακόμη έχουμε Ας Β άρα Ρ(Α)=λ2 ,Ρ(Β)=λ

i i) Δίνεται 5λ2- 1 3λ+6=0 οπότε Δ= Ι 69- 1 20=49

και λ 1 = 1_ , λ2=2 που απορρίπτεται . 5

Επομένως Ρ(Β)= � και Ρ(Α)= _2__ είναι 5 25

Ας Β οπότε AnB=A και AuB=B

και άρα Ρ(ΑυΒ)= Ρ(Β)= � 5

Ρ(Β-Α)= Ρ(Β)-Ρ(ΑΓΙΒ) = Ρ(Β)-Ρ(Α)= 3 9 6 5 25 25

3. Έστω Α,Β ενδεχόμενα ενός δειγματικού χώ­ρου Ω και Ρ(Α), Ρ(Β), P(AnB), P(AuB) πα­ρατηρήσεις μιας μεταβλητής Χ με διάμεσο

1 δ=- . Δίνεται επίσης ότι η μέση τιμή των 4

Ρ(Α), Ρ(Β), P(AnB), Ρ(Β-Α), Ρ(Α-Β), P(AuB) είναι χ = ..!_ . Υπολογίστε τις πιθα-5 νότητες P(AnB), P(AuB).

Είναι P(AnB) :::; Ρ(Α) :::; Ρ(Β) ::Ξ:Ρ(ΑυΒ) ή

P(AnB) ::Ξ:Ρ(Β) ::Ξ: Ρ(Α) ::Ξ: Ρ(ΑυΒ) οπότε

δ= Ρ (Α) +Ρ(Β)

. Άρα Ρ (Α) +Ρ(Β) = _!_ και ε-

2 2 4

πομένως Ρ(Α)+Ρ(Β)= _!_ ( 1 ) 2

- - ��+Ρ(Β)+�Αnτη+�Β-�+�Α-τη+�Αυτη χ -

6 _ _ 3Ρ(Α) + 3Ρ(Β) - 2Ρ(Α n Β) χ -

6 χ = 3Ρ(Α) + 3Ρ(Β) - 2Ρ(Α n Β)

οπότε 6

3Ρ(Α) + 3Ρ(Β) - 2Ρ(Α n Β) l 6 5

Άρα 3 [Ρ(Α)+Ρ(Β)]-2 P(AnB)= � και λό-5

γ ω της σχέσης ( 1 ) από τη σχέση αυτή προκύ-1 6

πτει: 3 · - -2 P(AnB)= - δηλαδή 2 5

2P(AnB)=_!2- .!3_ ή P(AnB) = ]__ 1 0 l O 20

Επίσης Ρ(ΑυΒ)=[Ρ(Α)+Ρ(Β)]-Ρ(ΑnΒ)= _!_-2

3 7 20 20

ΕΥΚΛΕΙΔΗΣ Β' 75 τ.3/58

Page 62: Ευκλειδης Β 75

-------------- Μαθηματικά για την Γ Λυκείου

Παραγώγιση

ο (ΞJχ ε Α ) (Ρ ( χ ) ) σημαίνει: Υπάρχει ένα τουλάχι­στον χ ε Α, ώστε να ισχύει Ρ( χ), ('ν' χ ε Α) (Ρ ( χ )) σημαίνει: Για κάθε χ Ε Α, ισχύει Ρ(χ) . Όπως ακριβώς αποδίδονται στο βιβλίο :

μαθήματα κλασσικής αναλύσεως : Λάμπρου Ντό­κα, Καθηγητού Πανεπιστημίου Πατρών και στα διεθνούς κύρους Μαθηματικά βιβλία : 1 . Duden, Rechnen und Mathematik, 2 . Schauιη ' s Outl ine of Theory and Problems of Discrete Mathematics .

- 1 : ;[ ... _ . 1.1

Θεωρούμε γνωστό τον ορισμό της συνεχούς συ­νάρτησης και τις βασικές προτάσεις που αναφέρο­νται στην έννοια της συνέχειας. Υπενθυμίζουμε ότι απαραίτητη προϋπόθεση για να είναι μία συ­νάρτηση f : Α � IR. με Α ς IR., συνεχής στην θέση χ0 είναι το χ0 να ανήκει στο πεδίο ορισμού της Α. Θεωρούμε επίσης γνωστό ότι η ΜΗ ύπαρξη της συνέχειας της f : Α � IR. με Α ς IR., σε σημείο χ0 έχει σαν αποτέλεσμα την ΜΗ ύπαρξη της πα­ραγώγου στην θέση χ0 .Ο μ ω ς θα πρέπει να θυ­μόμαστε ότι η συνέχεια της f στην θέση χ0 δεν σημαίνει πάντα την ύπαρξη της παραγώγου ( ( χ0 ) στην θέση αυτή . Θα πρέπει επίσης να θυ­μόμαστε ότι η f : Α � ΙR. με Α ς IR., δεν έχει παρά-γωγο σε σημείο χ0 Ε Α όταν οι πλευρικές παράγω­γοι στο σημείο χ0 Ε Α δεν είναι ίσες ή όταν υπάρ-

χει σημείο P (x0 , f ( x0 ) ) της γραφικής της παρά­στασης που η κλίση της τέμνουσας ΡΤ τείνει στο +οο, αριστερά του χ0 και στο -οο δεξιά ή στο -οο αριστερά του χ0 και στο +οο, δεξιά του . Είναι χρήσιμο να έχουμε κατά νου ότι . 1 , i

.;; i ι - της γραφικής παράστασης της f : Α � IR. (Α διάστημα) , στις εξής περιπτώσεις : i ) Στα Εσωτερικά σημεία του Α που μηδενίζουν την πρώτη παράγωγο (Προσοχή : Η ( ( χ0 ) = Ο δεν είναι ικανή συνθήκη ώστε να υπάρχει ακρότατο της f στην θέση χ0 ) .

Ολοκλήρωση

Σωτήρης Ε. Λουρίδας i i i) Άκρα του πεδίου ορισμού της συνάρτησης. Θα πρέπει επίσης να έχουμε κατά νου ότι � 1 1 � ''��

, '" : :ic J 1ί της γραφικής παράστασης της f : Α � IR. (Α διάστημα) σε θέση χ = χ0 ,όταν: i) Το χ0 είναι εσωτερικό σημείο του πεδίου ορι­σμού της f με ( ( χ0 ) = Ο . Προσοχή : Η ( ( χ0 ) = Ο δεν είναι ικανή συνθήκη ώστε στο χ0 να έχουμε σημείο καμπής της γραφι­κής παράστασης της f) i i ) Το χ0 είναι εσωτερικό σημείο του πεδίου ορι-σμού της f και Γτ · ' '� " ' · η τιμή ( ( χ0 )

Θεωρούμε τις συναρτήσεις f : [α, β] � IR., g : [α, β] � :JR. που είναι συνεχείς στο [α, βJ και παραγωγίσιμες στο (α, β). Να α­ποδειχθεί ότι: ( 3χ0 ε ( α, β} ){g(β} - g( α} ]r' ( χ0 ) =

= [ f ( β } - f (α) J g' ( Χ0 )) ( * ) , :: • Γνωρίζουμε ότι θα πρέπει να θεωρήσουμε

συνάρτηση ,έστω h : [α, β ] � IR. συνεχή στο διάστη­μα [α, β] και παραγωγίσιμη στο διάστημα (α, β) που αν είναι δυνατόν να είναι γραμμικός συνδυασμός των f +c, g + m με c,m πραγματικές σταθερές ( αφού ( f + c )' = (και (g + m )' = g' και με βάση την σχέση (*)), με σκοπό να εφαρμόσουμε για αυτήν το θεώρη­μα του Rolle (θυμόμαστε δηλαδή την ιδέα απόδειξης του θεωρήματος της Μέσης τιμής του Διαφορικού Λογισμού). Έστω συνάρτηση

( • ) h : [α, β] � ΙR., h ( χ ) = [g (β ) - g (α)} [f ( x ) + c] - [f ( β ) - f (α)J [g ( x ) + m ] ( * *)

με h (α) = Ο και h (β) = Ο. Από εδώ προκύπτει:

c = -f (α) , m = -g (α) . Όντως για τις τιμές αυτές από την (** ) παίρνουμε εύκολα h ( α) = h (β ) = Ο. Επομένως έχουμε :

' i ' η : Θεωρούμε την συνάρτηση h : [α, β] � IR., h ( χ ) = [ g (β ) - g (α) J [ f ( χ ) - f (α) J --[f (β) - f ( α) J [g ( χ ) - g ( α)] . Η συνάρτηση αυτή i i ) Εσωτερικά σημεία όπου η πρώτη παράγωγος

υπάρχει. προφανώς είναι συνεχής στο [α, β] , παραγωγίσιμη

ΕΥΚΛΕΙΔΗΣ Β ' 75 τ.3/59

Page 63: Ευκλειδης Β 75

------------ Μαθηματικά για την Γ Λυκείου -----------­

στο (α, β) .Εύκολα βλέπουμε ότι Αυτό προκύπτει άμεσα από το θεώρημα του h(α)=h(β)=Ο.Επομένως εφαρμόζεται το θεώρημα Cauchy, αν λάβουμε υπ' όψη ότι g' ( χ 0 ) * Ο, γιατί του Rolle από όπου προκύπτει ότι:

αν ίσχυε g ' ( χ 0 ) = Ο, θα είχαμε ( ( χ0 ) = Ο, που εί-( ΞJχ0 ε ( α, β) ) ([ g (β) -g( α) ] f' ( χ0 ) = [ f(β) - f( α) Jg' ( Χ0 ) ) Π αρατη ρήσεις : i) Από πλευράς τεχνικής η προηγούμενη απόδειξη μπορεί επίσης να πραγματοποιηθεί, θεωρώντας τηνσυνάρτηση : h : [α, β ] � IR, h ( χ ) = [ g (β ) - g (α) J f (χ ) - [ f (β ) - f (α) J g (χ ) , μόνο που από εδώ παίρνουμε h(α)=h(β) χωρίς κατ ' ανάγκη να έχουμε την ισότητα με το μηδέν. Απλά από πλευράς μεθοδολογίας όταν ' ζητάμε προέλευση από παραγώγιση ' πρέπει να δίνουμε βάση στην ισότητα ( f + c ) ' = ( ( c σταθερά ). Ας δούμε

ένα παράδειγμα: 'Έστω συνάρτηση f : IR � IR, παραγωγίσιμη στο IR , με lim f ( χ ) = Ο. Αν υπο-

χ ---t+χ·

θέσουμε ότι το lim ( ( χ ) υπάρχει, τότε να υπο-χ -t+χ

λογιστεί. Προφανώς αρκεί να προσδιορίσουμε το

lim ( f ( χ ) + c )' ( c -::1= Ο) ,όταν έχουμε X -t +:x:

lim ( f ( χ ) + c) = c -::1= Ο . X -t +X·

x {f ( x ) + c) lim ( f ( χ ) + c) = lim = c -::1= Ο, με

Χ ---7 +Χ: Χ ---7+Χ Χ

. x {f ( x ) + c) (�) . { x { f ( x ) + c)) ' l ιm = l ιm . =

Χ --->+χ Χ Χ ->+'>) Χ Δ� ( r ( χ ) + c + xf' ( χ ) ) = c => }i�., χ( ( χ ) = ο

και l im _!_ = Ο => l im ( ( χ ) = Ο . Εδώ ας επιση-χ ---++:Ν χ Χ ---7 +Χ

μάνουμε ότι το θέμα αυτό θα μπορούσε να αντιμε­τωπιστεί και με το Θ. Μ. τ. ii) Το θεώρημα αυτό μπορεί να χρησιμοποιηθεί σαν λήμμα ( αφού προηγηθεί η απόδειξή του) σε πάρα πολλά προβλήματα. Για παράδειγμα αν g(x)=x αμέσως έχουμε το γνωστό μας θεώρημα της μέσης τιμής του Διαφορικού Λογισμού. Το ί­διο συμβαίνει και για τις δύο επόμενες εκδοχές. iii) Έχουμε και την εξής 211 εκδοχή του θεωρήμα­τος Cauchy: Αν θεωρήσουμε συναρτήσεις f : [α, β ] � IR, g : [α, β ] � IR, συνεχείς στο διάστημα [α, β] και παραγωγίσιμες στο (α, β) ώστε g (α) * g (β ) και οι ( , g' δεν έχουν κοινή ρίζα στο διάστημα (α, β), ισχύει ότι [ f' ( x 0 ) f (β ) - f ( α) J (ΞJχ

ο ε (α, β ) ) g' ( x o )

= g (β ) - g (α ) j '

ναι άτοπο από την υπόθεση περί μη κοινής ρίζας. ίν) Υπάρχει και 3η εκδοχή του θεωρήματος Cauchy προερχόμενη από την 2η όταν αντικατα-στήσουμε τις απαιτήσεις [ g ( α) * g (β ) και οι

( , g · δεν έχουν κοινή ρίζα στο (α, β)] , από την

πρόταση ( Vx ε (α, β ) ) ( g' ( χ ) * Ο) που οδηγεί στην

g (α) * g (β ) ( ; ) . n αρ{tδειγμα : Έστω f : [α, β] � ΙR, μεα :2: Ο, συνεχής στο διά­στημα fα, β] και παραγωγίσιμη στο διάστημα (α, β).Να αποδειχθεί ότι υπάρχουν

r' ( Χ ι ) = (β + α ) ( ( Xz ) = ( βz + βα + αz ) ( (χ ξ ) . 2χ2 3χ3

Απόδειξη : Εφαρμόζοντας τρεις φορές την 2η εκδοχή του θεω­ρήματος του Cauchy για τα ζεύγη των συναρτήσε-ων ( f( x) ,g( χ) = χ) , ( f{ x) ,g( χ) = xc ) , ( f{ x) ,g ( χ) =χ3 ) ,

, . f(β) -f(α) _ f' (x1 ) f(β) -f(α) _ f' (x2 ) παιρνουμε . --- , > 2 ---, β-α 1 β- -α 2χ2

f (β ) - f (α) f' ( x ) ----'---Ξ-----7---'- = --,3- .'Ετσι έχουμε το ζητούμενο.

β3 - α3 3χ -3 Προφανώς ισχύει: Χ 1 , χ2 , Χ 3 > Ο. 2) Έστω μη σταθερή συνάρτηση f : lR � lR με f ( χ + y) = e'Yf ( χ ) f (y ) , Vx, y ε IR( * ) παραγωγίσιμη στην θέση Ο, με ( (Ο) = Ο. Ν α βρεθεί ο τύπος της συνάρτησης f. Λύση :

( • ) Παρατηρούμε ότι: χ = y = O�f (O) = I ν f (O) = Ο.

( • ) Αν f (Ο ) = O� f ( χ ) = Ο, Vx ε IR,

y =O πράγμα άτοπο

αφού υποθέσαμε ότι η f δεν είναι σταθερή . Τελικά έχουμε f(0)= 1 .

f ( x ) - f (O) f ( x ) - 1 ( (0 ) = 0 � l im = 0 � l im = 0 . χ-->0 χ - ο χ-->0 χ Για τον τυχόντα χ0 ε IR* ισχύει : . f ( x 0 + h ) - f ( x0 ) • eh'" f ( x0 ) f ( h ) - f ( x0 ) l ιm = lιm = h-->0 h h-->0 h . d"'f(h) -1 . e1' '" (f(h) - 1) +e1' x" - 1

=lιm f( Χο) = f( Χ0 ) limι-""-----'---ιι-4J h Ιι--.ο h

ΕΥΚΛΕΙΔΗΣ Β' 75 τ.3/60

Page 64: Ευκλειδης Β 75

------------ Μαθηματικά για την Γ Λυκείου ----------­

= f ( Χ 0 ) iim e1' · x ., + , [ f (h ) - 1 eh ·x u - 1 ) h � O h h

. f ( h ) - 1 . e•' x " - 1 (�) . ( e ι' χ " - 1 ) . lιm = 0, \ιm = l ιm . = με IHO h IHO h h � O h = χ lim e1' · x u = χ . ο IΙ �Ο ο

χ

Συνεπώς ισχύει: f' ( x) = xf( x) ,Vx εlR=>e:f' ( x) +

+-f) f ( x ) = Ο =; [ e-f f { x )) = Ο, Αρα

_.:Ξ_ ( χ =Ο) � e 2 f ( x ) = c, V'x ε iR => c = l => f ( x ) = e 2 , V'x ε iR. 3 ) Υποθέτουμε ότι υπάρχει συνάρτηση f : IR � IRμεf (IR) ς [-l , l ] , πoυ είναι δύο φορές παραγωγίσιμη στο IR , με την ιδιότητα [ f (ο) τ + [ r' (ο) τ = 2 + ε. με ε θετική σταθερή. Να αποδειχθεί ότι (3Χο ε ΙR) ( f( χσ) +( ( χ0 ) = 0) .

Απόδειξη : Από το θεώρημα τη� \1έσης Τιμής του διαφορικού Λογισμού έχουμε ( 3ξ 1 ε (-α, Ο) )

[ ΙΨ Ι l = i tΊ ο Ι -"f ( -α Jl 5 Ι r ( o ) l +Jf ( -α Jl 5 ι}

όμοια παίρνουμε ότι ( 3ξ2 ε (Ο , α ) ) ( /f' (ξ2 ) 1 � ι ) , για θετική σταθερή α 2': 2 . Θεωρούμε την συνάρτηση F : IR � IR, F ( χ ) = [ f ( χ ) τ + [ ( ( χ ) τ . Η συνάρτη­ση αυτή είναι παραγωγίσιμη στο IR. Άρα η F σαν συνεχής στο [ ξ 1 , ξ2 ] δέχεται Μέγιστο έστω στη θέση χ0 ε [ξ 1 , ξ2 ] , τοΜ = F ( χ0 ) . Παρατηρούμε ότι: F (ξ 1 ) � 2, F (ξ2 ) � 2 , F (O) = 2 + ε � Μ => F (ξ 1 ) < Μ και F ( ξ2 ) < Μ => χ0 ε (ξ 1 , ξ2 ) . Παραγωγίζουμε την F και έχουμε : F. (χ ) = 2f' (χ) [ f (χ) + ( (χ)] =>

=> 2f' ( Χ0 ) [ f ( Χ0 ) + ( ( Χ0 ) J = 0 => f ( Χ0 ) + ( ( Χ0 ) = 0,

αφού αν f' ( x 0 ) = 0 => F (x0 ) = [f ( x0 )T � I => => 2 + ε � 1 => Ο < ε � -1 ,

που είναι άτοπο, οπότε ισχύει ( ( χ0 ) :;e Ο . Τελικά έχουμε (3χ0 ε iR) ( f ( x 0 ) + f" ( x 0 ) = 0) . 4) Έστω συνάρτηση f : [α, β] � IR με f (α) = f (β) = Ο και ( V'x ε (α, β)) ( r ( χ) :;e Ο) , που είναι συνεχής

στο (α, β] και παραγωγίσιμη στο (α, β) .Να απο­δειχθεί ότι, υπάρχουν χ 1 , χ2 ε (α, β) με χ 1 :;e x2 ώστε r' ( χ . ) r' ( χ2 ) + g' ( χ. ) g' ( χ2 ) r ( χ . ) r ( χ2 ) = ο, για κάθε συνάρτηση g : [α, β ] � IR που είναι συνεχής στο (α, β( και παραγωγίσιμη στο (α, β) ώστε ( \i χ ε (α, β ) ) ( g · ( χ ) :;e Ο) . Απόδειξη : Αρκεί να υπάρχουν Χ 1 , Χ 2 ε ( α, β ) : f' ( χ 1 ) = -g' ( Χ 1 ) f ( Χ 1 ) και ( ( Χ 2 ) = = g' ( χ 2 ) f ( χ 2 ) , δηλαδη ( ( Χ 1 ) + g' ( Χ 1 ) f ( x 1 ) = Ο και( ( x2 ) - g' (x2 ) f ( χ2 ) = 0

ή eg( x ι )( ( χ . ) + ( eg( x , ) ) ' f ( χ . ) = ο

( ( χ2 ) eg( x , ) - ( eg( x , ) ) ' f ( χ2 ) και , = 0 (eg( x , ) ) -

( eg( x , )f ( x ) ) ' = Ο και [ f ( χ z ) ) ' = 0. I eg( x , )

ή τελικά

Θεωρούμε λοι-

πόν τις συναρτήσεις, h : [α, β] � IR, h ( χ) = eg( x Jf ( χ) και k : [α, β ] � ΙR, k ( χ ) = :�,:;. Οι συναρτήσεις αυτές είναι συνεχείς στο διάστημα [α, β] και παρα­γωγίσιμες στο (α, β) με h(α)=k(α)=Ο και h(β)=k(β)=Ο .Αυτό σημαίνει ότι μπορούμε να ε­φαρμόσουμε το θεώρημα του Rolle για τις συναρ­τήσεις αυτές. Έτσι για τις συναρτήσεις h, k αντί­στοιχα έχουμε : 3χ1 ε ( α, β) : h' ( χ1 ) =Ομεh' ( χ) =eg( x ) ( f' ( χ) +g' ( x) f{ χ)) => =>( ( χ1 ) =-g' ( χ1 ) f{ χ1 ) ,

3χ2 ε (α, β) : k' (χ2 ) = 0μεk' ( χ) = ( { x) -:;�

Jx) f(x) =>

e => ( ( χ2 ) = g' ( χ2 ) f{ χ2 ) . Από τις προηγούμενες ισότητες παίρνουμε: ( ( χ ι ) f' ( χ 2 ) + g ' ( Χ 1 ) g' ( Χ 2 ) f ( Χ 1 ) f ( Χ 2 ) = 0 . Από τα δεδομένα κατανοούμε ότι ( ( χ 1 ) f' ( χ 2 ) :;e Ο . Παρατηρούμε ότι: χ1 = χ2 =>2f' {x1 ) = 0=>f' {x1 ) = 0, πράγμα άτοπο, επομένως χ 1 :;e χ2 . Μεθοδολογικό σ,(όλιο: Το προηγούμενο πρόβλημα μας δίνει την δυνατότητα κατασκευής προβλημάτων αρκεί ως f, g να λαμβάνουμε συγκεκριμένες συναρτήσεις. S)Θεωρούμε την συνάρτηση f : ( O,+oo) � IR,f (x) = ( l + �)' , με α θετική

ΕΥΚΛΕΙΔΗΣ Β' 75 τ.3/61

Page 65: Ευκλειδης Β 75

------------ Μαθηματικά για την Γ Λυκείου -----------­

σταθερή. Να εξετάσετε αν η γραφική παράστα- ραγωγίσιμες στο 1-θ, θ l συναρτήσεις, ση της f έχει οριζόντια aσύμπτωτη. h : [-θ, θ] � JR, k : [-θ, θ] � JR με h (Ο) = Ο, Κατά τα γνωστά θα εξετάσουμε την ύπαρξη του 1 im f ( χ ) . Παρατηρούμε ότι:

x � +cc

k (Ο) = β, βh · ( χ) - αk ( χ) = αk · ( χ) + βh ( χ) = Ο, με α, β σταθερές διάφορες του Ο. Είναι δυνατόν με βάση τα δεδομένα αυτά να προσδιοριστούν οι συναρτήσεις h, k ; ( α ) ιη ( ι + �)

x ln l +- ( α ) χ , ι f (χ ) = e χ ' με Δ�, χ ln 1 +--;- = Δ�οc ' Παρατηρούμε ότι τα δεδομένα μπορούν να γρα-

φούν ως εξής:

με l im ιη ( 1 + �J = lim _!_ = ο X � +OC Χ Χ �+?:: Χ

. Ηι + �)] . ( ι + �) ι � " και lιm . = lιm χ

Η +Ύc (�) lim _α_ = α => lim f ( x ) = eα .

X --4- +Y.J α X --4-+:f.J 1 + -χ

χ

Άρα η y = eα είναι οριζόντια aσύμπτωτη (Εδώ ε­φαρμόστηκε ο κανόνας ι 'Hospital) . ι ) Ν α εξετάσετε αν η εξίσωση χ + 1 + Ι η { χ2 + 1 } = Ο ,έχει πραγματικές ρίζες και πόσες.

Θεωρούμε την συνάρτηση f : JR � JR, f ( χ ) = χ + 1 + Ι η ( χ 2 + 1 ) , αφού (Vx Ε JR ) ( χ 2 + 1 > Ο) . Η συνάρτηση αυτή εί­ναι συνεχής και παραγωγίσιμη στο JR . Παρατη-

ρούμε ότι f' (x) = 1+ ;χ =>f' (x) ;:::: OVx EJR-{-1} => χ + 1

=> f t ( γνησιως) . Παρατηρούμε επίσης ότι { ->? +οο ) Δ� r ( χ ) = Δ�, (χ + 1 + ι η ( χ 2 + 1 ) ) =

}lm. ++ι + ιn� ' + ι )}

με

. 1 + Ι η (χ 2 + 1 ) ( ::) . [ 1 + 1n ( χ 2 + 1 )} l ιm = lιm , =

Χ --4ο-Χ Χ X --4- -0C· χ = lim � = Ο => lim f ( χ ) = -οο , ενώ εύκολα

Χ -->-οο Χ + 1 Χ -->-οο

προκύπτει lim f ( χ ) = +οο. Επειδή η f είναι συνε-χ ---++οο

χής (ΞJρ Ε JR ) ( f (ρ ) = Ο) , με τον ρ μοναδικό αφού η f είναι γνησίως αύξουσα άρα και 1 - 1 . . , Έστω θ, θετική σταθερή. Θεωρούμε τις πα-

h(O) = Ο k (O) = 1 ( h(x) ] ' = k (x) [ k (x ) ] . = - h(x) α ' β ' α β ' β α '

' ' h ( χ) οποτε οι συναρτησεις f : [-θ, θ] � JRμεf( χ) =--, α

g : [-θ, θ] � JRμε g ( χ ) = k ( χ ) , έχουν παρόμοια β συμπεριφορά προς τις συναρτήσεις ημ, συν αντί­στοιχα. Είναι, λοιπόν λογικό να επιδιώξουμε να αποδείξουμε ότι f ( χ ) = ημχ , g ( χ ) = συνχ . Για αυ-τό, αν σκεφτούμε αλγεβρικά (Α, Β Ε 1R μεΑ2 + Β2 = 0 => Α = Β = Ο) , αρκεί να θεωρήσουμε την συνάρτηση λ : [-θ, θ) � JR, λ ( χ ) = [f ( χ ) - ημχ τ +

+[g ( χ ) - συνχΤ με λ(Ο)=Ο με 'την ελπίδα' λ(χ)=Ο, για κάθε στοιχείο του συνόλου [-θ, θ] . Παρατηρούμε ότι: λ ( χ ) = 2( ( x ) f ( x ) + 2g' ( x ) g ( x ) = Ο => λ (χ ) = c

με λ ( Ο) = Ο � c = Ο, οπότε: λ( χ ) = Ο => f ( χ ) = ημχ και g (χ ) = συνχ , Vx Ε [-θ, θ] . Τελικά έχουμε : h( x) = f(χ) =ημχ=>h(χ) =αημχ,

k(x) = α β

= g ( χ ) = συνχ => k ( χ ) = βσυνχ. Έστω οι παραγωγίσιμες στα πεδία ορισμού

τους συναρτήσεις f : [α, +οο) � JR και g : [β, +οο) � JR,μεα, β πραγματικές σταθε­ρές, ώστε: lim f (χ) = λ Ε JR' , lim g ( χ) = k Ε JR' ,

χ---++«> χ --7 +οο

υπαρχουν τα xll� { ( ( χ) - f ( χ) g' ( χ)} και lim g' ( χ) . Να υπολογιστεί το lim ( ( χ) . χ --7 +οο χ --7 +ο:>

Γενικά ,αν για μία παραγωγίσιμη συνάρτηση h : [ξ, +οο) � JR, με ξ Ε JR έχουμε : lim h ( χ ) = μ Ε JR' , υπαρχει το lim h ' (x ) ,

X ---t +oc X ---t+:x:;

τοτε lim h ' ( χ ) = Ο και αυτό διότι X --7 +0C

ΕΥΚΛΕΙΔΗΣ Β' 75 τ.3/62

Page 66: Ευκλειδης Β 75

------------ Μαθηματικά για την Γ Λυκείου ------------e' h ( x ) (�) (e 'h ( x ) ) ' μ = lim h ( χ ) = lim = lim , =

χ ->+"' χ -->+οο e' , __, _ ( e' ) = Διr, ( h . ( χ ) + h ( χ )) , Επομένως Δ�( h ( χ) + h. ( χ)) = μ=> Διr, h. ( χ) = 0. Α ν σκεφτούμε λίγο θα δούμε ότι ισχύει: . f ( χ ) λ * ' . [ f ( χ ) ) ' lιm -,-,( -) = k = e Ε JR . Άρα l ιm --;;--( · ) = ο =>

X ---t+x e� Χ e Χ �+'Χ e� Χ

. f' ( x ) - f ( x ) g' ( x ) . e( x ) k lιm = Ομε lιm e- · = e => X ---t +7') eg( x ) Χ ---t +'Χ:

Δ�( ( ( χ ) - f ( x ) g' ( χ ) ) = Ο με Διrχg. ( χ ) = ο, αφού lim g ( x ) = k E JR* .

χ --; .μχ:

l im f ( x ) g' ( x ) = ek · 0 = 0 ::::> lim f( x ) = O . X ---t+:YJ \ ---t - :ι:

θεωρούμε την συνάρτηση g : [ α1 , β 1 J� { O, +co ) μεα 1 < β 1 , που είναι συνεχής στο διάστημα [ α 1 , β 1 J ,παραγωγίσιμη στο ( α1 , β 1 ) , ώστε ( V'x Ε ( α 1 , β 1 ) ) ( g' ( χ) :;t Ο) , με την g' να είναι παραγωγίσιμη στο διάστημα ( α 1 , β 1 ) και με την g ' συνεχή στο πεδίο ορισμού της. Θεωρούμε α,βμεα1 < α< β < β1 α,βμειlι <α<β< β1 και {3χ0 ε (α, β)} {g" ( χο ) g ( χο ) � [g' ( χο )Τ ) ·

Αν τα σημεία κ (α, g (α )) , M { x0 , g ( x0 ) ) , Λ (β, g (β)} , είναι σημεία της ίδιας ευθείας, να αποδειχθεί ότι η δεύτερη παράγωγος της συ­νάρτησης f : [α, β] � JR, f ( χ) = ln g ( χ) έχει μία τουλάχιστον ρίζα.

Έστω ότι το σημείο Μ είναι μέσο του ευθ. τμήμα­τος ΚΛ έχουμε: g ( xo ) - g (α) = g (β ) - g ( xo ) ( * ) . Αν εφαρμόσου-χ0 - α β - Χ ο με τώρα το θεώρημα της Μέσης τιμής του Διαφο­ρικού Λογισμού για την g στα αντίστοιχα διαστή­ματα παίρνουμε : (3χ 1 Ε (α, χ 0 ) ) [ g' ( x 1 ) = g (χ:;=:(α)} ( )) ( · g (β ) - g ( xo ) ) 3χ 2 Ε ( χ ο , β g ( χ 2 ) = β - χο ' άρα g' ( χ 1 ) = g' ( χ2 ) οπότε από το θεώρημα του

Rolle για την g ' στο διάστημα ( χ 1 , χ 2 ) , έχου­με : (3ξ Ε ( χ 1 , χ 2 ) ) (g" (ξ) = Ο) . Έστω τώρα ότι το σημείο Μ δεν είναι μέσο του ευθύγραμμου τμήμα­τος ΚΛ. Στην περίπτωση αυτή μπορούμε να θεω­ρήσουμε ότι το σημείο Μ διαιρεί το ΚΛ έστω σε λόγο λ :;t: - 1 ώστε λ = Χο - α = g ( xo ) - g (α) => ' β - χο g (β ) - g ( xo ) => g ( Χο ) - g ( α) = g (β ) - g ( χο ) . Εδώ εφαρμόζου-χ0 - α β - Χ ο με εκ νέου το θεώρημα της Μέσης Τιμής του Δια­φορικού Λογισμού : ( · ( ) ) [ ' ( ' ) g ( x 0 ) - g (α) ) 3χ Ι Ε α, Χ ο g Χ Ι = Χ ο - α '

( ::J • ( β )) [ ' ( . ) - g (β ) - g ( xo ) ) ::J X 0 Ε Χ0 , g Χ0 - , - - β - Χ ο έτσι παίρνουμε g' ( χ ; ) = g' (χ � ) , δηλαδή (3ξ Ε ( χ ; , χ � ) ) (g" ( ξ) = Ο) . Σε κάθε λοιπόν περί­πτωση (3ξ Ε (α, β )) {g" ( ξ) = Ο) . Υπολογίζουμε τώ­ρα την δεύτερη παράγωγο της f : ( ( χ ) = g' ( χ ) => ( (χ ) = g" ( χ ) g ( χ ) - [ g' ( χ ) τ

g ( χ ) g2 ( χ ) ;::: Ο, με g' ( χ ) :;t: Ο. Α ν g" ( Χ0 ) g ( Χ0 ) = [ g' ( Χ0 ) τ => ( ( Χ0 ) = 0, ενώ αν g" ( Χ0 ) g ( Χ0 ) > [ g' ( Χ0 ) ]2 , για

" ( ) [g' (ξ)τ " ( ) . χ = ξ => f ξ = ο ( ) < Ο, με f χ0 > Ο. Απο g- χ το θεώρημα του Bolzano για την δεύτερη παράγω­γο της f έχουμε την ύπαρξη ρίζας ρ της 2ης παρα­γώγου στο διάστημα που ορίζουν οι αριθμοί ξ, χ0 . " Qι ) Έστω συνάρτηση f : JR � JR, δύο φορές πα­ραγωγίσιμη στο JR . Θεωρούμε ότι το σημείο { 7, f { 7)) είναι σημείο καμπής της γραφικής παράστασης της f .Θεωρούμε επίσης την συ­νάρτηση g : JR � JR, g ( χ) = r ( χ2 + 3 ) . Αποδείξτε ότι g' ( 2) = 2g" { 2) .

g' ( x ) = 2xf' ( x 2 + 3) , g" ( x ) = 2f' ( x 2 + 3) + +4χ2( (χ2 + 3) => g" (2) = 2( (7 ) . . . .

J ; Έστω δύο κάθετες ημιευθείες Οχ, Oy • Επί της ημιευθείας Oy παίρνουμε τα σημεία

ΕΥΚΛΕΙΔΗΣ Β' 75 τ.3/63

Page 67: Ευκλειδης Β 75

-------------- Μαθηματικά για την Γ Λυκείου ------------­

Β, Α : Ο < β = ΟΒ < ΟΑ = α. Θεωρούμε ότι επί της Οχ κινείται σημείο Μ . Να προσδιοριστεί η θέση του Μ για την οποία η �ΑΜΒ να είναι η μέγιστη. Λ\1ση : Θεωρούμε ω = �ΑΜΒ, φ = �ΒΜΟ, χ = ΟΜ. Πα-ρατηρούμε ότι :

α - β εφω = εφ(ω + φ) - εφφ = χ - (α - β) χ = f( x ) . 1 + εφ(ω + φ) εφφ l + αβ χ2 + αβ

J χ-Αρκεί, λοιπόν να προσδιορίσουμε το Μέγιστο (maximum) της f(x), που όπως παρατηρούμε είναι ορισμένη στο σύνολο των πραγματικών αριθμών και είναι παραγωγίσιμη ως πηλίκο παραγωγίσιμων (πολυωνυμικών) συναρτήσεων. Έτσι παίρνουμε : . (α - β) ( χ 2 + α β ) - 2 (α - β ) χ 2

f ( χ ) = 7

( χ 2 + αβ )-

( α - β ) ( αβ - χ 2 )

( χ 2 + αβ)2

με την πρώτη παράγωγο να μηδενίζεται όταν χ = -jc;β η χ = jc;β :::::> χ = jc;β, αφου χ > Ο. Προφανώς η γωνία ω γίνεται μέγιστη όταν χ = jc;β, δηλαδη χ 2 = α β.

Σz6λ ω : Ο προσδιορισμός του Μ ,ώστε χ 2 = αβ <:::> ΟΜ2 = ΟΑ · ΟΒ (*) , είναι απόλυτα α­κριβής, δηλαδή με κανόνα και διαβήτη και αυτό επειδή η σχέση (* ) οδηγεί στην κατασκευή κύκλου που να διέρχεται από τα σημεία Α, Β και να εφά­πτεται της Οχ σε σημείο που προφανώς είναι η θέ­ση του Μ την οποία ζητάμε (δύναμη σημείου ως προς κύκλο με την ΟΜ εφαπτομένη του). Αυτή εί­ναι μία Ιστορικής σημασίας κατασκευή καθότι εί­ναι μία από τις κατασκευές του Απολλώνι (έζησε στην Πέργη της Παμφυλίας περί το 262- 1 90 π.χ. ) . 1 2 ) Θεωρούμε την οικογένεια των συναρτήσεων fu : JR � JR , fα ( χ ) = αe' - χ2 , α > ο. Ν α αποδειχθεί ότι για κάθε συνάρτηση από αυ­τές υπάρχει ένα ακριβώς σημείο καμπής και ότι όλα αυτά τα σημεία καμπής είναι σημεία συ­γκεκριμένης γραμμής. Απ6ιiΕ ιξη : Παρατηρούμε ότι η παράγωγος της fα ,για συγκε­κριμένο α, ορίζεται στο σύνολο των πραγματικών αριθμών οπότε έχουμε f� : JR � JR, f� ( χ ) = αeχ - 2χ. 'Ομοια έχουμε την ύπαρξη της ( : JR � JR, ( ( χ ) = αe' - 2, με μονα-

δική ρίζα την I 2 ' ' λλ ' Ρα = η - στην οποια γινεται α αγη α

του πρόσημου της 2ης παραγώγου ,οπότε το σημείο

( ln � , 2 - ( ln � ) ' J = (ρ" , 2 - ρ" ' ) είναι σημείο

καμπής. Διαπιστώνουμε τώρα εύκολα ότι τα ση­μεία αυτά ανήκουν στην παραβολή y = -χ 2 + 2. 1 3 ) Θεωρούμε την συνάρτηση

f : [ο, π] � JR,f ( x) = �ημχ + �ημχ + �ημχ + .. . , με άπειρα ριζικά. Να υπολογιστεί η παράγωγος της. Λ\1ση :

f ( χ ) = �ημχ + �ημχ + �ημχ + . . . :::::> f ( x ) =

= Jημχ + f ( χ ) :::::> f2 ( χ ) = ημχ + f ( χ ) , Άρα έχουμε: ( ( χ) -f( χ) =ημχ :::::> 2f( x) f' ( χ) -f' ( χ) =

f. ( ) συνχ δ' , = συνχ :::::> χ = ( )

, με πε ιο ορισμου της 2f χ - 1

( επίσης το [0, π] , αφού 2f ( χ ) * I, για κάθε χ ε [Ο, π] . Αυτό προκύπτει από τον εξής συλλογισμό:

Αν υπήρχε 1 φ ε [Ο, π] με f ( φ) = - , θα είχαμε 2 1 1 Ι - - - = ημφ :::::> -- = ημφ :::::> ημφ < Ο που είναι ά-4 2 4 τοπο αφού : φ ε [Ο, π] :::::> ημφ ::::=: Ο .

1 4) Α) Να αποδειχθεί ότι: e' :?: χ + l, x ε JR. Β) Έστω η συνάρτηση : f : JR � JR , f ( χ) = ln2 "' + 1 (e'

' + t +

2συν2'· , + Ι ( ημιν_, + ι χ + χ2 '·, + 1 )), νι ε Ν.

α ) Να προσδιοριστεί το πεδίο ορισμού της . β) Να προσδιοριστεί η εξίσωση της εφαπτομέ­νης της γραφικής παράστασης της f στο σημείο της {Ο, f (Ο)) . Λ\Jση Α) Άσκηση 8 , σελίδα 292 του βιβλίου των Μαθη­ματικών της κατεύθυνσης. Β) α) Από την σχέση : e' > χ + Ι, χ ε JR: προκύ­πτουν τα ακόλουθα:

eχ' + Ι > Χ 2 + 2 > -2συν2ν0 + Ι ( ημ2ν3 + Ι Χ + Χ2ν, + Ι ) :::::>

:::::> eχ' + Ι + συν2ν, + Ι ( ημ2ν; + Ι χ + χ 2ν, + Ι ) > Ο .

Επομένως το πεδίο ορισμού της f είναι το JR, με την f να είναι παραγωγίσιμη στο JR σαν σύνθεση παραγωγίσιμων στο JR συναρτήσεων και πράξεων μεταξύ αυτών.

ΕΥΚΛΕΙΔΗΣ Β' 75 τ.3/64

Page 68: Ευκλειδης Β 75

------------- Μαθηματικά για την Γ Λυκείου -------------β) Παρατηρούμε ότι: f (O) = Ιη 2 ν , + Ι ( e + 2 ) . Θα προσδιορίσουμε τώρα την παράγωγο f' ( Ο) , που όπως είπαμε υπάρχει. Εδώ μπορούμε να διαπιστώ­σουμε ότι: (Vx Ε IR ) {f ( -χ ) = f ( χ ) ) ( αρτια ) . 'Ετσι παίρνουμε :

f' (O) = lim f (h + O ) - f (O )

= lim f (h ) - f (O)

= h ->0' h h ->0' h

= lim f (-u ) - f (O)

= - lim f (u ) - f (O)

= -f' (O ) , u ->0- -u u ->0- u

από όπου άμεσα προκύπτει ότι ( ( Ο) = Ο. Άρα η ε­ξίσωση είναι y = \η 2 ν , + ι ( e + 2) .

Π ροη:ινό μηα θέματα : I ) Θεωρούμε την παραγωγίσιμη στο IR συνάρ­τηση f : IR � IR για την οποία ισχύει

('11x E IR)(r2 ( x) + x2 (r ' ( x))2 + x (f 2 ( x)) ' � ι) . Να αποδειχθεί ότι η f είναι φ ρ αγμένη . \ π6δ::: ιξη : Η σχέση που είναι σε ισχύ δίνει - Ι � ( xf ( χ ) ) ' � Ι , στην συνέχεια χρησιμοποιώντας την σχέση μεταξύ του πρόσημου της παραγώγου με την μονοτονία συνάρτησης έχουμε : l f ( 0)1 :::; \και ( 'lix Ε �· ) ( l f ( χ ) I :::; Ι) � (Vx EIR) ( I f( χ )I :::; Ι) .

2 η Ν α μελετηθεί η συνάρτηση f (χ) = � . Ιη χ

Υπι'ιδt: ιξη : Θα πρέπει να ασχοληθούμε με τα εξής: α) εύρεση του πεδίου ορισμού, β) περιοδικότητα, γ) συμμετρίες (άρτια ή περιττή), δ) συνέχεια, ε) aσύμπτωτες , στ) μέγιστα - ελάχιστα - σημεία καμπής ,ζ) μονοτονία ,η) κυρτότητα ,θ) σημεία το­μής με τους άξονες ι) πίνακας καταγραφής και γραφική παράσταση .

Β - Ο ι\ Ο Κ Λ Η Ρ Ω Μ Λ 'Ι' Λ Θεωρούμε γνωστό το πρόβλημα της Π α ρ α γ ώ γ ι σ η ς μιας συνάρτησης f(x) . Υπενθυμίζουμε επίσης ότι το δ ι α φ ο ρ ι κ ό μίας συνάρτησης f(x) συμβολίζεται df(x) και ισούται με την παράγωγο της συνάρτη­σης επί dx δηλ. df ( χ ) = ( (χ ) dx , με dx aπειροστή μεταβολή του χ .Τώρα μας ενδιαφέ­ρει το α ν τ ί σ τ ρ ο φ ο πρόβλημα, δηλαδή δίνεται η παράγωγος ( (χ ) συνάρτησης f(x) και ζητάμε την f(χ).Τότε η συνάρτηση f(x) ονομάζεται αρχική ή παράγουσα (αφού παράγει την παράγωγο) ή αόριστο ολοκλήρωμα. Εδώ πρέπει να επισημά­νουμε ότι σε μία συνάρτηση f(x) aντιστοιχίζεται

μόνο μία παράγωγος ( ( χ ) .Αντίθετα σε μία πα­ράγωγο ( ( χ ) aντιστοιχίζονται άπειρες αρχικές συναρτήσεις f(x) οι οποίες διαφέρουν μεταξύ τους κατά σταθερά. Το να μας ζητήσουν να προσ­διορίσουμε την αρχική μίας συνάρτησης ή την παράγουσα ή το αόριστο ολοκλήρωμά της ,το πρόβλημα αυτό γίνεται κάτω από το σύμβολο f. Μετά το σύμβολο f ακολουθεί το διαφορικό της ζητούμενης συνάρτησης που με τύπους και ιδιότητες θα aποκρυπτογραφηθεί για να μας την προσδιορίσει. Άμεση συνέπεια είναι η : Jf' (χ ) · dx = f ( χ ) + c. Εδώ υπενθυμίζουμε ότι αν η συνάρτηση f είναι ορι­σμένη και συνεχής σε διάστημα Δ ,τότε κάθε χ συνάρτηση F ( χ ) = Jf ( t ) dt, χ ε Δ, με α στα-

α

θερά που ανήκει στο Δ ,είναι συνεχής στο Δ και είναι αρχική συνάρτηση ή παράγουσα ή αόριστο ολοκλήρωμα της f(χ) .Δηλαδή

ισχύει: F (χ ) � Ο f ( t ) dt) � f (χ ) .

Υπενθυμίζουμε επίσης ότι το ο ρ ι σ μ έ ν ο oλo­fJ

κλήρωμα Jf ( χ ) dx ( στα&ερα ακρα ) , είναι στα-α

θερή οπότε [ fr ( x ) dx) � ο. Θα ήθελα ακόμα να

αναφέρω ότι για να προσδιορίσουμε το πεδίο ορι­g( χ ) σμού της συνάρτησης F ( χ ) = f f ( t ) dt, κάνουμε

Ιι ( χ ) τις εξής κινήσεις : l η) βρίσκουμε τα πεδία ορισμού A Γ , A h , Ag των συναρτήσεων f,h,g αντίστοιχα, 2η) βρίσκουμε την τομή Α = Α1, n Ag και προσδιο-ρίζουμε εκείνα τα χ Ε Α ώστε τα άκρα h(x) και g(x) να ανήκουν στο πεδίο ορισμού ΑΓ της f , 3η) Το πεδίο ορισμού AF της F(x) αποτελείται από εκείνα τα στοιχεία που προσδιορίσαμε στην 2η κί­νηση και για τα οποία η f είναι συνεχής στο κλει­στό διάστημα που ορίζουν τα άκρα h(x), g(x).

Π uράδr.ιγμu: Για το πεδίο ορισμού της συνάρτησης

Γχ+ι dt F ( χ ) = f �· έχουμε: Το πεδίο ορισμού της

χ ' - ι ν t - 1

ΕΥΚΛΕΙΔΗΣ Β ' 75 τ.3/65

Page 69: Ευκλειδης Β 75

------------ Μαθηματικά για την Γ ' Λυκείου ------------

Ι f ( t ) = � , ειναι το Ar = ( Ι , +οο ) , της ν t - Ι

h ( χ ) = χ 2 - 1 το Ah = IR. και της g ( χ ) = Γχ+ι τοΑg = [ - Ι , +οο ) . Επομένως Α = Ah n Ag = [- Ι , +οο) .Για να ορίζεται η F πρέ­πει και αρκεί Γχ+ι > Ι και χ 2 - Ι > Ι � χ > J2 ε Α. Συνεπώς το πεδίο ορισμού της F είναι το Α F = ( J2, +οο) . " Να υπολογιστεί το ολοκλήρωμα:

I= s[�- r ' ' ( x) f2 ( x)}x. f (χ) (r ' ( x))3

Θεωρούμε ότι το ολοκλήρωμα αυτό είναι σημα­ντικό από την άποψη ότι μας δίνει δυνατότητα κα­τασκευής προβλημάτων στα ολοκληρώματα, το­ποθετώντας στην θέση της f(x) μία συγκεκριμένη συνάρτηση .

Να υπολογιστεί το ολοκλήρωμα:

J χ - 1 I = 3 ( ) 2 dχ,οταν α, β ε !R.*με α -::;:. β

χ - α + β χ + αβχ

Παρατηρούμε ότι χ 3 - (α + β) χ 2 + αβχ = χ ( χ - α) ( χ - β ) , επομένως ζητούμε πραγματικούς αριθμούς

χ - Ι κ λ μ . κ λ μ = - +--+--η ' ' ' χ ( χ - α) ( χ - β ) χ χ - α χ - β χ - Ι = κ ( χ - α) ( χ - β ) + λχ (χ - β ) + μχ ( χ - α ) ( * ) , για κάθε χ � {Ο, α, β} , οπότε έχουμε:

έχουμε : Ιίm ( χ - Ι ) = καβ � κ = --Ι , χ �ο αβ

α - Ι Ιίm ( χ - Ι ) = λα (α - β) � λ = ( ) ' χ � α α α - β

Ιίm(χ - Ι) =μαβ�μ r- I )

' Εύκολα διαπιστώνε-·� β β-α ται ότι επαληθεύουν την (* ) . Επομένως έχουμε

{ Ι α - Ι β - 1 } Ι= Jl- αβχ + α(α - β) ( χ - α)

+ β (β - α) ( χ - β)

χ =

Ι J 1 ct α - 1 J� β - 1 J� -αβ -; χ +

α (α - β ) χ - α χ +

β (β - α) χ - β χ =

--1 Ιn l x l + (α - Ι

)Ιn l x - αl + r - \

)Ιn l x - b l + c.

αβ α α - β β β - α _ , ; Να αποδειχθεί ότι

2 J dx χ ( 2 ι ) J dx ν ( 2 ) ν+ Ι

= ( 2 ) ν

+ ν -(

2 ) ν ' χ + 1 χ + 1 χ + 1 για κάθε φυσικό αριθμό ν.

Παρατηρούμε ότι το ολοκλήρωμα του 1 ου μέλους με το ολοκλήρωμα του 2"" μέλους αναφέρονται σε συναρτήσεις που η μορφή τους μας επιτρέπει να συμβολίσουμε f dx J dx . Ι ν = οποτε Ι ν+ 1 = 1 • Αρκει λοι-

( χ 2 + 1 ) ' ( χ 2 + 1Γ πόν να αποδείξουμε ότι

χ 2νl ν+ Ι = ν + (2ν - Ι ) Ι ν . ( χ 2 + 1 )

Σε τέτοιες περιπτώσεις ,συνήθως χρησιμοποιούμε την μέθοδο της ολοκλήρωσης κατά μέρη (ή κατά παράγοντες),οπότε παίρνουμε

lν = fx' ( 2

I γdχ =

( 2 χ

γ- Jx[

( ,

I )ν ]'dχ(*) . χ + Ι χ + Ι χ- + I

Όμως έχουμε : [ 1 ] - - - 2νχ (πα-( χ 2 + Ι )'

- . . . -( χ 2 + 1 )

ν+ Ι

ράγωγος πηλίκου ) .Από την σχέση (*) παίρνουμε : χ χ 2 I = + 2ν J dx = ν

( χ 2 + 1 )' ( χ 2 + ΙΓ•

χ + 2ν J χ 2 + I - I dx = ( χ2 + 1 )' ( χ 2 + Ι Γι

χ + 2ν J 1 dx - 2ν J Ι dx = ( χ 2 + 1 )' ( χ 2 + Ι )' ( χ 2 + ΙΓ•

ΕΥΚΛΕΙΔΗΣ Β ' 75 τ.3/66

Page 70: Ευκλειδης Β 75

------------ Μαθηματικά για την Γ Λυκείου -----------­

χ ---ν + 2νlν - 2νl ν+ ι . ( χ 2 + 1 ) Από εδώ προκύπτει εύκολα:

Ι! ; Η παρουσία του e' σαν παράγοντα σε γινόμενο μας κάνει να σκεφτούμε το πλεονέκτημα που έχει, ανά πάσα στιγμή να συμπεριφέρεται και σαν πα­ράγωγος του εαυτού του καθότι ισχύει ( e' )' = e' , οπότε έχουμε την δυνατότητα να θέ­τουμε σε ισχύ ισότητες του τύπου : e 'f ( x ) + e' ( ( χ ) = ( e' f ( χ ) )' ( i ) , χ f ( χ ) - ( ( χ ) = ι�J· ( " " ) e , ( ) ( ) 1 1 , r - χ r χ f' ( x ) - f ( x ) = e' f' ( x ) - e' f ( x ) = [ f ( x ) ) ' ( i i i ) e·' e2 ' e' '

e' (f ( x ) + f ' ( x ) ) g ( x ) - f ( x ) g ' ( x ) = g" ( x ) = (e' f ( x ) ) , g ( x ) - e ' f ( x ) g' ( x ) - ( e 'f ( x ) ) ' . 2 ( ) - ( ) ( ιν ) , g χ g χ και έτσι μπορούμε να προσανατολιζόμαστε σε μορφές αρχικών συναρτήσεων -� , Υπολογίστε το ολοκλήρωμα:

J χ χ 2 + χ + 1 d e z χ . ( χ + 1)

Λαμβάνοντας υπ ' όψη το μεθοδολογικό σχόλιο 1 , έχουμε: e' ( χ 2 + χ + 1 ) e' ( χ 2 + 2χ + 1 - χ ) ") = ') = ( χ + 1 )- ( χ + 1 )-

- e' [ ( χ + I )2 - χ]

_ e' ( χ + 1 ) ( χ + 1 ) - xe' - ( χ + 1 ) 2 - ( χ + 1 )2

= (e' x ), ( x + 1 ) - (e' x ) ( x + 1 ) , = [ xe' ) ' � ( χ + 1 ) 2 x + l f( xe' ) ' xe' -- dx =--+ c. χ + 1 x + l = Θεωρούμε συνάρτηση f : JR+ � JR που είναι δύο φορές παραγωγίσιμη στο JR+ . Θεωρούμε

επίσης ότι ισχύει: ('ν' χ Ε JR ) ( f" ( Χ) = ln ( Χ + 1 )) . Α ν υποθέσουμε ότι η γραφική παράσταση της f περνά από το σημείο (0,1 ) και στο σημείο αυτό εφάπτεται της γραφικής παράστασης της g : JR � JR, g ( χ) = χ 2 - α, όπου α πραγματική σταθερή, να υπολογιστεί το ολοκλήρωμα:

ι

I = J(r (x) - g ( x))dx. ο

Θα προσδιορίσουμε τους τύπους των f, g και στην συνέχεια το Ι . Από τα δεδομένα παρατηρούμε ότι: g (Ο ) = 1 � α = - 1 οποτε g ( χ ) = χ 2 + 1 . Παρατηρού-με επίσης ότι: g' ( χ ) = 2χ � g' ( 0) = 0 ( 1 ) . Για τον προσδιορισμό του τύπου της f έχουμε : η χ) = J( ( x)dx = JΙη( χ + 1) dx = J( χ + ι)Ίη( χ + Ι ) dχ = ( χ + 1 ) Ιη ( χ + 1 ) - J dx = = (χ + 1) Ιη ( χ + 1 ) - χ + C ι . Από τα δεδομένα του προβλήματος κατα-( ι J νοούμε ότι: f' ( O) = g' (0) = 0 � c ι = Ο � ( ( χ ) = (χ + 1 ) !η ( χ + Ι ) - χ . Συνεχίζοντας έχουμε: f ( x ) = Jf' ( x ) dx = J(χ + Ι ) Ιη ( χ + Ι)dχ - Jxdx = = J�[( x + 1/ ] ιη ( x + l ) dx - � x2 = = _!_( χ + 1 )2 Ιη ( χ + Ι ) _ _!_ J( χ + 1 )2 �χ _ _!_χ2 = 2 2 χ + 1 2 _!_ ( χ + 1 ) 2 Ι η ( χ + 1 ) -_!_ J( χ + I )dx 2 2 1 ο 1 ( )2 ( 3χ 2 χ -- χ- = - x + l Ιη x + l ) ---- + c με 2 2 4 2 '

f (Ο ) = 1 � c = I ( αφού η γραφική παράσταση της f περνά από το σημείο (0, 1 ) ) . Τελικά παίρνουμε: ( ) 1 ( )2 3χ 2 χ f χ = - χ + 1 Ι η ( χ + 1 ) ---- + I � 2 4 2 ( ) ( ) I 2 7χ 2 χ � f χ - g χ = -(χ + Ι ) Ιη ( χ + 1 ) -- - - . 2 4 2 1 ι ? 7 ι ? 1 ι

1 = - J( x + lγ lη ( x + 1 ) dx - - Jx -dx - - Jxdx = 2 0 4 0 2 0 ι ,

= _!_ fΓ( χ + Ι γ ] Ιη ( χ + Ι ) dχ -2[χ3 Τ _ _!_[χ 2 Τ = 6 � L 1 2 ο 4 ο

1 [( 3 ( J ι 1 ιr ? 5 = - χ + I ) !η χ + 1 ) - - j ( χ + I γ dx - - = 6 ο 6 ο 6

= 4 1η 2 _ _!_[(χ + 1 )3 ] ι - � = . . . = 1 2 Ιη 2 - Ι 1 _ 3 1 8 ο 6 9

ΕΥΚΛΕΙΔΗΣ Β ' 75 τ.3/67

Page 71: Ευκλειδης Β 75

------------- Μαθηματικά για την Γ Λυκείου ------------6)«Υπάρχει συνάρτηση f : [α, β] � JR, με α < β, συνεχης στο διαστημα [α, β] και

{ V' χ ε [α, β]) { f ( χ) > Ο) , με β β β Jr ( x)dx = κ2 , Jxr ( x )dx = κλ, Jx2f ( x )dx = λ2 , α α α

όταν κ, λ μη μηδενικές σταθερές ».

Να χαρακτηρίσετε τον παραπάνω συλλογισμό χρησιμοποιώντας έναν από τους χαρακτηρι­σμούς ΣΩΣΤΟ ή ΛΑΘΟΣ, αιτιολογώντας τα ε­πιχειρήματα σας. ΛίJση : Έστω ότι υπάρχει μία τέτοια συνάρτηση f. Παρα­τηρούμε ότι:

β β β κ2 Jx 2f ( χ ) dx + λ2 jf ( χ ) dx - 2κλ Jxf ( χ ) dx =

α α

κ2λ2 + κ2λ2 - 2κ2λ2 = Ο => β β

α

J( κ2χ2 +A_l -2κλχ ) f( x}Jx = 0=> j( κχ -λ)2 f( x}Jx = 0. α α

Θετουμεg( χ) = ( κχ -λ)2 f( χ) . Έχουμε : g ( χ ) � Ο, 'i:ix ε [α, β] . Τότε προκύπτει ό­

β τι: Jg ( χ )dx = Ο, με g ( χ ) � Ο, με την g(x) συνεχή

α

στο διάστημα [α, β] .Από γνωστό θεώρημα αυτό συμβαίνει μόνο όταν g ( χ ) = Ο, για κάθε χ ε [α, β] , πράγμα άτοπο αφού η εξίσωση g(x)=O

έχει ως μοναδική ρίζα την χ = .?:. . Τελικά ο συλλο-κ

γισμός χαρακτηρίζεται ως ΛΑΘΟΣ. Το γνωστό θεώρημα είναι το εξής: Έστω g συ­νεχής συνάρτηση σε διάστημα (α, β] . Αν {'tχ ε [α, β]) (g ( χ) � ο) και η συνάρ­τηση g δεν είναι παντού μηδέν στο διάστημα

β αυτό ,τότε Jg ( χ )dx > Ο.

α

Λ ναφομά σε ανισ6τητες και ολοκλη ριJψατα : α) Υπενθυμίζουμε ότι η ολοκλήρωση (κατά Rie­mann σε διάστημα [α, β]) των μελών μίας ανισό­τητας δεν αλλάζει την διάταξη της ανισότητας. β) Το θεώρημα που αναφέραμε στο προηγούμενο πρόβλημα οδηγεί στην εξής διαπίστωση : Αν έχου­με τις συναρτήσεις

f : [α, β] �JR,g : [α, β] �JR(α < β) : ('i:ix ε [ α, β])

(f ( x) � g( x)) και (ΞJχ0 ε [α, β]) (f (χ0 ) * g(χ0 )) , τότε στην περίπτωση αυτή ισχύει :

β β jf ( χ ) dx > Jg ( χ ) dχ(αυστηρα μεγαλυτερο). α α

I

7) Να αποδειχθεί ότι 3 J 1 dx <2. ο ν χ3 - 3χ + 4

Απόδειξη : Η συνάρτηση h ( χ ) = χ3 - 3χ + 4, είναι συνεχής στο διάστημα [0, 1 ] , με h(0)=4>0, h( 1 )=2>0 οπότε O<h( l )< h(O) και h ' ( x ) = 3 ( x - l ) ( x + 1 ) < 0, οταν χ ε (Ο, 1 ) , οπότε η h είναι γνησίως φθίνουσα στο [0, 1 ] που σημαίνει ,τελικά ότι h(x)>O, για κάθε στοιχείο χ του διαστήματος [0, 1 ] .Παρατηρούμε ό-{ (vx ε [0, 1 ] ) ( 4 - 3χ + χ3 � 4 - 3χ > ο ) => τι:

=> (vx ε [0, 1 ] ) ( 1 1 � �J ν 4 - 3χ + χ3 ν 4 - 3χ

με την ύπαρξη χ0 ε (Ο, 1 ) (π.χ. χ0 = .!_), 2

με 4 - 3χ0 + χ� > 4 - 3χ0 ( αυστηραμεγαλυτερο ) . Με ολοκλήρωση λοιπόν παίρνουμε:

Jι dx J1 dx J1 dx -;===== < με =

o .Jx3 - 3x + 4 0 �4 - 3χ ' 0 �4 - 3χ

= -3.[�4 - 3χ ] ι =

3. => Jι dx <3.

3 ° 3 o .Jx3 - 3x + 4 3 'Η 3 J dx < 2 .

o .Jx3 - 3x + 4 e·'

8 ) Να αποδειχθεί ότι 3 J dx < 2eJ . .π. ln χ e .

Απόδειξη : Θέλουμε να αποδείξουμε ότι:

� e3 3 3 f dx 2 3 , J dx 2 3 2e - < e η - < -e = -- .

.π. Ι η χ .π. Ι η χ 3 Ι η e3 e · e ·

Εδώ διαπιστώνουμε ότι το e3 εμφανίζεται σε τρία σημεία της προς απόδειξη ανισότητας, οπότε το γεγονός αυτό σε συνδυασμό με την μορφή της μας οδηγεί στο να θεωρήσουμε την συνάρτηση :

f : JR: � JR, f ( x ) = 2χ _ J� => f' ( x ) = ln x,- 2 .

ln x ji. ln t ιη - χ e .

Άρα: χ > eJ5 > e2 ( 5 > 4 => J5 > 2 => eJ5 > e2 ) => ln χ - 2 > Ι η e2 - 2 = Ο, που σημαίνει ότι η f θα είναι γνήσια αύξουσα στο

2 J5 eJ\ <e1 διάστημα [eJ5 , +oo ) , με f (eFs ) = Js > 0 =>

ΕΥΚΛΕΙΔΗΣ Β' 75 τ.3/68

Page 72: Ευκλειδης Β 75

------------- Μαθηματικά για την Γ Λυκείου ------------3 � � 3 ( J5 ) ( 3 ) 2e J dt J dt 2e O < f e < f e = -- - => - < - =>

3 .;; ln t J5 Ιη t 3 c'

3 J dx < 2e3 . J5 ln χ e .

c c

9) Θεωρούμε την μη σταθερή συνάρτηση f : [α, β] � IR (α < β) , με f ( [α, β]) ς ( -1, 1 ) που είναι συνεχής στο Ι α, βl και για την οποία ισχύει β Jr (x)dx = O (* ) . Aν η f έχει σαν ελάχιστη τιμή α

την μ και σαν μέγιστη τιμή την Μ , να αποδει­Ρ

χθεί ότι: Ο < J f4 (χ ) dx s; (α - β) μΜ. α

Απόδειξη : Αφού η συνάρτηση f είναι συνεχής στο διάστημα [α, β] θα είναι συνεχής συνάρτηση και η f" , με ν ε Ν = {2 ,3 , . . . } , στο ίδιο διάστημα. Γνωρί­ζουμε επίσης ότι η f δέχεται ελάχιστο (minimum) μ και μέγιστο (maximum) Μ που δεν ταυτίζονται καθότι η f δεν είναι σταθερή . Αν μ 2 Ο τοτε f ( χ ) 2 Ο, μ 2 Ο => f ( χ ) 2 Ο, οπότε από γνωστό θεώρημα και την σχέση (*) παίρνου­με : {Vχ ε [α, β] ) { f ( χ ) = Ο) , πράγμα άτοπο αφού η f είναι μη σταθερή συνάρτηση, άρα μ<Ο . Με ίδιο σκεπτικό καταλήγουμε στο συμπέρασμα Μ>Ο.Άραυπάρχουν χ 1 , χ 2 Ε [ α , β] , ωστε ( \f χ ε [α, β ] ) ( -1 < μ = f ( χ 1 ) ::::; f ( χ ) ::::; f ( χ 2 ) = Μ < 1 ) Έτσι έχουμε: ( f( χ) -μ) ( f( χ) -Μ) :::; O=>f2 ( χ) -(μ+Μ) f( χ) +μΜ:::; Ο

=>f2 ( χ) -(μ+Μ) f( χ) :::;-μΜ=> β β β Jr2 ( χ )dx - (μ + Μ) Jr ( χ )dx ::::; - JμMdx => α α α

β => Jr2 ( x)dx s; (α - β ) μM.

α

Όμως f { [ α, β] ) ς ( - Ι , Ι ) => f2 ( χ ) - Ι s; Ο => => f2 ( χ ) ( f2 (χ ) - 1 ) ::::; ο => f4 ( χ ) s; f2 ( χ ) => β β Jf4 ( x)dx :::; Jf2 ( χ)dχ :::; (α - β ) μΜ ( Ι ) . Ισχύει επί-α α

σης ότι: (ΞJχ0 ε [α, β] ) { f ( χ0 ) :;t: Ο) , καθότι η f δεν είναι σταθερή . Οπότε από γνωστό θεώρημα έχουμε

β την σχέση Ο < J f4 ( χ )dx ( 2) .

α

β Από τις ( Ι ) , ( 2 ) => 0 < Jr4 ( x)dx s; (α - β ) μM.

α

ι 0 ) Έστω οι θετικοί πραγματικοί αριθμοί x,y με

y<x. Να αποδειχθεί ότι: ( e:y ) ' > ( e;x )Υ Απόδηξη :

χ ν

Αρκεί να αποδείξουμε ότι � e2 Y > e2x Lή αρκεί Υχ Χ )

χ ? χ - ' ' χ χ -- > e-( Υ) η αρκει ln- > 2--y , ( )x+y Υ Υ x + y

ή τελικά ,αν θέσουμε t = � <:::::> χ = ty ( t > Ι ) , αρ­Υ

t - I κεί να αποδείξουμε ότι : ln t > 2 -- ή αρκεί t + I

1 2 - ln t > l - - ή 2 t + 1

αρκεί - [ Ιη u Τ > - -- ή Ι [ 2 ] ι 2 I U + Ι I

, 1 1Jdu 1J

du , αρκει- - > 2 2 ( * ) η 2 I U I (u + 1 ) τελικά αρκεί να

' ' 1 2 αποδειξουμε οτι : - ;::; 2 2u ( u + 1 )

που είναι αληθής πρόταση όταν u Ε [ 1, t] , με t > 1 . Η σχέση (* ) ισχύει καθότι για το τυχόν

Ι 2 Χ 0 Ε ( 1 , t ) => - > 2 • 2χ0 ( χ0 + 1 )

Μ εΟοδολογικό σχόλιο2 : Θα πρέπει να γνωρίζου­με το λήμμα που ακολουθεί :

β β Ισχύει ο τύπος: Jr ( χ ) dx = Jr ( α + β - χ ) dx .

α α

Η ισότητα αυτή αποδεικνύεται εύκολα, αν θεωρή­σουμε τον μετασχηματισμό u = α + β - χ => dx = -du και ξεκινήσουμε από το 2° μέλος κάνοντας ταυτόχρονα αλλαγή των άκρων ολοκλήρωσης . . . . . . . ι ι ) Να υπολογιστεί το ολοκλήρωμα

I = J f(x) dx . 0 f(α - x) + f(x)

Υπόδειξη : Να χρησιμοποιηθεί άμεσα το προηγού­μενο μεθοδολογικό σχόλιο . Απάντηση : Ι = � -

2 1 2 ) Θεωρούμε την f : 1R � JR. Αν υποθέσουμε

συνεχή στο IR συνάρτηση

ότι η συνάρτηση

ΕΥΚΛΕΙΔΗΣ Β' 75 τ.3/69

Page 73: Ευκλειδης Β 75

------------- Μαθηματικά για την Γ Λυκείου -----------­

g ( χ) = χ' τ f (-2 t_) dt, παρουσιάζει τοπικό α-

1 χ + 1 κρότατο στη θέση 1 να αποδειχθεί ότι I ( 1 ) 1 Jr2 ( x)dx � f - - - . 1 2 2 Γνωρίζουμε ότι η g είναι παραγωγίσιμη στο JR συ­νάρτηση . Από το θεώρημα του Fenηat έχουμε την ισότητα g. ( 1 ) = Ο . Θεωρούμε ότι:

χ 2 t+ 1

= u => t = ( χ2 + I ) u => dt = ( χ 2 + 1 ) du.

Για t = I => u = -? 1- , t = χ 2 + 1 => u = 1 . Άρα:

χ - + I ι x ::? + l g ( x ) = ( x 2 + 1 ) J f ( u ) du = - (x 2 + 1 ) J f (u ) du => ι ι g' ( χ ) = -2χ 'Ί ' f ( u ) du + �f (-?-1-) .

1 Χ + I χ - + 1 Από την σχέση g' ( I ) = Ο παίρνουμε ι ι 1 ( I ) 2 2 -f - = Jf (u ) du = Jf ( x ) dx . Από 2 2 ι ι ι ι ι J( f ( x ) + 1 )2dx � 0 => Jf2 ( x ) dx + 2 Jf ( x ) dx + ι ι ι - -2 2 2

2 2 1 ( 1 ) Ι Jf2 ( x ) dx � f - -- . ι 2 2

� 3 \ Δίνεται η συνεχής στο σύνολο JR συνάρτηση h : JR � JR και οι αριθμοί κ, λ Ε JR με lκ l * l λ l . Θεωρούμε την ύπαρξη συνάρτησης f : JR � JR για την οποία ισχύει:

( "d χ Ε JR) ( κf (χ - α) + λf ( β - χ ) = h ( χ)) ( * ) ,

να αποδειχθεί ότι : Η f είναι συνεχής στο σύνολο των πραγματι­κών αριθμών. β 1 β : ; ' J f (χ - α )dx =- J h (χ )dx. u κ + λ α

r� ' Υπολογίστε το J1 2.Jx2 - 3χ + 2 - .Jx2 + χ ο 3,_/-χ2 + χ + 2 dx.

ολοκλήρωμα:

Α) i ) Αν στην (* ) όπου χ θέσουμε α+ β -χ παίρ­νουμε την σχέση : κf (β - χ ) + λf ( χ - α) = h (α + β - χ ) , "d χ ε JR ( * * ) . Λύνοντας το σύστημα των (*), (* *) εύκολα έχουμε: f ( χ - α) = 2

κ 2 h ( χ ) - 2

λ 2 h (α + β - χ ) . Από κ - λ κ - λ

την τελευταία αυτή σχέση προκύπτει ότι η f είναι συνεχής ( ;) . i i ) Επίσης από την τελευταία σχέση παίρνουμε ότι:

β κ β λ β Jf(x-α)dx= � -lt Jh(x)dx- κ2 -/t Jh(α+β-x)dx, α α α

οπότε λαμβάνοντας υπ' όψη το μεθοδολογικό σχό-β I Ρ λιο2, τελικά έχουμε: f f ( χ - α )dx = -- f h (χ )dx. α κ + λ α

Β) Έστω η συνεχής συνάρτηση στο διάστημα [0, 1 ]

( ;), r , [ο, ι ] --> ΙR, r ( χ) = z.Jx' �3' + 2 -� "" 3 -χ2 + χ + 2

=> f ( x ) = 3_�1 - x _ }_� χ . 3 1 + χ 3 2 - χ

Θεωρώντας τώρα την συνάρτηση

h : [0, 1 ] � JR, h ( χ ) = J§, έχουμε : 2 1 f ( x ) = f ( x - 0) = 22 _ 1

h ( x ) -2" _ 1

h ( 0 + 1 - x ) . ι I ι 1 '{§-Επομένως ισχύει: Jf( x) dx =- Jh( x)dx =- f �dx, 0 3 0 3 0 1 + χ

οπότε αρκεί να υπολογίσουμε το ολοκλήρωμα του δεύτερου μέλους. Αφού έχουμε Ο ::;; χ ::;; I , μπορού-με να θέσουμε χ = ημt => dx = ( συνt ) dt =>

π

fl J§- Χ χ �Ο=>ι�ο 2J 1 - ημt -dx = -- (συνt ) dt = 0 1 + Χ χ � Ι =>ι�.": 0 1 + ημt

2 π π - -2 1 ημt " π J-

--(συνt ) dt = J( I - ημt ) dt = . . . = - - 1 .

0 συνt 0 2 ι Τελικά παίρνουμε: f f ( χ ) dx = 2: - }_ .

ο 6 3

1 : ! Έστω συνάρτηση f : JR � JR συνεχής στο JR , ώστε r(x- l) = ''J' r( 2 t )dt - 'J[(2t+3) Jr(θ)dθ]dι

ο χ + x+ l ο ο Να υπολογιστεί το εμβαδό του χωρίου που πε­ρικλείεται μεταξύ της γραφικής παράστασης της συνάρτησης h : JR � JR,μεh( χ) = ( χ2 + χ + ι) r ( χ- l)καιh( ο) = ι,

ΕΥΚΛΕΙΔΗΣ Β' 75 τ.3/70

Page 74: Ευκλειδης Β 75

------------ Μαθηματικά για την Γ Λυκείου ------------των ευθειών χ = α, χ = β, με Ο < α < β και του ά- 3

χ με lim � = +οο � lim Jeι' dt = +οο. ξονα Οχ. χ ->+>:> 3 x-->+oc 0

Παρατηρούμε ότι:

Για t = Ο � u = Ο, t = χ 3 - Ι � u = χ - 1, οπότε έ-χουμε την ισχύ της ισότητας που ακολουθεί:

f( x- I) = 'J (x2 + x + I)f(u)ctu-

'J[(2t +3) Jf(θ)dθ}� ο ο ο

f( χ - Ι ) = ( χ2 + χ + Ι ) Ί f( u)du -'J [(2t +3) Jf(θ)dθ}t

ο ο ο

χ- Ι

� f' ( x - I ) = (2x + I ) J f (u)du + ο

χ - Ι + (x2 + x + I ) f ( x - I ) - ( 2x + I ) J f (θ)dθ �

ο

{ ( χ - Ι ) = ( χ 2 + χ + Ι ) f ( χ - Ι ) � f' ( x - I ) - (x 2 + x + I ) f ( x - I ) = O �

e 'i-f·γ ( χ - ι ) - ( e 'i f ' } f ( χ - ι )

( χ ' , : \ :

= ο � e τ :- ' j

Έτσι έχουμε : χ·' χ :! h ( x ) = c ( χ 2 + χ + I ) e 3+2-- χ , με h (Ο ) = Ι � c = Ι �

=> h {ψ (χ ' + χ + l ) e'i+ ' + 'i+') Άρα το ζητούμενο εμβαδό είναι

β β� χ 3 χ ' \ β3 β' α3 α' Ε = fh ( x)dx = ! le3+2+x ) dx = e3+2+P - e3+2+α .

( χ )� J Να υπολογιστεί το όριο : ,Il� feι' dt

Αποδεικνύεται ότι: (Vx ε IR ) ( ex ;::: χ + Ι ) (Άσκη­ση 8 , σελίδα 292 του βιβλίου των Μαθηματικών της κατεύθυνσης) . Έτσι έχουμε:

χ >Ο χ χ 3

x ε!R�e· :::: χ + Ι �/ :::: t2 + l > t2 � f/dt > Jt2dt =� , ο ο 3

Συνεπώς καταλήγουμε στην επίλυση που ακολου-

. ιη J•' . dt (;)

. ( ιψ ' dι} θεί : lιm ο = lιm ,

Χ -> +>: χ - X -HOC (χ2 ) (�) (�)

= I im _e_x __ = Iim __

2_xe_x_' --x-->+z 2χ Jeι ' dt x-->+oc 2 Jeι' dt + 2xex'

ο ο

ι

lim ( xs· e1 ' dt)� = e. χ---++Χ> ο

: Ι φ ο . " ) θεωρούμε τις συναρτήσεις f : [α, β] � IR, g : [α, β] � JR, συνεχείς στο διά-στημα Ια, β] και {'v'χ ε [α, β]) (r ( χ) , g ( χ) > Ο) . Να αποδειχθεί ότι:

f {ξ) {3ξ ε (α, β}) ξ Jr ( x) dx α

β g (ξ) = 1

Jg (x) dx ξ

'Εστω : F (α, β) -> IR, F( χ) = e-' σf( t)dt ng( tflt)=>

� F ( α) = F (β) = Ο. Εδώ για την F εφαρμόζεται το θεώρημα Rolle . . . " ΆΈστω η συνάρτηση f : [α, β] � IR,α < β, με r {[ α, β]) = lf ( α ) , f ( β)J ,

που είναι γνησίως αύξουσα και παραγωγίσιμη στο διάστημα Ια,β Ι . Λαμβάνοντας υπ' όψη ότι η αντίστροφή της Γ1 : [f (α) , f ( β)J � [α, β] , είναι παραγωγίσιμη στο διάστημα [r (α) , f (β)] , να αποδειχθεί ότι:

β f{β) Jr ( x) dx + J Γ1 ( χ)dχ = βf ( β) - αf (α) . α r(α)

t = f ( χ ) � χ = f-- 1 ( t ) � dx = [ Γ1 ( t )} dt. . . . . και β β J f ( χ ) dx = J f ( u ) du = . . . (βουβή μεταβλητή) α α

ΕΥΚΛΕΙΔΗΣ Β ' 75 τ.3171

Page 75: Ευκλειδης Β 75

Επιμέλεια: Γιάννης Στρατής - Βαγγέλης Ευσταθίου

Η διδασκαλία της συνεπαγωγής, στους μαθητές Λυκείου Γ.Σ. Τασσόπουλος Καθηγητής Μαθηματικών Βαρβακείου Λυκείου

Α ν θεωρήσουμε δυο προτάσεις p, q, τότε εύ­κολα μπορούμε να δώσουμε στους μαθητές να κα­ταλάβουν την έννοια της σύζευξης (p Λ q), καθώς και την έννοια της διάζευξης (p ν q), αυτών των προτάσεων. Ως γνωστόν η σύζευξη αληθεύει μό­νον όταν και οι δυο προτάσεις p, q αληθεύουν, ενώ η διάζευξη αληθεύει μόνον όταν αληθεύει μια τουλάχιστον από τις p, q .

Επίσης εύκολα μπορεί να οριστεί η άρνηση -μιας πρότασης p που συμβολίζεται με p και είναι αληθής μόνον όταν η p είναι ψευδής. Είναι δε - -προφανές ότι η άρνηση της p , δηλαδή η p είναι η ίδια η p. (Αρνούμαι ότι δεν ήμουν παρών, σημαί­νει ότι ήμουν παρών) .

Η έννοια όμως που είναι αρκετά δύσκολο να γίνει κατανοητή από τους μαθητές, και όχι μόνο, είναι η έννοια της συνεπαγωγής με υπόθεση p και συμπέρασμα q, που συμβολίζεται με (p => q) και αποδίδεται με την έκφραση "Α ν p τότε q" .

Η προσπάθειά μας λοιπόν αφορά στο πώς θα μπορέσουμε όχι μόνο να καταστήσουμε κατανοη­τό τον ορισμό αυτής της έννοιας αλλά και στο πώς θα γίνει ο ορισμός λογικά αποδεκτός, θα συμφωνεί δηλαδή με την κοινή λογική, αν και αυτό δεν είναι πάντοτε απαραίτητο στα Μαθηματικά.

Ισχυρίζεται κάποιος ότι για οποιονδήποτε άνθρωπο Α ισχύει το εξής: «Α ν ο Α κατοικΕί στην Πελοπόννη­σο, τότε ο Α κατοικεί στην Καλαμάτα».

Έχουμε τότε μια συνεπαγωγή (p => q), όπου p, q είναι αντιστοίχως οι προτάσεις :

p : ο Α κατοικεί στην Πελοπόννησο q : ο Α κατοικεί στην Καλαμάτα Για να ανατρέψουμε έναν τέτοιο ισχυρισμό,

δηλαδή να αποδείξουμε ότι αποτελεί ψευδή πρό­ταση , είναι λογικά αποδεκτό, πως αυτό επιτυγχά­νεται μόνον αν διαπιστώσουμε ότι υπάρχει περί­πτωση να είναι (η p αληθής) και (η q ψευδής) .

Στο παράδειγμά μας πράγματι ο ισχυρισμός αυ­τός αποτελεί μια ψευδή πρόταση, αφού υπάρχει πε­ρίπτωση ο Α να κατοικεί στην Πελοπόννησο, χωρίς να κατοικεί στην Καλαμάτα, π.χ. να κατοικεί στην Τρίπολη. Κάποιος άλλος ισχυρίζεται ότι:

«Α ν ο Α κατοικεί στην Καλαμάτα, τότε ο Α κατο ικεί στην Π Ελοπόννησο».

Έχουμε τότε μια αληθή πρόταση , διότι δεν υ-

πάρχει άνθρωπος που κατοικεί στην Καλαμάτα χωρίς να κατοικεί στην Πελοπόννησο .

Αφού λοιπόν θεωρήσαμε λογικό να δεχτούμε ότι η μόνη περίπτωση που είναι ψευδής η πρόταση (p => q) είναι όταν ( η p αληθής) και (η q ψευδής), οφείλουμε να δεχτούμε ότι σε όλες τις άλλες περι­πτώσεις θα είναι αληθής. Δεχόμαστε λοιπόν ότι η πρόταση (p => q), θα είναι αληθής, 'μόνο στις πε­ριπτώσεις :

I ) · η p αληθής και η q αληθής } 2) · η p ψευδής και η q αληθής ( Ι ) , 3 ) · η p ψευδής και η q ψευδής

αφού στην περίπτωση που είναι : (η p αληθής) και (η q ψευδής) δεχτήκαμε ότι η πρόταση (ρ => q) εί­ναι ψευδής. Παρατηρείστε ότι η παραδοχή (Ι) γράφεται συντομότερα ως διάζευξη : (η p ψευδής) ή (η q αληθής). Συμβολικά: (Ρ ν q ) .

Σύμφωνα με αυτή την παραδοχή (ορισμό), η πρόταση: <<Α ν ο Α κατοικεί στην Καλαμάτα, τότε ο Α κατοικεί στην Πελοπόννησο», αληθεύει για κάθε άνθρωπο Α, είτε αυτός κατοικεί στην Καλαμάτα είτε όχι, δηλαδή είτε είναι αληθής η υπόθεση είτε όχι, όπως ακριβώς η συνεπαγωγή: <<Αν κόψω

το χέρι μου τότε θα ματώσει» είναι προφανώς αληθής ανε­ξαρτήτως του αν έκοψα το χέρι μου ή μάτωσε με άJJ..o τρόπο. Χαρακτηριστικό δε παράδειγμα συνεπαγωγής με ψευδή υπό­θεση αποτελεί η απλή λαϊκή έκφραση «Α ν εσύ είσαι Δεσπό­της, τότε εγώ είμαι Πρωθυπουργός», που χρησιμοποιείται όταν είναι γνωστό ότι ο πρώτος δεν είναι Δεσπότης (ψευδής υπόθεση). Φαίνεται ότι ο απλός λαός έχει κατανοήσει πλήρως την έννοια της συνεπαγωγής πρίν οι "σοφοί" δάσκαλοι επι­χεφήσουν να του την διδάξουν.

Δ ιπλt1 συνεπαγωγή t1 ισοδυναμία Με το σύμβολο (p <=> q), εννοούμε προφανώς

πλέον τη σύζευξη : [(p => q) Λ (q=>p)], των δυο συ­νεπαγωγών (p => q), ( q=>p ). Επομένως η πρόταση που αποδίδεται με το σύμβολο (p <=>q) οφείλουμε να δεχτούμε πως είναι αληθής, μόνον όταν αληθεύ­ουν και οι δυο συνεπαγωγές (p => q), ( q=>p ), δηλαδή μόνο στις περιπτώσεις που είναι και οι δυο προτά­σεις p, q αληθείς ή και οι δυο ψευδείς, αφού στις υ­πόλοιπες περιπτώσεις θα είναι προφανώς ψευδής.

Πράγματι: Αν (η p είναι αληθής και η q είναι ψευδής), τότε δεν αληθεύει η συνεπαγωγή (p => q), ενώ αν (η q αληθής και η p ψευδής), τότε δεν αληθεύει η συνεπαγωγή ( q=>p ). Η πρόταση

ΕΥΚΛΕΙΔΗΣ Β' 75 τ.3/72

Page 76: Ευκλειδης Β 75

Το Βήμα του Ευκλείδη

(p <::::> q), λέγεται ισοδυναμία ή διπλή συνεπαγω- •:• Η άρνηση δηλαδή της σύζευξης (α=Ο και β=Ο) γή των p, q και αποδίδεται με την έκφραση (p αν είναι η διάζευξη (α ;t. Ο ή β ;t. Ο) και μόνον αν q) . , (-) (- -) Για να γίνουν κατανοητά τα παραπάνω, παραθέτουμε Συμβολικα : Ρ Λ q <=> Ρ ν q , (I) μερικά παραδείγματα συνεπαγωγών (απλών ή διπλών), Επίσης: α . β = Ο <=> α = Ο ή β = Ο των οποίων η αλήθεια ή το ψεύδος θα πρέπει να αιτιολο­γηθεί αναλυτικά στους μαθητές από τον διδάσκοντα. Παμό.δηγμα I 1 . 5>3�4>2 (α) 3 . 5<3�4>2 (α) 2 . 5>3�4<2 (ψ) Π αριiδειγμα 2

4. 5<3�4<2 (α)

1 . 5>3<::::>4>2 (α) 3 . 5<3<::::>4<2 (α) (ψ) 2 . 5<3<::::>4>2 (ψ) 4. 5>3<::::>4<2

Π αρό.δηγμΗ 3 1 . Για κάθε χ Ε IR. ισχύει: χ>3�χ2>9(α)

ισχύει: χ2<9�χ<3 (α) ισχύει: χ2>9�χ>3 (ψ)

2 . 3 .

Για κάθε χ Ε IR. Για κάθε χ Ε IR.

Π αρ(ι.δειγμΗ 4 1 . Για κάθε χ Ε IR. 2 . Για κάθε χ Ε IR. 3 . Για κάθε χ Ε IR. Π u. μ ύ.δi : ιγμα 5

ισχύει: J x J<5<::::>-5<x<5 (α) ισχύει: J x J<-5<::::>5<x<-5 (α) ισχύει: J x J>-5<::::>x<5 ή χ>-5(α)

Να χαρακτηρίσετε ως αληθή ή ψευδή κάθε μια από τις παρακάτω προτάσεις : 1 . Υπάρχει χ Ε 1R. , τέτοιο ώστε να ισχύει:

( 1 - χ )4 � - (χ 2 + χ )2 2 . Υπάρχει y Ε IR , τέτοιο ώστε, για κάθε χ<Ο να

ισχύει: xJY = .J;!Y 3 . Για άπειρες τιμές του χ Ε IR. , ισχύει η συνεπα-

• 2 < 2 4 ? ο ' γωγη : χ + χ _ � χ + χ- < και για απειρες δεν ισχύει.

4. Για κάθε χ Ε IR. , ισχύει η συνεπαγωγή : χ 2 + [χ [ � Ο � χ4 + χ 2 > Ο

5 . Υπάρχει χ Ε IR. , τέτοιο ώστε να ισχύει η ισο­δυναμία: lx2 - χ Ι � 2 <=> -2 � χ � 1

6 . Υπάρχει χ Ε JR. , τέτοιο ώστε να ισχύει η συνε­παγωγή : lx 2 - x l :2:: 2 � -2 � χ � 1 Σε περίπτωση που η πρόταση χαρακτηριστεί αληθής, να βρείτε τις τιμές των x,y που καθι­στούν την αντίστοιχη ισότητα, ανισότητα, συ­νεπαγωγή ή ισοδυναμία αληθή . l:z6λια : (Για την περίπτωση που θα έχουμε ανταπόκριση από τους μαθητές) . Σαν μια επιπλέον προσπάθεια εμβάθυνσης στα όσα προαναφέραμε δίνουμε μέσα από έναν απλό σχολιασμό μερικές ακόμη πληροφορίες και παρατηρήσεις . Γνωρίζουμε ότι για κάθε α, β Ε 1R. ισχύουν οι ισοδυναμίες : α: + β� = Ο <=> α = Ο και β = Ο α� + β: :;t: Ο <=> α ;t. Ο ή β ;t. Ο

α · β ;t. Ο <=> α ;t. Ο και β ;t. Ο . •:• Η άρνηση δηλαδή της διάζευξης ( α=Ο ή β=Ο)

είναι η σύζευξη (α ;t. Ο και β ;t. Ο) Συμβολικά : ( p v q) <=> (Ρ Λ g) , (Π) Οι ισοδυναμίες (1), (ΙΙ) που ισχύουν για οποιο­

δήποτε ζεύγος προτάσεων p, q αποτελούν τους νό­μους του De' Morgan στη Λογική . •:• Παρατηρείστε επι πλέον ότι: Αφού δεχτήκαμε

ότι η συνεπαγωγή (ρ � q) είναι ψευδής μό­νον όταν (p αληθής και q ψευδής), συμβολικά (Ρ � q) <=> (Ρ Λ g) , θα έχουμε και:

(ρ � q) <=> (Ρ Λ q) <=> (ρ v q) <=> (Ρ v q) ' ( 1 ) Εντελώς όμοια : (q�p)��vp)�qvp) , (2)

Από τις ( 1 ) , (2) βλέπουμε ότι: ( p � q) <=> (g � p) , (3)

•:• Η σχέση (3) ονομάζεται νόμος της αντιθετοαντι­στροφής και θα μας φανεί αρκετές φορές χρήσι­μος, π. χ. για να δείξουμε ότι για κάθε τρίγωνο ΑΒΓ ισχύει: Β ;t. Γ � β ;t. γ μπορούμε να δείξουμε ότι: β = γ � Β = Γ, που είναι πολύ mo εύκολο. Τέλος παρατηρούμε ότι η σύζευξη ( q Λ g) εί­

ναι πάντοτε ψευδής, για αυτό λέγεται άτοπο (πα­ράλογο) .

Α ν λοιπόν είναι αληθής μια συνεπαγωγή της μορφής [Ρ � ( q Λ g) J , της οποίας το συμπέρα­σμα, όπως προαναφέραμε, είναι ψευδές, συμπε-

-ραίνουμε ότι θα είναι και η υπόθεση αυτής p ψευ-δής, άρα η πρόταση p αληθής. •:• Στην παρατήρηση αυτή βασίζεται η μέθοδος

της εις άτοπον απαγωγής, η οποία εφαρμόζε­ται όταν δεν μπορούμε να αποδείξουμε απευ­θείας ότι η πρόταση p είναι αληθής, οπότε αρ-

-κεί να αποδείξουμε ότι η p είναι ψευδής. Βιβλιογραφία

I . Bittinger Μ . (Logic and proof) 2 . Copi I . (symbol ic Logic) 3 . Exner-Rosskopf (Logic i n Elementary Mathemat-

ics) 4. Ki lmister C . (Language Logic and Mathematics) 5 . Novikov Ρ. (Instruction a la logique mathematique) 6. Α. Κ. Κυριακόπουλος (Μαθηματική Λογική)

ΕΥΚΛΕΙΔΗΣ Β' 75 τ.3173

Page 77: Ευκλειδης Β 75

Χάρης Καλλ ιγάς 1 946 - 2009

Χαράλαμπος (Χάρης) Καλλιγάς γεννήθηκε το 1 946 στην Αθήνα. Ο πατέρας του με κα­

ταγωγή από την Κεφαλλονιά και η μητέρα του με καταγωγή από την Τραπεζούντα, γεν­

νήθηκαν και οι δύο στην Ρωσία και κατέληξαν στην Ελλάδα προσφυγες μετά την Μι­

κρασιατική καταστροφή .

Τέλειωσε το Ε ' Γυμνάσιο Αθήνας και στη συνέχεια έλαβε πτυχίο από το Μαθημα­

τικό Τμήμα του Πανεπιστημίου Αθηνών. Παρακολούθησε διετές μεταπτυχιακό πρόγραμμα με

θέμα «πρότυπα δημιουργίας εκπαιδευτικού λογισμικού» στο Τμήμα Πληροφορικής του Πανεπι­

στημίου Αθηνών. Επιπλέον παρακολούθηκε μεγάλο αριθμό σεμινάριων σε θέματα Ψυχολογίας

της Μάθησης, Επικοινωνίας, Παιδαγωγικής, Μαθηματικών και Πληροφορικής. Υπήρξε υποψή­

φιος διδάκτωρ στο Τμήμα Ψυχολογίας του Παντείου Πανεπιστημίου με ερευνητικό θέμα: <<Δυ­ναμική κοινωνικών αναπαραστάσεων, προϋποθετικότητα και θεσμική πολιτική: Η διαχείριση αξι­ολογικών διεργασιών της εκπαιδευτικής διαδικασίας ως προς τη χρήση των νέων τεχνολογιών». Ουσιαστικά η διατριβή του αποτελούσε τη σύνοψη των επί πολλά έτη ερευνητικών ενδιαφερό­

ντων του που κυρίως ήταν η κατάλληλη διδακτικά και παιδαγωγικά χρήση των Νέων Τεχνολο­

γιών στο χώρο της εκπαίδευσης. Δυστυχώς ο πρόωρος θάνατός του δεν του επέτρεψε την ολο­

κλήρωση της διατριβής, η οποία είχε στόχο να αποτελέσει σημείο αναφοράς και ορόση μο στον

ευαίσθητο τομέα της σωστής χρήσης της Πληροφορικής στα εκπαιδευτικά δρόμενα.

Παντρεύτηκε το 1 97 1 και χώρισε 1 Ο χρόνια μετά. Είναι πατέρας δύο αγοριών του Διονύση

και του Μανου, των οποίων λίγο μετά το χωρισμό ανέλαβε την επιμέλεια, πράγμα αρκετά σπά­

νιο για τα ελληνικά δεδομένα.

Εργάστηκε, ως ιδιοκτήτης Φροντιστηρίων Μέσης Εκπαίδευσης και Εισαγωγικών Εξετάσε­

ων, ως καθηγητής στα Μεταλυκειακά Φροντιστήρια, σε Ιδιωτικό και Δημόσιο Γυμνάσιο και σε

Φροντιστήρια Εισαγωγικών Εξετάσεων για την Τριτοβάθμια Εκπαίδευση .

Διετέλεσε υπάλληλος στο ΥΠΕΧΩΔΕ ( 1 Ο έτη Προϊστάμενος Τμήματος Πληροφορικής) από

το 1 98 1 μέχρι την ημέρα του θανάτου του, εξαιτίας του καρκίνου, στις 3 1 / 1 0/ 2009 . Παρότι για

περίπου 30 χρόνια εργάστηκε στο ΥΠΕΧΩΔΕ, μεγάλες αγάπες του Χάρη και κεντρική δραστη­

ριότητα υπήρξε πάντα αφενός η Παιδεία και αφετέρου η κοινωνική και συνδικαλιστική δράση

και προσφορά με στόχο ένα κόσμο δημοκρατικό και δίκαιο . Ο λόγος που δεν διορίστηκε στην

Εκπαίδευση , αλλά παρέμεινε στο ΥΠΕΧΩΔΕ είναι ότι του ήταν αδύνατο να μετακινηθεί μόνος

και χωρίς βοήθεια με δύο μικρά παιδιά στην Επαρχία. Ο Χάρης ήταν από τους πρωτεργάτες, μαζί με πολλους άλλους συναδέλφους, των αλλαγών

στη διοίκηση, τη δομή , τη λειτουργία και κυρίως το πνεύμα της ΕΜΕ, μετά την επταετή δικτα­

τορία. Αποφασιστικός ήταν ο ρόλος του στην οργάνωση της διακίνησης και διάθεσης του έντυ­

που υλικού (Ευκλείδης) της ΕΜΕ. Με χρήση του δικού του αυτοκινήτου γύριζε τα σχολεία της

Αττικής φορτωμένος με περιοδικά της ΕΜΕ και διέδιδε τον Ευκλείδη στηριζόμενος κυρίως στην

προσωπική επαφή με τους συναδέλφους.

Συμμετείχε και άφησε τη σφραγίδα του σε πολλές από τις δραστηριότητες της ΕΜΕ. Ενδει-

ΕΥΚΛΕΙΔΗΣ Β ' 75 τ.3/74

Page 78: Ευκλειδης Β 75

κτικά αναφέρουμε τη συμμετοχή του σε Επιτροπές Παιδείας και Αναλυτικών Προγραμμάτων

και Συντακτικές Επιτροπές περιοδικών της ΕΜΕ.

Η κοινωνική και συνδικαλιστική του δράση στο ΥΠΕΧΩΔΕ αποτυπωθηκαν κυρίως στα ε­

ξής: Αιρετό μέλος του Υπηρεσιακού Συμβουλίου, ταμίας και υπεύθυνος σύνταξης της εφημερί­

δας της Πανελλήνιας Ομοσπονδίας Υπαλλήλων και Αντιπρόεδρος του Συλλόγου Υπαλλήλων.

Πολύ σημαντικό υπήρξε και το επιστημονικό του έργου, κεντρικός άξονας του οποίου ήταν

η σύνδεση της Πληροφορικής με τη Διδακτική και τα Παιδαγωγικά. Διετέλεσε εξωτερικός επι­

στημονικός και τεχνικός συνεργάτης της Ομάδας Διοίκησης του Π. Ι, Υπεύθυνος της Ομάδας

Εργασίας του κόμβου του Υ πΕΠΘ για το έργο και Συντάκτης του Τεχνικού Δελτίου Έργου & Υποέργων της πράξης της ΚτΠ: «Προετοιμασία του Δασκάλου της Κοινωνίας της Πληροφορίας/ Αρχική Επιμόρφωση όλων των Εκπαιδευτικών στις ΤΠΕ» ( 1 999-2004) . Συμμετείχε ως επιμορ­

φωτής ΤΠΕ για τα Μαθηματικά, στο πλαίσιο του προγράμματος «ΟΔ ΥΣΣΕΑΣ», που αφορά

στην αξιοποίηση των Δικτυακών και Υπολογιστικών Τεχνολογιών στην Εκπαίδευση . Επίσης

συ μετείχε ως επιμορφωτής στο 3° ΠΕΚ Αθήνας στο αντικείμενο : «Προϋποθέσεις χρήσης των ΤΠΕ στη διδακτιΚή πράξη» .

Συμμετείχε στη συγγραφή μεγάλου αριθμού άρθρων σε περιοδικά και εισηγήσεων σε Διε­

θνή και Ελληνικά Συνέδρια. Υπήρξε μέλος της συγγραφικής ομάδας που ανέλαβε, κατόπιν δια­

γωνισμού, τη συγγραφή των σχολικών βιβλίων Μαθηματικά Α ' Γυμνασίου - Βιβλίο μαθητή και

Μαθηματικά Α ' Γυμνασίου - Βιβλίο Καθηγητή. Χαρακτηριστικό της υπευθυνότητας, της δύναμης του χαρακτήρα, και της ανάγκης για άρ­

τια προσφορά του Χάρη είναι το εξής: Παρότι ήταν αρκετά κοντά στην τελική φάση της αρρώ­

στειας του, διέθετε τέτοιο κουράγιο και ψυχική δύναμη ώστε να αναλάβει μόνος του το δύσκο­

λο έργο της νέας μορφοποίησης του βιβλίου των Μαθηματικών της Α' Γυμνασίου, προκειμένου

να ενσωματωθούν σε νεώτερη έκδοση όσες από τις αλλαγές και βελτιώσεις που προτάθηκαν από

τους συναδέλφους καθηγητές Μαθηματικών κρίθηκαν θετικές. Μάλιστα αξίζει να τονισθεί ότι ο

ίδιος είχε την πρωτοβουλία για τη σύσταση ομάδας εμπείρων μάχιμων μαθηματικών και Σχολι­

κών Συμβούλων η οποία, σε συνεργασία με τα στελέχη των Μαθηματικών του Π. Ι . , ανέλαβε

την αξιολόγηση των προτάσεων για αλλαγές και βελτιώσεις που κατατέθηκαν από τους συνα­

δέλφους από όλη των Ελλάδα. Η αγωνιστικότητα και η συνέπεια που έδειξε στο παραπάνω θέμα

ήταν ένα από τα χαρακτηριστικά του Χάρη που ποτέ δεν τα εγκατέλειψε, ακόμη και στη δύσκο­

λη τελευταία διετία της ζωής του που σημαδεύτηκε από την αρρώστεια.

Ευχόμαστε, για λογαριασμό του Χάρη αλλά και όσων έχουν ανάλογες πεποιθήσεις, να

πραγματοποιηθεί κάποτε το βασικό του όραμα που ήταν ένας κόσμος δημοκρατικός, με ανθρω­

πιά, δίκαιος, ενωτικός, με έμφαση στις πνευματικές αξίες και με ευαισθησία στις ανάγκες των

άλλων. Η δυσκολία πραγμάτωσης ενός τέτοιου οράματος καθώς και η «απολυτότητα» στα πι­

στεύω του στάθηκαν και η αιτία που ο Χάρης συνάντησε στη ζωή του αρκετές ματαιώσεις και

aπογοητεύσεις.

Κλείνοντας, παραθέτουμε αυτούσια τα λόγια ενός μαθητή του φροντιστηρίου που άνοιξε ο

Χάρης στην περιοχή του Παπάγου,στα χρονια της δικτατορίας.

Είμαι σίγουρος, ότι για όλους εμάς που ζήσαμε αυτό το πρωτόγνωρο φροντιστήριο, η εποχή εκείνη μας σημάδεψε για όλη μας τη ζωή. Πρέπει να ήταν μια από τις ελάχιστες εφαρμογές της εκ­παίδευσης τύπου SummerHίll.

Να τον θυμόμαστε πάντα με αγάπη .

ΕVΚΛΕΙΔΗΣ Β ' 75 τ.3/75

Page 79: Ευκλειδης Β 75

Ε u κλε i δ n ς π ρ οτ ε i vε ιι

Επιμέλεια: Γ . Κ . ΤΡΙΑΝΤΟΣ - Ν . Θ . ΑΝΤΩΝΟΠΟΥ ΛΟΣ

ΑΣΚΗΣΗ 1 54 ( ΤΕΥΧΟΥΣ 7 1 ) Α ν α, β, γ ::?: Ο , α + β + γ = 1 , να δειχθεί ότι I I ,

2 2 - - 2αβγ ::; α- + β + γ :5: 1 - 2αβγ . 27

(Προτείνεται από τον συνάδελφο Γ Ε Ω ΡΓ Ι Ο N I KHTAKH - Σητεία Κρήτης )

1 2 I 3

= -(γ - 1) [4 (γ - Ι) + γ] = - 4 (γ - 1) - γ(γ - Ι ) = g(γ)

, 3 , I , με g (γ) = - - (γ - I)- - 2γ + I = - - (3γ- + 2γ - I)

4 4

και g '(γ) = 0 <=> γ = - 1 ν γ = .!. , g'(γ) > Ο <=> 3

Λ ΥΣΗ (από τον συνάδελφο Γ Ι Ω Ρ ΓΟ ΔΕΛΗΣΤΑΘΗ - Κάτω Πατήσια)

Αρκεί να δειχθεί ότι .!..!. ::; α 2 + β2 + γ 2 + 2αβγ ::; Ι . 27

Ε�αι : α2 + β2 + γ2 + 2αβγ = (α + β + γf

-2(αβ + βγ + γα) + 2αβγ = I - 2(αβ + βγ + γα) + 2αβγ = I - 2(αβ + βγ + γα - αβγ) = I - 2[α(β + γ) + βγ(Ι - α)] = 1 - 2[α(β + γ) + βγ(β + γ)] = I - 2(β + γ)( α + βγ) = 1 - 2(1 - α)( α + βγ) ::; 1 αφού είναι (1 - α)( α + β γ) ::?: Ο με την ισότητα να ισχύει όταν δύο από τους τρεις αριθμούς είναι ίσοι με Ο και ο τρίτος είναι ίσος με την μονάδα. Αποδεικνύουμε στην

, , Α 1 1 , συνεχεια οτι ::?: - , οπου 27

Α = α 2 + β2 + γ2 + 2αβγ . Σύμφωνα με τα προηγούμενα είναι Α = 1 - 2( α β + βγ + γα - αβγ) οπότε αρκεί να

δ θ ' ' I 2Β I I ' ' ειχ ει οτι : - ::?: - η το αυτο 27

Β ::; _!_ , όπου Β = αβ + βγ + γα - αβγ = 27

= (α + βγ)(β + γ) = (I - β - γ + βγ)(β + γ) = β + γ - β2

_γ2 _ 2βγ + β2γ + βγ2 = (γ _ Ι )β2 + (γ2 _ 2γ + ! )β -Αν

γ( γ - I ) = (γ - 1)β2 + (γ - 1)2 β - γ(γ - I ) � Β = (γ - Ι)β2 + (γ - 1)2 β - γ(γ - I) . γ = Ι ( οπότε α = β = Ο) , είναι Β = Ο . Αν γ :;t Ι , είναι γ - I < Ο και το τριώνυμο ως προς β δέχεται μέγιστη

τιμή για β = - (γ - !)2 = Ι - γ ίση με 2(γ - I) 2

Β = -4(γ - 1)2 γ - (γ - 1)4 -4(γ - Ι)γ - (γ - Ι )3 ιnax 4(γ _ J) 4

I Ο < γ < - , 3

Συνεπώς, η

συνάρτηση g έχει μέγιστη τιμή για γ = .!. . Τότε 3

β = 1 - γ = .!. , α = 1 - (β + γ) = .!. . Η μέγιστη τιμή του Β

2 3 3 , I I 3 1 1 8 8

ειναι: B ,nax = -4 (3 - 1) - 3 (3 - I ) = 27

. Δηλ. Β ::; 27

.

Λύσεις έστειλαν επίσης οι συνάδελφοι: Β αγγέλη ς Μ ουρούκος - Αγρίνιο, Γιώργος Μ ήτσιος - Ράμια Ά ρτας, Ορέστη ς Κατσάνος - Π ρέβεζα, Ροδόλφος Μ πόρη ς - Δάφνη . ΑΣΚΗΣΗ ! 55 ( ΤΕΥΧΟΥΣ 7 1 )

Σε κυρτό τετράπλευρο ΑΒΓ Δ είναι: Α = I 20°

Β = Δ = 90° ΓΔ2 - ΑΒ2 = 2.J3 και AB <

.J3. '

ΓΔ 2 Ι) Να υπολογισθεί το εμβαδόν του ΑΒΓ Δ.

1 1) Να δειχθεί ότι: ΑΓ > ΒΔ και ΑΓ = 2

0 ΒΔ 2 - εφ1 5 (Προτείνεται από τον συνάδελφο Γ Ι Ω ΡΓΟ Α ΠΟΣΤΟΛΟΠΟΥ ΛΟ - Μεσολόγγι ) Λ ΥΣΗ (από τον συνάδελφο ΒΑΓΓΕΛΗ ΜΟΥΡΟΥΚΟ - Αγρίνιο ) I) Έστω Ε το σημείο τομής των ευθειών ΑΔ και ΓΒ. Από την ομοιότητα των ορθογωνίων τριγώνων Γ ΔΕ και

ΑΒΕ , (ΓΔΕ) ΓΔ2 Ά εχουμε:

(ΑΒΕ) =

ΑΒ2 . ρα,

(ΑΒ ΓΔ) =

(ΓΔΕ) - (ΑΒΕ) = (ΓΔΕ) - \ = (ΑΒΕ) (ΑΒΕ) (ΑΒΕ)

ΓΔ2 ΓΔ2 - ΑΒ2 2.)3 -- - 1 = --....,----ΑΒ2 ΑΒ2 ΑΒ2

Γ

' 1 Ι h .)3 Ί Επισης, (ΑΒΕ) = - ΑΒ · ΒΕ = - ΑΒ(ν3ΑΒ) = - ΑΒ-2 2 2

Ά (ΑΒΓΔ) = 2.J3

(ΑΒΓΔ) = 3 ρα, r;; 2

� τ.μ . ν 3 , ΑΒ - ΑΒ-2

Il) Από το θεώρημα των συνημιτόνων στα τρίγωνα ΑΒΔ και ΒΓΔ έχουμε:

ΕΥΚΛΕΙΔΗΣ Β' 75 τ.3176

Page 80: Ευκλειδης Β 75

--------------- Ο Ευκλείδης προτείνει . . .

ΒΔ2 = ΑΒ2 + ΑΔ2 - 2ΑΒ · ΑΔ · συν\ 20° = ΑΒ2 + ΑΔ2 + ΑΒ · ΑΔ ( \ )

ΒΔ2 = ΒΓ2 + ΓΔ2 - 2ΒΓ - ΓΔ · συν60° = ΒΓ2 + ΓΔ2 - ΒΓ · ΓΔ (2) Με πρόσθεση κατά μέλη των σχέσεων ( I ) ,(2) : 2ΒΔ2 = (ΑΒ2 + ΒΓ2 ) + (ΑΔ2 + ΓΔ2 ) + ΑΒ · ΑΔ - ΒΓ · ΓΔ = 2ΑΓ2 + ΑΓημΓ2 · ΑΓημΓ1 -ΑΓσυνΓ 2 · ΑΓσυνΓ1 = 2ΑΓ2 - ΑΓ2συν(Γ1 + Γ 2 )

= ΑΓ2 (2 - συν60° ) = ΑΓ\2 - .!_) = � ΑΓ2 2 2

ΑΓ2 2 => - = r:;3

> I => ΑΓ > ΒΔ ΒΔ2

\f j

2 ΑΓ 2

J3cJ3 - ι) J3(J3 + 1)

= - => - = ------::-J3 ΒΔ 2 - εφl 5 °

2

2 2 - (2 - J3)

Λύσεις έστειλαν επίσης οι συνάδελφοι: Ροδόλφος Μπόρης -Δάω,η. Γιά\'\η.:: 2:ταματογιάννης - Δροσιά, Φίλιππος Σερέφογί.ου - \Ιεί.ίmα. Κω,· νο; Τζαγκαράκης - Χανιά, Γιιίψγος Μιiτσιος - Ράμια Αρτα.::. ΑΣΚΗΣΗ 1 56 ( ΤΕΥΧΟΥΣ 72)

Αν χ, y, z είναι θετικοί αριθμοί με χ 2 + y2 + z2 = Ι , τότε να βρεθεί η ελάχιστη τιμή της συνάρτησης :

χ 2 χ2 y2 / 22 22 f(x , y, z) = - + - + - + - + - + - .

Υ z χ z Υ χ (Προτείνεται από τον συνάδελφο ΓΙΩΡΓΟ ΤΣΑΠΑΚΙΔΗ - Κυψέλη Αιτ/νίας ) . Λ ΥΣΗ ι Η (από τον συνάδελφο ΑΘΑΝΑΣIΟ ΚΑΛΑΚΟ - Κ.Πατήσια)

χ 2 χ 2 y2 y2 22 22 f(x , y, z) = - + - + - + - + - + - =

Υ z χ z Υ χ / + 22 22 + χ 2 χ 2 + y2 1 _ χ 2 1 _ /

-- + -- + --- = -- + --χ Υ z χ Υ

Ι - z2 ι ι ι +-- = -(χ + y + z) + (- + - + -) (I ) z χ y z

Από την ανισότητα του SCHWARTZ έχουμε: ( 1 : + 1 2 + 12 )(χ 2 + / + z2 ) ;::: (1 - x + l · y + l · z)2

::::> 3 ;::: ( x + y + z)2 ::::> x + y + z � J3 (2) Από την ω1σότητα αριθμητικού - γεωμετρικού μέσου για

θ'

θ ' 2 ' ' 1 1 1 τους ετικους αρι μους χ , y- , z- και τους - - -

χ Υ z

ΨΥz � _ι => _ι - ;::: J3 => 3ΓΙ ;::: J3 (3) J3 ΨΥz νφ

� + t + � � ( 3 1 I I Ι r:; ---=---- ;::: 3 - => - + - + - ;::: 3 ν 3 (4)

3 xyz χ y z Η ( I ), λόγω των σχέσεων (2) και ( 4) δίνει : f(x, y, z) ;::: -J3 + 3J3 = 2J3 με την ισότητα να ισχύει

· · 1 τ λ

· · μονον οταν χ = y = z =

J3 . ε ικα, ειναι

· r r;; f ι ι ι mιn (x , y, z) = 2 ν 3 = (J3

, J3

, J3

) .

Λ ΥΣΗ 2 Η : ( από τον συνάδελφο ΔΗΜΗΤΡΙΟ ΚΟΥΚΑΚΗ - Κάτω Απόστολοι Κιλκίς ) Παρατηρούμε ότι η f γράφεται:

χ 2 χ 2 y2 y2 22 22 f(x, y, z) = - + - + - + - + - + - =

Υ z χ z Υ χ y2 + 22 22 + χ 2 χ 2 + / I _ χ 2 I _ y2 I _ 22

-- + -- + -- = -- + -- + -- (\) χ Υ z χ Υ z

Θεωρούμε την συνάρτηση g : (0, I ) --t R με τύπο

g( t) = lftt . Παρατηρούμε ότι η ( I ) γράφεται:

I - χ 2 I - y2 I - z2 f(x, y, z) = -- + -- + -- =

χ Υ z = g(x2 ) + g(y2 ) + g(z2 ) (2)

για κάθε x, y, z > O : x 2 + y2 + z2 = 1 . Η g είναι δύο φορές παραγωγίσιμη στο (0, 1 ) με

g"(t) = .!. ( h + ;, ) > Ο , για κάθε t ε (O, J) . 4 ν' t5 ν' t3

Από την ανισότητα του JENSEN έχουμε ότι g(x 2 ) + g(/ ) + g(z2 ) χ 2 + 2 + z2 ������ ;::: g( Υ )

3 3 ι

� f(x, y, z) ;::: g(.!.) = Ι - 3 = 2J3 =>

3 3 ι 3 J3

r:; ι ι ι f(x , y, z) ;::: 2ν j = f(J3

, J3

, J3

)

Λύσεις έστειλαν επίσης οι συνάδελφοι: Γίαννης Ηλιόπουλος - Καλαμάτα, Ροδόλφος Μπόρης - Δάφνη, Βαγγέλης Μουρούκος - Αγρίνιο, Διονύm1ς Γιάνναρος - Πύργος, Δημήτρης Μαντζαβίνος - Λαμία, Αντώνι1ς Ιωαννίδης -Λάρισα, ο Πολιτικός Μηχανικός Ανδριiς Ιωάννης - ΑθιΊνα. ΑΣΚΗΣΗ 1 57 ( ΤΕΥΧΟΥΣ 72 ) Αν � = 2χ2 - 1 + 2χ� ( ! ), τότε να δειχθεί ότι

• 2 5 - J5 (Π ' ' ' δ λ ισχυει: χ = -- . ροτεινεται απο τον συνα ε φο 8

Γ. Ν Ι ΚΗΤ ΑΚΗ - Σητεία Κρήτης ) ΛΥΣΗ (από τον συνάδελφο Δ. ΓΙΆΝΝΑΡΟ - Πύργος) Επειδή η ( \ ) ορίζεται για χ Ε R : l x l � ι ' θέτουμε

ΕΥΚΛΕΙΔΗΣ Β' 75 τ.3/77

Page 81: Ευκλειδης Β 75

---------------- Ο Ευκλείδης προτείνει . . . ----------------

χ = συνω , ω Ε [0, π] . Η ( I ) γράφεται

.Jι - συνω = 2συν2ω - 1 + 2συνωJη μ2ω

<=> .J1 - συν ω = συν2ω + 2συνωη μω

<=> .J1 - συν ω = συν2ω + η μ2ω (2) Από την (2), με ύψωση στο τετράγωνο, έχουμε 1 - συνω = 1 + η μ4ω <=> η μ4ω = -συνω

3π 3π <=> η μ4ω = η μ(- + ω) <=> 4ω = 2kπ ± (- + ω) , k Ε Ζ Ο 2 2 3π 2kπ π

3ω = 2kπ + - ω = -- + -

2 3 2 <=> ν , k Ε Ζ ::::> ν , k E Z

3π 2kπ 3π 5ω = 2kπ - - ω = -- - -

2 5 ι λύσεις της (2) στο διάστημα [Ο,π] είναι :

3π 7π Ε δ

' , ω = - ν ω = - . πει η ει ναι 1 0 1 0 7π 3π 3π

συν - = συν( π - -) = -συν -1 0 1 0 1 0 '

1 0

' θ ' ' 2 ' ' 3π σε κα ε περιπτωση εχουμε χ = συν-ω = συν- - .

Ά ' 23π

ρα χ- = συν -

3π 1 + συν -5

2π 1 - συν -

1 0 2 2 Γ

λ ' 2π ' ια τον υπο ογισμο του συν - , εχουμε

5 3π 2π 3 π π συν - = η μ - ::::> 4συν - - 3συν - = 1 0 1 0 1 0 1 0 π π ' π π 2η μ - συν - => 4συv- - - 3 = 2η μ -1 0 1 0 1 0 1 0

2 π π ::::> 4(1 - η μ -) - 3 = 2η μ -

1 0 1 0

5

2π π π .JS - 1 => 4η μ - + 2η μ - - 1 = 0 => η μ - = --

1 0 1 0 1 0 4

Σ ' 2π π .J5 - 1

λ '

υνεπως, συν - = η μ - = -- και τε ικα 5 1 0 4

.J5 - ι 1 - -

4- 5 - .JS

(2) => χ2 = --'------2 8

Λύσεις έστειλαν επίσης οι συνάδελφοι :

1 0

(3)

Ο ρέστη ς Κ ατσάνος - Π ρέβεζα, ΑντιίΝης ι ωαννίδης -Λ ά ρ ισα. ΓιιίJ ργος Δεληστάθης - Κ . Γl αηΊσια, Γιιίφγος Μ ΙΊτσιος - Ράμια Ά ρτας, Δη μι1φης Μ αντζαβ ίνος -Λαμία , Β αγγ{:λη ς Μ ουροί>κος - Αγρίν ιο . Α θ ανάσιος

3x3 - 7x2 - 6xy - 2 ιx - 1 8y + 88 = 0 ( ι ) . (Προτείνεται από τον Χημικό Δ Η Μ Η Τ Ρ Ι Ο ΚΑ Ρ Β ΕΛΑ - Πεύκη ) Λ ΥΣΗ ( από τον συνάδελφο Γ Ι Ω Ρ ΓΟ ΤΣΑ Π Α Κ Ι Δ Η - Κυψέλη Αιτ/νίας ) Η εξίσωση ( 1 ) γράφεται ισοδυνάμως:

(7χ + 6y)(x + 3) = 3χ3 + 88 (2) . Αν ήταν χ = -3 η (2) θα έδινε 0=7, άτοπο, άρα είναι χ * -3 , οπότε

3χ3 + 88 7 (2) <=> Ίχ + 6y = = 3χ2 - 9χ + 27 + -- Επειδ

χ + 3 χ + 3 ή οι αριθμοί 7x + 6y , 3x2 - 9χ + 27 είναι ακέραιοι για

όλες τις τιμές των ακεραίων χ, y , πρέπει ο ακέραιος

χ + 3 να διαιρεί τον αριθμό 7 . Δηλ. χ + 3 = - Ι ν χ + 3 = ι ν χ + 3 = -7 ν χ + 3 = 7

οπότε θα είναι: χ = -4 ν χ = -2 ν χ = - ι ο ν χ = 4 Από την (2) για κάθε μία από τις παραπάνω τιμές του χ, παίρνουμε τις αντίστοιχες τιμές του y που είναι y = 22 ν y = 1 3 ν y = 8 ι ν y = 2 . Λύσεις έστειλαν επίσης οι συνάδελφοι: Β αγγέλης Μ ο ιφοl> κος Αγρίνιο, Γ ιάννης Η λΗ)πουλος ­Καλαμάτα, Κ ων/νοc Τζαγκαρ<Ίκης - Χ ανι<Ί, Διονl>σης ι Ί6νναρος - Πύργο�. Γl έφος ΣκαλτσιΊς Αγρίνιο, Δη μΙ1τρης Μ αντζαβ ί\ '0� Λ α μ ί α . Α\ 'Ι < : ) \ ·ηc Ι ω ωνίδης ­

Λάρισα, Γι rί)ργος Μ ι1τσιος - Ρ rΊ. μ ι ιι. ' \ ιηα.: . . "ωιΙ Ι<ιχ.: Χυη1 ρ η ς - Κέρκυρα, Δ η μΙ1 τρ ιος Κοι ι κύ. κη � - Κ ύ.τω Απόστολο ι Κ ιλκίς, Ν ικόλαος Π αuί.οι ι Χ αλκίδα, Ο Η λεκτρολόγος Μ η χαν ικό� E . :vi . Π Βασιλέας Καρδαμ ίτσης - Κορωπί και ο Π ολιτικός Μ η χανικός Ε . Μ. Π ΑνδρΙ1ς ι ω ιΊννης - Α θΙ1να. Α Σ Κ Η Σ Η 1 59 ( ΤΕΥΧΟΥΣ 72 ) Έστω Ρ(χ) = χ6 + χ5 + χ4 + χ3 + χ2 + χ + 1

Να βρεθεί το υπόλοιπο της διαίρεσης: Ρ(χ 7 ) : Ρ(χ) ( Προτείνεται από τον συνάδελφο Γ Ι Λ Ν Ν Η Η Λ Ι Ο Π ΟΥ ΛΟ - Καλαμάτα) Λ ΥΣΗ ( από τον συνάδελφο Β Α Γ Γ Ε Λ Η \1 0 Υ ΡΟ Υ ΚΟ - Αγρίνιο)

Παρατηρούμε ότι: (x - l)P(x) = x 7 - 1 ( \ ) Το υπόλοιπο της διαίρεσης Ρ( χ) : (χ - I) είναι ίσο με

P(l ) = 7 . Επομένως, υπάρχει μοναδικό πολυώνυμο

Q( χ) τέτοιο, ώστε να ισχύει

P(x) = (x - l)Q(x) + 7 (2). Στην ταυτότητα (2) θέτουμε

όπου χ το χ 7 και έχουμε την ταυτότητα

P(x 7 ) = (x 7 - \)Q(x 7 ) + 7 (3). Βάσει της ( I ) η (3)

γράφεται: Ρ( χ 7 ) = (χ - \)P(x)Q(x7 ) + 7 ή το αυτό

P(x7 ) = P(x)[(x - l)Q(x 7 )] + 7 = P(x)R(x) + 7 (4) Όπου R(x) = (χ - I )Q(x) . Η (4) αποτελεί την ταυτότητα

Κ αλάκος - Κ . Π ατΙΊ σ ι α. Γ ιάννης Η λιόπουλος - της Ευκλείδιας Διαίρεσης Ρ( χ 7 ) : Ρ( χ) Κ αλαμ<Ίτα, Π έτρος Σκαλτσάς - Α γ ρ ίνιο, ΡοΜ)λφος Μ πόρης - Δ<Ίφνη , Δη μΙ1τριος Κουκ<Ίκης - Κ άτω Απόστολο ι Κ ιλκίς, Ο Η λεκτρολόγος Μ ηχανικός Ε . Μ . Π Βασιλέας Καρδαμ ίτσης - Κορωπί και ο Π ολιτικός Μ η χανικός ΑνδρΙΊς ι ω <Ίννη ς - ΑθΙΊνα. Α Σ Κ Η Σ Η 1 58 ( ΤΕΥΧΟΥΣ 72 ) Να βρεθούν οι ακέραιες λύσεις της εξίσωσης:

Με αποτέλεσμα το ζητούμενο υπόλοιπο να είναι υ = 7 .

Λύσεις έστειλαν επίσης οι συνάδελφοι: Δη μι]τριος Μ αντζαβ ίνος - Λ α μ ία, Π έτρος Σ καλτσάς - Αγρίνιο, Α θ αν6σιος Κ αλάκος Κ. Π αηΊ σια, ΓιιίJργος Τσαπακίδης - Κυψέλη Α ιτ/νίας, Αντιί)\ιης ι ω αννί δ η ς -Λ άρισα, Γ ιιi)ργος Μ 1]τσιος - Ράμια Άρτας, Κων/νος

ΕΥΚΛΕΙΔΗΣ Β' 75 τ.3/78

Page 82: Ευκλειδης Β 75

---------------- Ο Ευκλείδης προτείνει . . .

ΤζαγκαρrΊ.κη-; Χ αvιrΊ. Διονί>σιος Γ ιάvvαρος - Π ίφγος .

Δη μιiτριος Κ ουκrΊκης Κάτω Απόστολο ι Κ ιλκίς, Ν ικόλαος Π αύλου - Χαλκίδα, Ροδόλφος Μπόρης -

ΔrΊφνη , και ο Η λεκτρολόγος Μ η χανικ6ς Ε . Μ . Π Β ασιλέας Κ α ρδαμίτση ς - Κορωπί .

Α Σ Κ Η Σ Η 1 60 ( ΤΕΥΧΟΥΣ 72 ) Αν α,β,γ είναι θετικοί αριθμοί να δειχθεί

β ι 4 , 3 3 α + - + 1_ + - (α' + β3 + γ3 ) :2: - (α4 + β4 + γ4 ) . β γ α 3 3

(Προτείνεται από τον συνάδελφο Γ Ι Ω Ρ ΓΟ Α Π ΟΣΤΟΛΟ Π ΟΥ Λ Ο - Μεσολόγγι) Λ ΥΣΗ ( από τον συνάδελφο Γ Ι Α !\ Ν Η Η Λ Ι Ο Π ΟΥ ΛΟ

- Καλαμάτα ) Από την ανισότητα μέσου αριθμητικού - μέσου γεωμετρικού για τους θετικούς αριθμούς α β γ ι . . . β , Υ ,

α, 3 (α> + β> + γ> ) παίρνουμε :

α β γ ι . . . - + - + - + - (α> + β> + / ) β γ α 3 α β ν ι . . .

_,______,_ _______ :2: 4 _ . - . _!_ . - (α·' + β' + γ ) = 4 β γ α 3

3 3 ι ' ' ' α3 + β3 + γ' .!_ α� + β� + γ4 = 4 - (α> + β> + / ) = ( , ) � :2: -----'---'---3 3 3

β ι 4 , , 3 => � + - +l + - (α� + β' + / ) :2: - (α:; + β4 + γ4 ) β γ α 3 3

Όπου έγινε zρήση της ανισότητας του μέσου αριθμητικού των δυνάμεων. Αποδεικνύουμε την

_!_ Για την συνάρτηση f(x) = χ 4 , χ > Ο έχουμε:

3 7 f , ι - :; f"( )

3 -4 0 . . (χ ) = - χ · , χ = - - χ < , που σημαινει οτι η 4 1 6

συνάρτηση f είναι μη κυρτή και συνεπώς, από την ανισότητα του JENSEN παίρνουμε

f( x + y + z) :2: f(x) + f(y) + f(z) =>

3 3 .!_ _!_ ι

χ + Υ + z χ 4 + y4 + z 4 (

3 ) :2:

3 και για χ = α 3 , y = β3 , z = γ3 προκύπτει η αποδεικτέα. Λύσεις έστειλαν επίσης οι συνάδελφοι: Δη μ ιiτρ ιος Μ αντζα β ίνο-; - Λ α μ ία. Ροδόλφος Μ π6ρης - Δ6φvη , Α ΟανrΊσιος Κ αλrΊκος Κ . Π ατιi σια, Β αγγέλη ς Μ ου ρο\ ηως Αγρίνιο . Δ ιονύσ ιος Γ ιcΊνναρος - Π ι'ι ργος .

Αντι\>νη ς Ι ωαννίδη ς - Λ r'φ ισα, Δ η μιi τρ ιος Κ ουκr'ικης -

Κ cΊτω Λπί>στολοι Κ ιλκίς. ΖελεβcΊρη Άννα - Θες/νίκη. και ο Η λεκτρολ(>γος Μ η zα\' ικ(κ Γ . Μ . Π Β α σ ιλέας Καρδαμίτση ς - Κ ορωπί .

Π Ρ ΟΤ Ε Ι :\ 0 \ιJ Ε :\ Α Θ Ε ." ΛΤΛ

1 77. Ν α βρεθούν όλες οι ακέραιες λύσεις της εξίσωσης: 73 ι χ + 779y = 720408 (1 ) .

Προτείνεται από τον Χημικό κ. Δ Η Μ Ή Τ Ρ Ι Ο Κ Λ Ρ Β ΕΛ Α - Πεύκη . 1 78. Α ν α, β, γ είναι θετικοί πραγματικοί αριθμοί, τότε να δειχθεί ότι ισχύει η ανισότητα: αz βz / α + β + γ -, + ----,- + -, :2: 2 ο , r::;;:: β- γ- α- Ζ,αβγ

Προτείνεται από τον συνάδελφο Γ Ι Ω Ρ ΓΟ Α Π ΟΣΗΜ.Ο ΓΙ Ο Υ Λ Ο - Μεσολόγγι. 1 79. Δίνεται ορθογώνιο παραλληλόγραμμο ΑΒΓΔ με

ΑΒ=α , ΒΓ=β και α= φ (όπου φ = J5 + 1 ο λόγος της

β 2 χρυσής τομής ) Έστω Ε , Ζ τα μέσα των ΑΔ και ΒΓ αντιστοίχως. Στην προέκταση της ΓΔ θεωρούμε σημείο

Η τέτοιο, ώστε _1_ = φ και ονομάζουμε Θ το σημείο ΗΕ

τομής της ΗΕ με την διαγώνιο ΑΓ. Να υπολογισθεί ο

λ . β δ ' λ (ΑΒΓΔ) ογος των εμ α ων : = . (ΘΕΖ)

Προτείνεται από τον συνάδελφο Γ ΙΩΡΓΟ Μ Η ΤΣ Ι Ο - Ράμια Άρτας . 1 8 0 . Δίνεται οξυγώνιο τρίγωνο ΑΒΓ με AB= l και μέτρα γωνιών Α <Β<Γ που αποτελούν διαδοχικούς όρους αριθμητικής προόδου . Φέρουμε το ύψος ΒΔ. Αν ΑΔ=ρ, όπου ρ (θετική) ρίζα της εξίσωσης 8χ 3 - 6χ + Ι = Ο ( Ι ) , τότε να υπολογισθεί το εμβαδόν του τριγώνου ΑΒΓ συναρτήσει του ρ . Αν - επιπλέον- Κ ,Λ, Μ είναι σημεία των πλευρών ΒΓ , Γ Α , ΑΒ αντιστοίχως τέτοια, ώστε οι ευθείες ΑΚ , ΒΛ , ΓΜ να συντρέχουν σε σημείο Ο και να ικανοποιούν τη

. ΟΑ ΟΒ ΟΓ 6 Τ . δ θ . . . σχεση - + - + -- = , οτε να ει χ ει οτι ισχυει: ΟΚ ΟΑ ΟΜ

7 .fi - < (ΚΛΜ) < - . ι οο 20

Προτείνεται από τον συνάδελφο Γ Ι Ω ΡΓΟ Μ Η ΤΣ Ι Ο - Ράμια Άρτας .

ΕΝ Η Μ Ε ΡΩΣ Ε Ι Σ - Π Α ΡΑ τ ι � Ρ Η Σ c Ι Σ - Σ Χ Ο Λ Ι Α 1 . Παρακαλούμε όλους τους συναδέλφους που μας στέλνουν τις εργασίες τους να γράφουν πάνω σε αυτές την ημερομηνία αποστολής τους στην ΕΜΕ καθώς και τα τηλέφωνά τους ( κινητό - σταθερό) 2 . Με μεγάλη Ταχυδρομική καθυστέρηση λάβαμε από τον συνάδελφο ΓΗ;ψγο Λ ποστολι'>τιουλο - \11 εσολόγγ ι , λύση της άσκησης 1 45 του τεύχους 69 (που προτάθηκε από τον συνάδελφο I Ί r ;φγο Τρ ιcΊηο ) η λύση της οποίας δημοσιεύθηκε ήδη στο τεύχος 73 . 3 . Η στήλη καλωσορίζει την συμμετοχή των τριών φίλων και επί πτυχίω φοιτητών του Α.Π.Θ. Δη μιi η> ιο Κουκr'1κη (Κιλκίς) - Ν ικ6λαο Π αύλου ( Χαλκίδα) και Ζελεβ6ρη Άννα (Θεσσαλονίκη) Τους ευχαριστεί θερμά, για τα καλά τους λόγια και τη θετική ανταπόκριση στις ασκήσεις του περιοδικού, τους θεωρεί πλέον πολύτιμους συνεργάτες της στήλης και περιμένει νέες εργασίες τους .

ΕΥΚΛΕΙΔΗΣ Β ' 75 τ.3/79

Page 83: Ευκλειδης Β 75

Τα Μαθηματικά μας διασκεδάζουν

Γνωρίζετε ότι : •:• Ο Ήρωνας το 2° π.Χ. αιώνα κατάφερε να δημιουρ­

γήσει πολυμέσα (multimedia) ; Κατάφερε με κίνηση νερού και ατμό να κάνει αυτόματο θέατρο. Σε ομοίωμα του ναού του Διονύσου, κινεί φιγούρες, ανάβει φωτιές, στεφανώνει το επιστύλιο με λουλούδια, ενώ ταυτόχρο­να ακούγονται ήχοι από τύμπανα και κύμβαλα.

•:• Ο πρώτος Η/Υ έγινε το 1939 με λυχνίες τις οποίες κα­τασκεύασε ο Lee de Forest το 1906; Το 1 948 έγινε η ε­φεύρεση του τρανζίστορ, έτσι από το 1 956 τα τρανζίστορς

Παναγιώτης Π. Χριστόπουλος

αντικαθιστούν τις λυχνίες στους ΗΝ ενώ εμφανίστηκε και η πρώτη γλώσσα υψηλού επιπέδου FOR­TRAN. Το 1 964 έχουμε το ολοκληρωμένο κύκλωμα ( chip) και την τρίτη γενιά ΗΝ. Από το 1 97 1 με τον μικροεπεξεργαστή (microprocessor) πήγαμε στην τέταρτη γενιά. Σήμερα είμαστε σε μια γενιά «έξυπνων» υπολογιστών, με ιστούς, με νευρωνικά δίκτυα που τα χειριζόμαστε όχι μόνο με τα δάκτυλα αλλά και με τη φωνή, ή τα μάτια, ή την κίνηση στους μυς. Το 1 997 ο υπερυπολιστής Big Blue της ΙΒΜ νίκησε τον Κασπάροβ στο σκάκι.

•:• Η Εμι Νέδερ ονομάστηκε μητέρα της σύγχρονης άλγεβρας; Το 1 9° αιώνα και αρχές του 20ου δια­πρέπουν στα Ευρωπαϊκά Πανεπιστήμια μόνο άνδρες. Έτσι γινόταν ανέκαθεν. Τα Πανεπιστήμια ήταν μόνο για αγόρια. Στη Γερμανία είναι ο Γκάους, ο Ντέντεκιντ, ο Ντιρικλέ, ο Φίσερ, ο Χίλμπερτ, ο Βά­ιερστρας, ο Κλάιν, ο Αϊνστάιν και ανάμεσά τους, μόνο τρείς γυναίκες που κατά εξαίρεση γίνονται δεκτές στο Πανεπιστήμιο του Γκέτινγκεν αλλά χωρίς μισθό. Μια από αυτές η Εμι Νέδερ (1882-1935). Όταν ζήτησε τη θέση στο Πανεπιστήμιο υπήρχαν αντιρρήσεις, ήταν γυναίκα με εβραϊκή κα­ταγωγή και ειρηνόφιλη . Τότε ο Χίλμπερτ είπε «Κύριοι μιλάμε για μια θέση σε πανεπιστήμιο, όχι σε δημόσια λουτρά. Το φύλο δεν πρέπει να αποτελεί κριτήριο». Ο Αϊνστάιν πρόσθεσε «Χθες συζητούσα με τη δεσποινίδα δόκτορα Νέδερ. Η μαθηματική απόδειξη της θεωρίας μου, ακόμα και για εμένα, εί­ναι μεγάλη αγγαρεία. Οι επαναστατικές τεχνικές της Νέδερ στην αφηρημένη άλγεβρα οδηγούν στην τέλεια μαθηματική διατύπωση της σχετικότητας» . Η Νέδερ προσέδωσε στην άλγεβρα την αξιωματι­κή δομή που επιζητούσε ο Χίλμπερτ στο 2° Παγκόσμιο Μαθηματικό Συνέδριο στο Παρίσι 8-8- 1 900 με τα 23 προβλήματα που έθεσε και από τα οποία 2 ακόμη μένουν άλυτα. Η Έμι κατόρθωσε να δημι­ουργήσει τη μαθηματική διατύπωση της σχετικότητας. Ονομάστηκε μητέρα της σύγχρονης άλγε­βρας. Γνωρίστε την. Διαβάστε το βιβλίο που κυκλοφόρησε με τίτλο «Εμι Νέδερ η κυρία της άλγε­βρας» . Φαί)λος κί)κλος: Κάποτε στην αρχαία Ελλάδα ο σοφός Επιμενίδης από την Κρήτη είπε: «οι Κρήτες είναι πάντα ψεύτες». Το 1 9° αιώνα διατυπώθηκε ως λογικό παράδοξο του Επιμενίδη . Δηλαδή αφού ο Εmμενί­δης λέει ότι οι Κρήτες είναι πάντα ψεύτες, ο Εmμενίδης είναι Κρητικός, άρα ο Επιμενίδης λέει ψέματα, άρα οι Κρήτες λένε την αλήθεια, άρα και ο Επιμενίδης λέει την αλήθεια, άρα οι Κρήτες είναι ψεύτες κ.ο.κ. Ο κύκλος των συλλογισμών που οδηγεί από τις αρχικές σου σκέψεις σε δεύτερες και έπειτα σε επαναφέ­ρει στις πρώτες λέγεται φαύλος κύκλος. Οι πίθη κοι : Δύο πίθηκοι μπήκαν 3 φορές σε οπωρώνα και συγκέντρωσαν ισάριθμες ποσότητες από μπανάνες και καρύδες, ετοιμάζονται για το φαγοπότι αλλά βλέπουν τον ιδιοκτήτη να τους πλησιάζει

με το μπαστούνι. Υπολογίζουν ότι για να τους φτάσει θα περάσουν 2 .!._ λεπτά. Τρώνε με λαιμαργία, ο 4

ΕΥΚΛΕΙΔΗΣ Α . 75 τ.3/80

Page 84: Ευκλειδης Β 75

------------- Τα Μαθηματικά μας διασκεδάζουν ------------­

πρώτος που τρώει 1 Ο καρύδες στο λεπτό, τις τρώει όλες στα 2/3 αυτού του χρόνου και βοηθά το φίλο του να φάει τις μπανάνες, προλαβαίνουν στο παρά πέντε. Ο πρώτος καταναλώνει τις μπανάνες 2 φο­ρές πιο γρήγορα από ότι τις καρύδες. Με πόση ταχύτητα ο δεύτερος τρώει τις μπανάνες;

τ 1 ' ' 1 'Ε θ ' ' 1 1 ' - 1 Ι ' Γ-ϊ Γι δ ' ο ει ναι ισο με το - ; νας μα ητης εγραψε - =- αρα -

.- = =τ αρα ντ-1- = ν =τ και επει η

( � = � ) έχουμε J7f = � αλλά Γ-Ϊ = i δηλαδή i . i = 1 . 1 άρα ί 2 = 1 . 1 ή l - 1 =

1 ! ! ! . IΠού έκα-'J"b νb ν 1 ν- 1 νε λάθος ; Π ότε γεννή θη κε ; Ο καθηγητής των μαθηματικών την 1 η Απρίλη 20 1 Ο είχε τα γενέθλιά του αλλά πό­τε γεννήθηκε; Είπε όμως στους μαθητές του ότι θα είναι Χ χρόνων το έτος Χ2 • Τα ψάρια : Τρείς φίλοι συμφώνησαν να πάνε την Κυριακή για ψάρεμα και το βράδυ να φάνε μαζί με τις συζύγους τα ψάρια που θα φέρουν. Ο Θωμάς έφερε ένα ψάρι 3 κιλά, ο Σταύρος ένα ψάρι 5 κιλά και ο Χρίστος ένα ψάρι 40 κιλά το οποίο όμως πούλησε προς 30€ το κιλό . Το βράδυ αφού έφαγαν τα άλλα 2 ψάρια ο Χρίστος έδωσε στους φίλους του να μοιραστούν 80€. Πώς πρέπει να τα μοιράσουν; Είναι δίκαιο το ποσό που έδωσε στους φίλους του; Ο Χυμός: Ένας πελάτης ζήτησε χυμό πορτοκάλι. Ο σερβιτόρος έβαλε το χυμό από ένα πορτοκάλι σε ένα κωνικό ποτήρι και έφτασε στο μέσο του ύψους του ποτηριού το συμπλήρωσε με πάγο και το σέρβιρε. Ο επόμενος πελάτης ζήτησε χυμό αλλά χωρίς πάγο, με το χυμό πόσων πορτοκαλιών θα γε-

ίσει το ποτ ' ι; Ευχαριστώ τους συναδέλφους μαθηματικό Σκλάβου Ελένη, τη φιλόλογο Πρέκα Μαρία και τον καθη­γητή πληροφορικής Πέτρο Χριστόπουλο, για τη στήριξη που πρόσφεραν στη στήλη .

Απαντήσεις στα μαθηματικά μας διασκεδάζουν. Οι π ί θ η κ ο ι

2 2 2 Ο πρώτος πίθηκος στα - του χρόνου που είχε στη διάθεσή του δηλαδή - .2 14= - .

3 3 3 1 35 90 - = - = 1 5 60 60 '

λεπτό έφαγε 15 καρύδες αφού τις τρώει με ταχύτητα 1 0 στο λεπτό . Ύστερα βοήθησε το φίλο του να φάνε τις μπα­νάνες. Άρα από τον υπόλοιπο χρόνο που είχε στη διάθεσή του των 45 δευτερολέπτων έφαγε μπανάνες για 40 δεύτε-

, ' ' ' Δ λ δ ' ' 40 20

40 1 3 1 δ ' ' ' δ ' ρα αφου εφυγαν στο παρα πεντε. η α η εφαγε - . = - = - επει η ομως οι καρυ ες και οι μπανανες που 60 3 3

' ' θ 1 3 1

1 2 ' ' δ ' 'θ

1 30 λ ' ' ' συγκέντρωσαν ηταν ισαρι μες 1 5 - - = - μπανανες εφαγε ο ευτερος πι ηκος σε - επτα αρα η ταχυτητα

3 3 60 2 1 30 5 1 3 5 6 1 ο ' ' που τις έτρωγε ήταν 1 - : - = - : - = - . - = - μπανανες το λεπτο. 3 60 3 6 3 1 3 1 3

�ba

--Jf-b Το Ι είναι

ίσο με

το - 1 ; Δεν ισχύει η ιδιότητα 'Jb ν 0

Πότε γεννήθηκε; Τέλειο τετράγωνο είναι μόνο το έτος 2025 = 452 δηλαδή ο καθηγητής το 2025 θα είναι 45 ετών άρα γεννήθηκε το 1 980 και σήμερα είναι 30 ετών. Τα ψάρια Οι φίλοι έφαγαν μαζί με τις γυναίκες τους 5+3=8 κιλά ψάρια. Κάθε ζεύγος έφαγε 8/3 κιλά. Ο Χρίστος τα

υπολόγισε με 30 € το κιλό που πούλησε τα δικά του έτσι τους έδωσε να μοιραστούν � . 30=80€. Άρα ο Θωμάς θα 3

' 8 7 ' 8 1 παρει (5- - ) .30= - .30=70€ και ο Σταυρος (3- - ) .30= - .30= 1 0€ .

3 3 3 3 Ο Χυμός Η κορυφή του κώνου είναι στη βάση του ποτηριού και η βάση του είναι τα χείλη του δηλαδή κύκλος α-

R κτίνας R. Όταν το γεμίσουμε μέχρι τη μέση η επιφάνεια της πορτοκαλάδας είναι κυκλικός δίσκος ακτίνας - . Ο

2

συνολικός όγκος του ποτηριού είναι V= _!_ πR2.h Με το χυμό από ένα πορτοκάλι φτάνει στη μέση άρα ο όγκος του 3

λ , , 1

( R

)2 h 1 1 2 h

1 , 'ζ , 8 λ πορτοκα ιου ειναι Υπ= - π - . - = - . - πR . = - V. Το ποτηρι γεμι ει με χυμο πορτοκα ιών. 3 2 2 8 3 8

ΕΥΚΛΕΙΔΗΣ Α ' 75 τ.3/81